Med Surg HESI Comprehensive

Réussis tes devoirs et examens dès maintenant avec Quizwiz!

A client is taking gentamicin sulfate for the treatment of pelvic inflammatory disease. What does the nurse ask the client during assessment for adverse effects of the medication? "When was your last bowel movement?" "When was your last menstrual period?" "Are you having any difficulty hearing?" "Are you having any difficulty breathing?"

"Are you having any difficulty hearing?" Rationale: Serious adverse reactions to aminoglycosides include ototoxicity and nephrotoxicity. Gentamicin sulfate is an aminoglycoside. It inhibits bacterial protein synthesis and has a bactericidal effect. The nurse must assess the client for changes in hearing, balance, and urine output. The remaining assessment questions are not associated with the adverse effects of this medication.

A nurse is performing an assessment of a client with Ménière disease. Which question does the nurse ask to elicit data about the manifestations of this disease? "Do you have headaches?" "Have you had any loss of appetite?" "Do you have episodes of dizziness?" "Have you been having any diarrhea?"

"Do you have episodes of dizziness?" Rationale: Ménière disease or syndrome, also called idiopathic endolymphatic hydrops, occurs when the normal fluid and electrolyte balance of the inner ear is disrupted. The classic triad consists of episodic true vertigo (dizziness), sensorineural hearing loss, and tinnitus. Some clients experience vertigo associated with nausea, vomiting, and ataxia. Diarrhea, headaches, and loss of appetite are not associated with this disease.

Calcium disodium edetate and dimercaprol is prescribed for a child with lead poisoning. What does the nurse ask the child's mother before administering the medications? "Can your child swallow pills?" "Has your child been running a fever?" "Does your child have an allergy to peanuts?" "How long has your child been exposed to the lead?"

"Does your child have an allergy to peanuts?" Rationale: Dimercaprol must not be used in the presence of a glucose-6-phosphate dehydrogenase deficiency (G6PD) or peanut allergy, nor should it be given in conjunction with iron. Therefore, the nurse must ask about allergy to peanuts. Dimercaprol may be used in conjunction with EDTA to treat lead poisoning. Dimercaprol is administered by way of deep intramuscular injection. Calcium disodium edetate is administered by way of the intravenous or intramuscular route. The assessment questions noted in the remaining options are unrelated to the administration of this medication.

A hospitalized female client demonstrating mania enters the unit community room and says to a client who is wearing a blue shirt, "Boys in blue are fun to do! Boys in blue are fun to do!" What is the appropriate response by the nurse? "Why are you saying that?" "Stop saying that. It's not true!" "You wouldn't like someone saying that to you. Would you?" "Don't say that. If you can't control yourself, we'll help you."

"Don't say that. If you can't control yourself, we'll help you." Rationale: The nurse should respond using a firm, calm approach, providing the client with clear expectations. The appropriate response is the only one that involves a firm, calm approach and offers the client help if she needs it. The other three statements challenge the client.

A nurse, performing an assessment of a client who has been admitted to the hospital with suspected silicosis, is gathering both subjective and objective data. Which question by the nurse would elicit data specific to the cause of this disorder? "Do you chew tobacco?" "Do you smoke cigarettes?" "Have you ever worked in a mine?" "Are you frequently exposed to paint products?"

"Have you ever worked in a mine?" Rationale: Silicosis is a chronic fibrotic disease of the lungs caused by the inhalation of free crystalline silica dust over a long period. Mining and quarrying are each associated with a high incidence of silicosis. Hazardous exposure to silica dust also occurs in foundry work, tunneling, sandblasting, pottery-making, stone masonry, and the manufacture of glass, tile, and bricks. The finely ground silica used in soaps, polishes, and filters also presents a risk. The assessment questions noted in the other options are unrelated to the cause of silicosis.

A client with diabetes mellitus calls the clinic nurse and reports that he/she has been vomiting during the night and now has diarrhea. Which question does the nurse make a priority of asking the client? "Do you have a fever?" "Did you eat any breakfast?" "Are you urinating frequently?" "Have you tested your blood glucose?"

"Have you tested your blood glucose?" Rationale: The priority is learning whether the client has tested his/her blood glucose level and what the level is. During illness, the client should follow the "sick-day rules" for diabetics. The blood glucose level should be checked every 4 hours. Urine is tested for ketones if the blood glucose level is greater than 240 mg/dL (13.3 mmol/L). The client should take the prescribed insulin or oral antidiabetic medication and should drink 8 to 12 oz (135 to 350 mL) of sugar-free liquids every hour when awake. The client should notify the primary health care provider if he/she becomes ill. The remaining options are questions that may be asked, but they are not the priority.

Tolbutamide is prescribed to a client whose type 2 diabetes mellitus has not been controlled with diet and exercise alone. The nurse provides instructions to the client about the medication. Which statement by the client indicates a need for further instruction? "I need to maintain my exercise program." "I need to stay on my calorie-restricted diet." "I can take the medication with food if it upsets my stomach." "I can have a beer or glass of wine as long as I stay within my daily dietary restrictions."

"I can have a beer or glass of wine as long as I stay within my daily dietary restrictions." Rationale: There is a need for further instruction when the client says "I can have a beer or glass of wine as long as I stay within my daily dietary restrictions." Tolbutamide, a first-generation sulfonylurea, is used as an adjunct to calorie restriction and exercise to maintain glycemic control in clients with type 2 diabetes mellitus. When alcohol is combined with tolbutamide, a disulfiram-like reaction, including flushing, palpitations, and nausea, may occur. Alcohol also potentiates the hypoglycemic effects of tolbutamide. The client must be told about the dangers of alcohol consumption. The statements in the remaining options are correct.

As a nurse prepares to administer medications to an assigned client, the client asks, "Why don't you just leave me alone?" What is the best response by the nurse? "Don't yell at me." "These medications will help you feel much better." "Why do you want to be left alone? I'm here to help you." "I can see that you're upset. Would you like to talk about it?"

"I can see that you're upset. Would you like to talk about it?" Rationale: The best response is, "I can see that you're upset. Would you like to talk about it?" The nurse addresses the client's feelings. The client may not want to talk about his/her feelings, but the nurse has provided the opportunity to the client and is maintaining an open channel of communication. "Don't yell at me" is confrontational and nonfacilitative and imposes control. "These medications will help you feel much better" avoids the client's feelings. "Why do you want to be left alone? I'm here to help you" is not a facilitative response and may place the client, who may not be able to express his or her feelings, on the defensive.

A client experiencing delusions says to the nurse, "I am the only one who can save the world from all of the terrorists." What is the appropriate response by the nurse? "Tell me your plan for saving the world." "Why do you think that you can accomplish this by yourself?" "I don't think anyone can save the world from the terrorists by himself/herself." "You must be powerful. Do you really believe that you can do this by yourself?"

"I don't think anyone can save the world from the terrorists by himself/herself." Rationale: The appropriate response by the nurse is "I don't think anyone can save the world from the terrorists by himself/herself." The nurse should not go along with or reinforce the client's delusion. The nurse should respond to the client by presenting reality. "Tell me your plan for saving the world," "Why do you think that you can accomplish this by yourself?" and "You must be powerful. Do you really believe you can do this by yourself?" all reinforce the delusion and encourage further conversation about it.

The nurse is giving discharge instructions to a client recently diagnosed with hypothyroidism. The client was also given a prescription for levothyroxine by the primary health care provider. What comment by the client indicates a need for further instruction? "I'm glad constipation won't be such a problem anymore." "I'll remember to take the medication without food in the morning." "I'm looking forward to maybe losing some weight with this medication." "I know I have to notify the primary health care provider if I have any problems with a slow heart rate."

"I know I have to notify the primary health care provider if I have any problems with a slow heart rate." Rationale: There is a need for further instruction if the client says, "I know I have to notify the primary health care provider if I have any problems with a slow heart rate." The primary health care provider must be notified if the client has any problems with tachycardia or fast heart rate, not slow heart rate. Excessive doses of levothyroxine sodium can produce signs/symptoms of hyperthyroidism. These include tachycardia, chest pain, tremors, nervousness, insomnia, hyperthermia, extreme heat intolerance, and sweating. The other comments are correct.

The nurse is instructing a client who has been diagnosed with myasthenia gravis. What comment by the client indicates a need for further instruction? "I know that treatment for myasthenia gravis is life-long." "I need to take medications on time to help prevent weakness." "I must take my medications about 45 minutes before eating a meal." "I should take the medications with a very small amount of food to prevent gastrointestinal (GI) symptoms."

"I must take my medications about 45 minutes before eating a meal." Rationale: There is a need for further instruction if the client states, "I must take my medications about 45 minutes before eating a meal." The client needs to eat a meal 45 to 60 minutes after, not before, taking medications to decrease the risk for aspiration. The other client statements are correct. Myasthenia gravis is a disease that affects the myoneural junction. It is believed to result from an autoimmune response that destroys a variable number of acetylcholine receptors at the myoneural junction.

nurse provides instructions to a client who has been prescribed lithium carbonate for the treatment of bipolar disorder. Which of these statements by the client indicate a need for further instruction? Select all that apply. "I need to avoid salt in my diet." "It's fine to take any over-the-counter medication with the lithium." "I need to come back to the clinic to have my lithium blood level checked." "I should drink 2 to 3 quarts (1.9 to 2.8 litres) of liquid every day." "Diarrhea and muscle weakness are to be expected, and if these occur I don't need to be concerned."

"I need to avoid salt in my diet." "It's fine to take any over-the-counter medication with the lithium." "Diarrhea and muscle weakness are to be expected, and if these occur I don't need to be concerned." Rationale: Lithium carbonate is a mood stabilizer used to treat manic-depressive illness. Equilibrium of sodium and potassium must be maintained at the intracellular membrane to maintain therapeutic effects. Lithium competes with sodium in the cell. Therefore the client should maintain a normal salt intake and drink 2 to 3 quarts (1.9 to 2.8 litres) of fluid each day. Many over-the-counter medications contain sodium and would therefore affect the lithium concentration, possibly pushing it out of the therapeutic range. For this reason, over-the-counter medications must be avoided. The blood level of lithium should be tested every 3 or 4 days during the initial phase of therapy and every 1 to 2 months during maintenance therapy. Vomiting, diarrhea, muscle weakness, tremors, drowsiness, and ataxia are signs/symptoms of toxicity; if any of these problems occur, the primary health care provider must be notified.

A nurse provides instructions to a client who is preparing for discharge after a radical vulvectomy for the treatment of cancer. Which statement by the client indicates a need for further instruction? "I can resume sexual activity in 4 to 6 weeks." "I need to avoid straining when I have a bowel movement." "I should wear support hose for 6 months and elevate my legs frequently." "I need to contact my surgeon immediately if I feel any numbness in my genital area."

"I need to contact my surgeon immediately if I feel any numbness in my genital area." Rationale: After radical vulvectomy, the client is instructed to wear support hose for 6 months and to elevate the legs frequently. The client should avoid straining during defecation and should be told that alteration in the direction of urine flow may occur. The client may resume sexual activity in 4 to 6 weeks; the nurse should discuss the possible need for lubrication and position changes during coitus. Genital numbness may be present, but it is not necessary to notify the surgeon immediately if numbness occurs.

A client is scheduled to undergo an upper gastrointestinal (GI) series, and the nurse provides instructions to the client about the test. Which statement by the client indicates a need for further instruction? "The test will take about 30 minutes." "I need to fast for 8 hours before the test." "I need to drink citrate of magnesia the night before the test and give myself a Fleet enema on the morning of the test." "I need to take a laxative after the test is completed, because the liquid that I'll have to drink for the test can be constipating."

"I need to drink citrate of magnesia the night before the test and give myself a Fleet enema on the morning of the test." Rationale: No special preparation is necessary before a GI series, except that NPO (nothing by mouth) status must be maintained for 8 hours before the test. An upper GI series involves visualization of the esophagus, duodenum, and upper jejunum by means of the use of a contrast medium. It involves swallowing a contrast medium (usually barium), which is administered in a flavored milkshake. Films are taken at intervals during the test, which takes about 30 minutes. After an upper GI series, the client is prescribed a laxative to hasten elimination of the barium. Barium that remains in the colon may become hard and difficult to expel, leading to fecal impaction.

A nurse has given a client with viral hepatitis instructions about home care. Which statement by the client indicates to the nurse that the client needs further teaching? "I can't drink alcohol." "I have to avoid having sex until the test for antibodies comes back negative." "I need to rest a lot during the day and get enough sleep at night." "I need to eat three meals a day with foods high in protein, fat, and carbs."

"I need to eat three meals a day with foods high in protein, fat, and carbs." Rationale: The client needs further teaching if the client states, "I need to eat three meals a day with foods high in protein, fat, and carbs." The client with viral hepatitis should consume a high-carbohydrate, low-fat diet, not a diet high in fat. The client should avoid hepatotoxic substances such as alcohol. Sexual intercourse is avoided until antibody testing results are negative. The client with hepatitis is easily fatigued and may require several weeks to reach his or her former activity level. It is important for the client to get adequate rest both during the day and at night so that the liver may heal.

A client with type 1 diabetes mellitus is instructed by the primary health care provider to obtain glucagon hydrochloride for emergency home use. The nurse provides information to the client's wife about the medication. Which statement by the client's wife indicates that she understands the information? "I need to store this medication in the freezer." "I know that this is used to treat episodes of high blood sugar." "I can give this medication instead of insulin if his insulin runs out." "I need to give this if he has signs of low blood sugar and goes into a coma."

"I need to give this if he has signs of low blood sugar and goes into a coma." Rationale: The statement by the client's wife that indicates she understands the information is, "I need to give this medication if he has signs of low blood sugar and goes into a coma." Glucagon is used to treat hypoglycemia resulting from insulin overdose. If hypoglycemic coma develops, glucagon is administered promptly. The family of the client is instructed in how to administer the medication. In an unconscious client, arousal usually occurs within 20 minutes of injection. Once consciousness has been produced, oral carbohydrates should be given. Insulin should never be stored in the freezer. Glucagon is not used to treat hyperglycemia or used in place of insulin; also, the client should be instructed to have additional vials of insulin on hand at home so that he will not run out. Glucagon is stored at room temperature.

A nurse is providing instruction about insulin therapy and its administration to an adolescent client who has just been found to have diabetes mellitus. Which statement by the client indicates a need for further instruction? "It's important to rotate injection sites." "I need to store the insulin in a cool, dry place." "I need to keep any unopened bottles of insulin in the freezer." "I need to check the expiration date on the insulin before I use it."

"I need to keep any unopened bottles of insulin in the freezer." Rationale: There is a need for further instruction if the client states, "I need to keep any unopened bottles of insulin in the freezer." Insulin is stored in a cool, dry place. It should not be placed in the freezer or exposed to excess heat or agitation. Injection sites should be rotated to ensure adequate insulin absorption and to prevent complications of insulin administration. Once a bottle of insulin has been opened, it is dated and discarded as recommended. The client should check the expiration date on the insulin vial before using it.

A nurse provides home care instructions to a client with coronary artery disease (CAD) who is being discharged from the hospital. Which statement by the client indicates a need for further instruction? "I need to carry my nitroglycerin with me at all times." "I need to check my pulse before, during, and after exercise." "I need to avoid foods with saturated fats and foods high in cholesterol." "I need to participate in aerobic and weightlifting exercise three times a week."

"I need to participate in aerobic and weightlifting exercise three times a week." Rationale: There is a need for further instruction if the client states, "I need to participate in aerobic and weightlifting exercise three times a week." The client should avoid activities that involve straining, including weightlifting, push-ups and pull-ups, and straining during bowel movements. The client with CAD should participate in a simple exercise program on a regular basis. The client may begin a simple walking program by walking 400 feet (122 metres) twice a day at a rate of 1 mph (1.6 km/hr) the first week after discharge and increasing the distance and rate as tolerated, usually weekly, until he or she can walk 2 miles (3.2 km) at 3 to 4 mph (4.8 to 6.4 km/hr). The client should always carry nitroglycerin and must comply with dietary restrictions, including avoiding foods with saturated fats and foods high in cholesterol. The nurse instructs the client to take a pulse reading before, halfway through, and after exercise.

Colchicine has been prescribed for a client with a diagnosis of gout, and the nurse provides information to the client about the medication. Which statement by the client indicates to the nurse that the client understands the information "I need to limit my intake of fluids while I'm taking this medication." "I need to stop the medication and call my doctor if I have severe diarrhea." "I can expect skin redness and a rash when I take this medication." "I may get a burning feeling in my throat, but it's normal and will go away."

"I need to stop the medication and call my doctor if I have severe diarrhea." Rationale: The client understands the information if stating, "I need to stop the medication and call my doctor if I have severe diarrhea." The client is instructed to report a rash, sore throat, fever, unusual bruising or bleeding, weakness, tiredness, or numbness. A burning sensation in the throat or skin, severe diarrhea, and abdominal pain are signs of overdose. Colchicine is classified as an antigout agent. It interferes with the capacity of the white blood cells to initiate and maintain an inflammatory response to monosodium urate crystals. The client should maintain a high fluid intake (eight to ten 8-oz [235 mL] glasses of fluid per day) while taking the medication.

A nurse provides information about smoking-cessation measures to a client diagnosed with coronary artery disease (CAD). Which statement by the client indicates a need for further information? "A community support group will help me quit." "I should drink a cup (235 ml) of coffee if I feel the urge to smoke." "Relaxation exercises will help control my urge to smoke." "I can try chewing gum or sucking on hard candy if I feel the urge to smoke."

"I should drink a cup (235 ml) of coffee if I feel the urge to smoke." Rationale: The nurse should reinforce education about the hazards of smoking and provide encouragement to clients who are interested in or willing to try smoking cessation. The client should avoid using coffee and other caffeine-containing products, which can cause restlessness and anxiety, increasing the urge to smoke. There are appropriate strategies to assist in smoking cessation. Among these are community support groups, relaxation exercises, and strategies such as chewing gum or sucking on hard candy if the urge to smoke arises.

A nurse provides information to a client with chronic obstructive pulmonary disease (COPD) about methods of alleviating shortness of breath while the client is eating. Which statement by the client indicates a need for further instruction? "I should rest before I eat." "I should use my bronchodilator 30 minutes before I eat." "Pursed-lip breathing will help relieve my shortness of breath." "I should eat three meals a day, and the biggest meal should be at suppertime."

"I should eat three meals a day, and the biggest meal should be at suppertime." Rationale: There is a need for further instruction if the client states, "I should eat three meals a day, and the biggest meal should be at suppertime." The biggest meal of the day is planned for the time when the client is hungriest and most rested. Most clients are increasingly tired at the end of the day. Four to six small meals per day are preferable to three larger ones. Pursed-lip or abdominal breathing may alleviate dyspnea.Shortness of breath is the most common problem related to eating for a client with COPD. The client can ease dyspnea by resting before meals. A bronchodilator used 30 minutes before a meal may be helpful if the meal-related dyspnea is a result of bronchospasm or secretions.

A nurse provides information about activity and exercise to the wife of a client with Parkinson's disease. Which statement by the spouse indicates a need for further instruction? "He needs to have a broad base of support when ambulating." "He needs to avoid staying in one position for a prolonged period." "I should encourage him to keep his hands hanging at his side when he walks." "I should help him perform range-of-motion exercises of his joints three times a day."

"I should encourage him to keep his hands hanging at his side when he walks." Rationale: There is a need for further instruction if the client's spouse states, "I should encourage him to keep his hands hanging at his side when he walks." Parkinson's disease is a chronic degenerative disorder that primarily affects the neurons of the basal ganglia. The client's muscles become stiff and rigid and require increased effort to move. The client is taught to hold the hands clasped behind the back when walking to keep the spine erect and counter the problems that may result when the arms hang stiffly at the sides. The statements in the other options indicate correct understanding of measures regarding activity and exercise for the client with Parkinson's disease.

Oral candidiasis (thrush) develops in a client infected with HIV, and the nurse provides instruction to the client about measures to relieve the discomfort. Which statement by the client indicates a need for further instruction? "I should avoid spicy foods." "I should eat foods with a soft texture." "I should use a soft-bristled toothbrush." "I should put ice in my drinks to help soothe the discomfort."

"I should put ice in my drinks to help soothe the discomfort." Rationale: There is a need for further instruction if the client states, "I should put ice in my drinks to help soothe the discomfort." Oral candidiasis lesions make it difficult for the client to tolerate temperature extremes, so it is best for the client to consume foods and fluids that are tepid or room temperature rather than iced or hot. The other options are useful measures for alleviating the discomfort associated with this disorder.

A nurse assigns an assistive personnel (AP) to care for a client who is hearing impaired and provides instructions to the AP about the effective methods for communicating with the client. Which statement by the AP indicates that further instruction is needed? "I should speak slowly and clearly to the client." "I should stand directly in front of the client when I'm talking." "I should make sure that the room is well lit when I'm talking to the client." "I should raise the volume of my voice and stand on the client's affected side when I'm talking to him."

"I should raise the volume of my voice and stand on the client's affected side when I'm talking to him." Rationale: Further instruction is needed if the AP states, "I should raise the volume of my voice and stand on the client's affected side when I'm talking to him." When communicating with a hearing-impaired client, the nurse should position herself or himself directly in front of the client so that the client may read the nurse's lips. The room should be well lit, and the nurse should get the client's attention before speaking. The nurse should move closer to the ear with better hearing and speak slowly and clearly. The nurse should speak in a normal tone of voice and keep his or her hands and other objects away from the mouth while talking to the client.

A client has been scheduled for an electronystagmography (ENG), and the nurse provides instructions to the client about the test. Which statement by the client tells the nurse that the client understands the instructions? "I shouldn't drink coffee before the test." "I'll need to receive sedation before the test." "I won't be able to eat for 24 hours after the test." "I can eat a light breakfast on the morning of the test."

"I shouldn't drink coffee before the test." Rationale: The client understands the instructions for an ENG if the client states, "I shouldn't drink coffee before the test." The client must fast for several hours (but not for 24 hours) before the test and should avoid caffeine-containing products for 24 to 48 hours before the test. An ENG is a test that is sensitive in detecting both central and peripheral disease of the vestibular system in the ear. It detects nystagmus (involuntary eye movements), which can be recorded. After electrodes are taped to the skin near the eyes, procedures are performed to stimulate nystagmus. No sedation is used for the ENG. Fluids are given to the client after the test but are introduced carefully to prevent nausea.

A nurse is teaching a client with angina pectoris who is being discharged from the hospital about managing chest pain at home. Which statement by the client indicates a need for further teaching? "I need to keep fresh nitroglycerin available in case I need it." "I need to check the expiration date on the nitroglycerin bottle." "If I have any chest pain, I need to stop what I am doing and sit or lie down." "If I get chest pain, I should put 3 nitroglycerin tablets under my tongue and then go to the emergency department if that doesn't work."

"If I get chest pain, I should put 3 nitroglycerin tablets under my tongue and then go to the emergency department if that doesn't work." Rationale: There is a need for further teaching if the client states, "If I get chest pain, I should put 3 nitroglycerin tablets under my tongue and then go to the emergency department if that doesn't work." The client is instructed to place 1 nitroglycerin tablet, not 3 tablets, under the tongue and allow it to dissolve. The client needs to obtain instructions from the primary health care provider about the preferred procedure for administering nitroglycerin. The remaining statements reflect accurate understanding of the nurse's teaching.

A nurse has provided nutrition instructions to a mother of an infant. Which statement by the mother indicates to the nurse that the mother requires further instruction? "It's best to use cow's milk, as long as it's whole milk and not skim." "When I start feeding solid foods, I might need to add water to the food." "When the baby starts to take juices, I shouldn't warm the juice, because that will destroy the vitamin C." "The baby will get the right nutrition if I feed breast milk or store-bought formula that's been fortified with iron."

"It's best to use cow's milk, as long as it's whole milk and not skim." Rationale: Cow's milk (whole, skim, 1%, 2%) is not recommended in the first 12 months of life. It contains too little iron, its high renal solute load and unmodified derivatives can put small infants at risk for dehydration, and the tough, hard curd is difficult for infants to digest. In addition, skim milk and reduced-fat milk deprive the infant of needed calories and essential fatty acids. Breast milk or commercially prepared iron-fortified formula should be used to feed the infant and provides adequate nutrition throughout the infancy period. The remaining options are correct statements about feeding an infant.

A rape victim being treated in the emergency department says to the nurse, "I'm really worried that I've got HIV now." What is the most appropriate response by the nurse? "HIV is rarely an issue in rape victims." "Every rape victim is concerned about HIV." "You're more likely to get pregnant than to contract HIV." "Let's talk about the information that you need to determine your risk of contracting HIV."

"Let's talk about the information that you need to determine your risk of contracting HIV." Rationale: The most appropriate response by the nurse is the one that encourages the client to talk about her condition. HIV is a concern of rape victims. Such concern should always be addressed, and the victim should be given the information needed to evaluate his or her risk. Pregnancy may occur as a result of rape, and pregnancy prophylaxis can be offered in the emergency department or during follow-up, once the results of a pregnancy test have been obtained. However, stating, "You're more likely to get pregnant than to contract HIV" avoids the client's concern. Similarly, "HIV is rarely an issue in rape victims" and "Every rape victim is concerned about HIV" are generalized responses that avoid the client's concern.

client diagnosed with type 1 diabetes mellitus has just been told that she is 6 weeks pregnant. The nurse provides information to the client about dietary and insulin needs. What comment by the client suggests an understanding of the information?"I know I will have to increase my insulin during this time period." "My insulin needs should decrease during the first trimester." "Needs for insulin will not change during the first 3 months of pregnancy." "I will have to double up on the insulin dose during this time span."

"My insulin needs should decrease during the first trimester." Rationale: Insulin needs generally decrease during the first trimester of pregnancy because the secretion of placental hormones antagonistic to insulin remains low. An increase in insulin need, lack of change in insulin need, and doubling of insulin need are all incorrect.

A client diagnosed with depression is being encouraged to attend art therapy as part of the treatment plan. The client refuses, stating, "I can't draw or paint." Which response by the nurse is therapeutic? "Why don't you really want to attend?" "This is what your primary health care provider has prescribed for you as part of the treatment plan." "OK, let's have you attend music therapy. You can sing there. How does that sound?" "Perhaps you could attend and talk to the other clients and see what they're drawing and painting."

"Perhaps you could attend and talk to the other clients and see what they're drawing and painting." Rationale: The correct response encourages the client to socialize and deflects the client's attention from the issue of drawing and painting. "Why don't you really want to attend?" challenges the client. "This is what your primary health care provider has prescribed for you as part of the treatment plan" ignores the client's rights. "OK, let's have you attend music therapy. You can sing there. How does that sound?" does not address the client's concern.

A client is taking prescribed ibuprofen 200 mg orally four times daily, to relieve joint pain resulting from rheumatoid arthritis. The client tells the nurse that the medication is causing nausea and indigestion. What should the nurse tell the client? "I will contact your primary health care provider." "Stop taking the medication." "Take the medication with food." "Take the medication twice a day instead of four times a day."

"Take the medication with food." Rationale: Ibuprofen is a nonsteroidal antiinflammatory medication. Side effects include nausea (with or without vomiting) and dyspepsia (heartburn, indigestion, or epigastric pain). If gastrointestinal distress occurs, the client should be instructed to take the medication with milk or food. The nurse would not instruct the client to stop the medication or instruct the client to adjust the dosage of a prescribed medication; these actions are not within the legal scope of the role of the nurse. Contacting the primary health care provider is premature, because the client's complaints are side effects that occasionally occur and can be relieved by taking the medication with milk or food.

A client diagnosed with post-traumatic stress disorder tells the nurse that he/she has stopped taking his/her prescribed medication because he/she didn't like how the medication was making him/her feel. Which initial response by the nurse is appropriate? "That's all right. I'd stop, too, if it made me feel funny." "Tell me more about how the medication was making you feel." "Did you let your doctor know that you stopped taking the medication?" "It doesn't make sense to stop the medication. I don't know why you took it upon yourself to do that."

"Tell me more about how the medication was making you feel." Rationale: The appropriate response by the nurse acknowledges the client's feelings and opens the channel of communication between the nurse and client. "That's all right. I'd stop, too, if it made me feel funny," indicating approval, is a nontherapeutic response and is therefore inappropriate. "Did you let your doctor know that you stopped taking the medication?" may be an appropriate question at some point during the conversation, but it is not the most appropriate initial question. "It doesn't make sense to stop the medication. I don't know why you took it upon yourself to do that" demeans the client.

A client who recently underwent coronary artery bypass graft surgery comes to the primary health care provider's office for a follow-up visit. On assessment, the client tells the nurse that he is feeling depressed. Which response by the nurse is therapeutic? "Tell me more about what you're feeling." "That's a normal response after this type of surgery." "It will take time, but I promise you, you will get over this depression." "Every client who has this surgery feels the same way for about a month."

"Tell me more about what you're feeling." Rationale: The therapeutic response by the nurse is, "Tell me more about what you're feeling." When a client expresses feelings of depression, it is extremely important for the nurse to further explore these feelings with the client. In stating, "This is a normal response after this type of surgery" the nurse provides false reassurance and avoids addressing the client's feelings. "It will take time, but I promise you, you will get over the depression" is also a false reassurance, and it does not encourage the expression of feelings. "Every client who has this surgery feels the same way for about a month" is a generalization that avoids the client's feelings.

A client who has undergone extensive gastrointestinal surgery is receiving intermittent enteral tube feedings that will be continued after he/she is discharged home. When the nurse tells the client that he/she will be taught how to administer the feedings, the client states, "I don't think I'll be able to do these feedings by myself." Which response by the nurse is appropriate? "Have you told your doctor how you feel?" "Tell me more about your concerns regarding the tube feedings." "Don't worry. We'll keep you in the hospital until you're ready to do them by yourself." "We'll ask the doctor about having a visiting nurse come to your home to give you your feedings."

"Tell me more about your concerns regarding the tube feedings." Rationale: An open statement such as "Tell me more about..." often elicits valuable information about the client and the client's concerns. Clients often have fears about leaving the secure environment of the healthcare facility, where he/she is cared for. This client fears that he/she will not be able to care for self at home by administering the tube feedings. The remaining nursing responses are incorrect because they are nontherapeutic statements that do not address the client's expressed concern.

A maternity nurse providing an education session to a group of expectant mothers describes the purpose of the placenta. Which statement by one of the women attending the session indicates a need for further discussion of the purpose of the placenta? "Many of my antibodies are passed through the placenta." "The placenta maintains the body temperature of my baby." "Glucose, vitamins, and electrolytes pass through the placenta." "It provides an exchange of oxygen and carbon dioxide between me and my baby."

"The placenta maintains the body temperature of my baby." Rationale: There is a need for further discussion if the woman states that the placenta maintains the body temperature of the baby. Many of the immunoglobulin G (IgG) class of antibodies are passed from mother to fetus through the placenta. Glucose, fatty acids, vitamins, and electrolytes pass readily across the placenta; glucose is the major source of energy for fetal growth and metabolic activities. The placenta provides an exchange of nutrients and waste products between the mother and fetus. Oxygen and carbon dioxide pass through the placental membrane by way of simple diffusion. The amniotic fluid surrounds, cushions, and protects the fetus and maintains the body temperature of the fetus.

A child is brought to the emergency department by ambulance after swallowing several capsules of acetaminophen. What statement by the nurse indicates a need for further information? "We need to administer the antidote N-acetyl cysteine and dilute it in juice or soda." "A loading dose of N-acetyl cysteine has to be followed by maintenance doses." Incorrect "We need to give N-acetyl cysteine before we do gastric lavage with activated charcoal." "If the child is unconscious, we must do gastric lavage with activated charcoal to decrease the absorption of acetaminophen."

"We need to give N-acetyl cysteine before we do gastric lavage with activated charcoal." Rationale: There is a need for further information if the nurse states, "We need to give N-acetyl cysteine before we do gastric lavage with activated charcoal." Activated charcoal with lavage is done if the child is unconscious, but N-acetyl cysteine cannot be used because activated charcoal inactivates the antidote. If given orally, it can be diluted in juice or soda, and a loading dose of N-acetyl cysteine must be followed by maintenance doses.

A child who has just been found to have scoliosis will need to wear a thoracolumbosacral orthotic (TLSO) brace, and the nurse provides information to the mother about the brace. Which statement by the mother indicates a need for further information? "My child will need to do exercises." "My child needs to wear the brace 18 to 23 hours per day." "Wearing the brace is really important in curing the scoliosis." "I need to check my child's skin under the brace to be sure it doesn't break down."

"Wearing the brace is really important in curing the scoliosis." Rationale: Scoliosis is a lateral curvature of the spine. There is a need for further information when the mother says, "Wearing the brace is really important in curing the scoliosis." Bracing is not curative of scoliosis but may slow the progression of the curvature to allow skeletal growth and maturation. A brace needs to be worn 18 to 23 hours a day, but it may be removed at night for sleep if this is prescribed. To be more cosmetically acceptable, a brace is usually worn under loose-fitting clothing. Back exercises are important in maintaining and strengthening the abdominal and spinal muscles. The child's skin must be meticulously monitored for signs of breakdown.

A nurse is providing morning care to a client in end-stage kidney disease. The client is reluctant to talk and shows little interest in participating in hygiene care. Which statement by the nurse would be therapeutic? "What are your feelings right now?" "Why don't you feel like washing up?" "You aren't talking today. Cat got your tongue?" "You need to get yourself cleaned up. You have company coming today."

"What are your feelings right now?" Rationale: Asking, "What are your feelings right now?" encourages the client to identify his or her emotions or feelings, which is a therapeutic communication technique. In stating, "Why don't you feel like washing up?" the nurse is requesting an explanation of feelings and behaviors for which the client may not know the reason. Requesting an explanation is a nontherapeutic communication technique. "You aren't talking today. Cat got your tongue?" is a nontherapeutic cliché. The statement "You need to get yourself cleaned up. You have company coming today" is demanding, demeaning to the client, and nontherapeutic.

A client diagnosed with schizophrenia says to the nurse, "I decided not to take my medication because it can't help. I am the only one who can help me." Which nursing response is therapeutic in this situation? "Only you can help?" "You decided not to take your medication?" "If you can make that observation, you probably don't need your medication any longer." "Your doctor wants you to continue this medication because it's helping you. Do you recall needing to be hospitalized because you stopped your medication?"

"Your doctor wants you to continue this medication because it's helping you. Do you recall needing to be hospitalized because you stopped your medication?" Rationale: Noncompliance with antipsychotic medication is one of the reasons clients with schizophrenia have relapses. The nurse should give a response to the schizophrenic client that will help the client identify the causes of relapse. The therapeutic response by the nurse is, "Your doctor wants you to continue this medication because it's helping you. Do you recall needing to be hospitalized because you stopped your medication?" In asking, "Only you can help?" the nurse is employing restating, which can be therapeutic but is not useful in this client's situation. "You decided not to take your medication?" is another example of restating. In stating, "If you can make this observation, you probably don't need your medication any longer," the nurse is using an illogical, judgmental, and biased response that is not therapeutic.

The nurse is developing standards of care for a client with gastroesophageal reflux disease and wants to review current evidence for practice. Which one of the following resources will provide the most helpful information? 1. A review in the Cochrane Library. 2. A literature search in a database, such as the Cumulative Index to Nursing and Allied Health Literature (CINAHL). 3. An online nursing textbook. 4. The policy and procedure manual at the health care agency.

1 . The Cochrane Library provides systematic reviews of health care interventions and will provide the best resource for evidence for nursing care. CINAHL offers key word searches to published articles in nursing and allied health literature, but not reviews. A nursing textbook has information about nursing care, which may include evidence-based practices, but textbooks may not have the most up-to-date information. While the policy and procedure manual may be based on evidence-based practices, the most current practices will be found in evidence-based reviews of literature.

After the administration of t-PA, the nurse should: 1. Observe the client for chest pain. 2. Monitor for fever. 3. Review the 12-lead electrocardiogram (ECG). 4. Auscultate breath sounds

1 Although monitoring the 12-lead ECG and monitoring breath sounds are important, observing the client for chest pain is the nursing assessment priority because closure of the previously obstructed coronary artery may recur. Clients who receive t- PA frequently receive heparin to prevent closure of the artery after administration of t- PA. Careful assessment for signs of bleeding and monitoring of partial thromboplastin time are essential to detect complications. Administration of t-PA should not cause fever.

The nurse is teaching the client about home blood glucose monitoring. Which of the following blood glucose measurements indicates hypoglycemia? 1. 59 mg/dL (3.3 mmol/L). 2. 75 mg/dL (4.2 mmol/L). 3. 108 mg/dL (6 mmol/L). 4. 119 mg/dL (6.6 mmol/L).

1 Although some individual variation exists, when the blood glucose level decreases to less than 70 mg/dL (3.9 mmol/L), the client experiences or is at risk for hypoglycemia. Hypoglycemia can occur in both type 1 and type 2 diabetes mellitus, although it is more common when the client is taking insulin. The nurse should instruct the client on the prevention, detection, and treatment of hypoglycemia.

The client probably has not progressed to an advanced stage. 98. The nurse is developing a discharge plan about home care with a client who has lymphoma. The nurse should emphasize which of the following? 1. Use analgesics as needed. 2. Take a shower with perfumed shower gel. 3. Wear a mask when outside of the home. 4. Take an antipyretic every morning.

1 Analgesics are used as needed to relieve painful encroachment of enlarged lymph nodes. Perfumed shower gel will increase pruritus. Wearing a mask does not protect the client from infection if pathogens are not spread by airborne droplets. Antipyretics should be used to treat fever symptomatically after infection is ruled out.

When monitoring a client who is receiving tissue plasminogen activator (t-PA), the nurse should have resuscitation equipment available because reperfusion of the cardiac tissue can result in which of the following? 1. Cardiac arrhythmias. 2. Hypertension. 3. Seizure. 4. Hypothermia.

1 Cardiac arrhythmias are commonly observed with administration of t-PA. Cardiac arrhythmias are associated with reperfusion of the cardiac tissue. Hypotension is commonly observed with administration of t-PA. Seizures and hypothermia are not generally associated with reperfusion of the cardiac tissue.

Which of the following is a risk factor for hypovolemic shock? 1. Hemorrhage. 2. Antigen-antibody reaction. 3. Gram-negative bacteria. 4. Vasodilation.

1 Causes of hypovolemic shock include external fluid loss, such as hemorrhage; internal fluid shifting, such as ascites and severe edema; and dehydration. Massive vasodilation is the initial phase of vasogenic or distributive shock, which can be further subdivided into three types of shock: septic, neurogenic, and anaphylactic. A severe antigen-antibody reaction occurs in anaphylactic shock. Gram-negative bacterial infection is the most common cause of septic shock. Loss of sympathetic tone (vasodilation) occurs in neurogenic shock.

The client with acute lymphocytic leukemia (ALL) is at risk for infection. The nurse should: 1. Place the client in a private room. 2. Have the client wear a mask. 3. Have staff wear gowns and gloves. 4. Restrict visitors

1 Clients with ALL are at risk for infection due to granulocytopenia. The nurse should place the client in a private room. Strict hand-washing procedures should be enforced and will be the most effective way to prevent infection. It is not necessary to have the client wear a mask. The client is not contagious and the staff does not need to wear gloves. The client can have visitors; however, they should be screened for infection and use hand-washing procedures.

The client admitted with peripheral vascular disease (PVD) asks the nurse why her legs hurt when she walks. The nurse bases a response on the knowledge that the main characteristic of PVD is: 1. Decreased blood flow. 2. Increased blood flow. 3. Slow blood flow. 4. Thrombus formation.

1 Decreased blood flow is a common characteristic of all PVD. When the demand for oxygen to the working muscles becomes greater than the supply, pain is the outcome. Slow blood flow throughout the circulatory system may suggest pump failure. Thrombus formation can result from stasis or damage to the intima of the vessels.

When assessing the client with Hodgkin's disease, the nurse should observe the client for which of the following findings? 1. Herpes zoster infections. 2. Discolored teeth. 3. Hemorrhage. 4. Hypercellular immunity.

1 Herpes zoster infections are common in clients with Hodgkin's disease. Discoloring of the teeth is not related to Hodgkin's disease but rather to the ingestion of iron supplements or some antibiotics such as tetracycline. Mild anemia is common in Hodgkin's disease, but the platelet count is not affected until the tumor has invaded the bone marrow. A cellular immunity defect occurs in Hodgkin's disease in which there is little or no reaction to skin sensitivity tests. This is called anergy.

The nurse is assessing an older adult with a pacemaker who leads a sedentary lifestyle. The client reports being unable to perform activities that require physical exertion. The nurse should further assess the client for which of the following? 1. Left ventricular atrophy. 2. Irregular heartbeats. 3. Peripheral vascular occlusion. 4. Pacemaker placement

1 In older adults who are less active and do not exercise the heart muscle, atrophy can result. Disuse or deconditioning can lead to abnormal changes in the myocardium of the older adult. As a result, under sudden emotional or physical stress, the left ventricle is less able to respond to the increased demands on the myocardial muscle. Decreased cardiac output, cardiac hypertrophy, and heart failure are examples of the chronic conditions that may develop in response to inactivity, rather than in response to the aging process. Irregular heartbeats are generally not associated with an older sedentary adult's lifestyle. Peripheral vascular occlusion or pacemaker placement should not affect response to stress.

The client asks the nurse to explain what it means that his Hodgkin's disease is diagnosed at stage 1A. Which of the following describes the involvement of the disease? 1. Involvement of a single lymph node. 2. Involvement of two or more lymph nodes on the same side of the diaphragm. 3. Involvement of lymph node regions on both sides of the diaphragm. 4. Diffuse disease of one or more extralymphatic organs

1 In the staging process, the designations A and B signify that symptoms were or were not present when Hodgkin's disease was found, respectively. The Roman numerals I through IV indicate the extent and location of involvement of the disease. Stage I indicates involvement of a single lymph node; stage II, two or more lymph nodes on the same side of the diaphragm; stage III, lymph node regions on both sides of the diaphragm; and stage IV, diffuse disease of one or more extralymphatic organs.

A client with diabetes is taking insulin lispro (Humalog) injections. The nurse should advise the client to eat: 1. Within 10 to 15 minutes after the injection. 2. 1 hour after the injection. 3. At any time, because timing of meals with lispro injections is unnecessary. 4. 2 hours before the injection.

1 Insulin lispro (Humalog) begins to act within 10 to 15 minutes and lasts approximately 4 hours. A major advantage of Humalog is that the client can eat almost immediately after the insulin is administered. The client needs to be instructed regarding the onset, peak, and duration of all insulin, as meals need to be timed with these parameters. Waiting 1 hour to eat may precipitate hypoglycemia. Eating 2 hours before the insulin lispro could cause hyperglycemia if the client does not have circulating insulin to metabolize the carbohydrate.

The client has been managing angina episodes with nitroglycerin. Which of the following indicate the drug is effective? 1. Decreased chest pain. 2. Increased blood pressure. 3. Decreased blood pressure. 4. Decreased heart rate

1 Nitroglycerin acts to decrease myocardial oxygen consumption. Vasodilation makes it easier for the heart to eject blood, resulting in decreased oxygen needs. Decreased oxygen demand reduces pain caused by heart muscle not receiving sufficient oxygen. While blood pressure may decrease ever so slightly due to the vasodilation effects of nitroglycerine, it is only secondary and not related to the angina the patient is experiencing. Increased blood pressure would mean the heart would work harder, increasing oxygen demand and thus angina. Decreased heart rate is not an effect of nitroglycerine.

A nurse is monitoring a hospitalized client who is being treated for preeclampsia. Which finding elicited during the assessment indicates that the condition has not yet resolved? Type the option number that is the correct answer. _____ Nursing Progress Notes Hyperreflexia is present. Urinary protein is not detectable. Urine output is 45 mL/hr. Blood pressure is 128/78 mm Hg.

1 Rationale: In a client with preeclampsia, deep tendon reflexes may be very brisk (hyperreflexia) and clonus (series of involuntary, rhythmic, muscular contractions and relaxations)may be present, suggesting cerebral irritability resulting from decreased brain circulation and edema. Hypertension, generalized edema, and proteinuria are the three classic signs of preeclampsia. Decreased urinary output (less than 30 mL/hr) indicates poor perfusion of the kidneys and may precede acute renal failure. Negative findings of the urinary protein assay, urine output of 45 mL/hr, and a blood pressure of 128/78 mm Hg are all signs that preeclampsia is resolving.

In providing care to the client with leukemia who has developed thrombocytopenia, the nurse assesses the most common sites for bleeding. Which of the following is not a common site? 1. Biliary system. 2. Gastrointestinal tract. 3. Brain and meninges. 4. Pulmonary system.

1 The biliary system is not especially prone to hemorrhage. Thrombocytopenia (a low platelet count) leaves the client at risk for a potentially life-threatening spontaneous hemorrhage in the gastrointestinal, respiratory, and intracranial cavities.

A male client expresses concern about how a hypophysectomy will affect his sexual function. Which of the following statements provides the most accurate information about the physiologic effects of hypophysectomy? 1. Removing the source of excess hormone should restore the client's libido, erectile function, and fertility. 2. Potency will be restored, but the client will remain infertile. 3. Fertility will be restored, but impotence and decreased libido will persist. 4. Exogenous hormones will be needed to restore erectile function after the adenoma is removed.

1 The client's sexual problems are directly related to the excessive prolactin level. Removing the source of excessive hormone secretion should allow the client to return gradually to a normal physiologic pattern. Fertility will return, and erectile function and sexual desire will return to baseline as hormone levels return to normal.

The nurse should teach the diabetic client that which of the following is the most common symptom of hypoglycemia? 1. Nervousness. 2. Anorexia. 3. Kussmaul's respirations. 4. Bradycardia.

1 The four most commonly reported signs and symptoms of hypoglycemia are nervousness, weakness, perspiration, and confusion. Other signs and symptoms include hunger, incoherent speech, tachycardia, and blurred vision. Anorexia and Kussmaul's respirations are clinical manifestations of hyperglycemia or ketoacidosis. Bradycardia is not associated with hypoglycemia; tachycardia is.

Propylthiouracil (PTU) is prescribed for a client with Graves' disease. The nurse should teach the client to immediately report which of the following? 1. Sore throat. 2. Painful, excessive menstruation. 3. Constipation. 4. Increased urine output.

1 The most serious adverse effects of PTU are leukopenia and agranulocytosis, which usually occur within the first 3 months of treatment. The client should be taught to promptly report to the health care provider signs and symptoms of infection, such as a sore throat and fever. Clients having a sore throat and fever should have an immediate white blood cell count and differential performed, and the drug must be withheld until the results are obtained. Painful menstruation, constipation, and increased urine output are not associated with PTU therapy.

A client with toxic shock has been receiving ceftriaxone sodium (Rocephin), 1 g every 12 hours. In addition to culture and sensitivity studies, which other laboratory findings should the nurse monitor? 1. Serum creatinine. 2. Spinal fluid analysis. 3. Arterial blood gases. 4. Serum osmolality.

1 The nurse monitors the blood levels of antibiotics, white blood cells, serum creatinine, and blood urea nitrogen because of the decreased perfusion to the kidneys, which are responsible for filtering out the Rocephin. It is possible that the clearance of the antibiotic has been decreased enough to cause toxicity. Increased levels of these laboratory values should be reported to the physician immediately. A spinal fluid analysis is done to examine cerebral spinal fluid, but there is no indication of central nervous system involvement in this case. Arterial blood gases are used to determine actual blood gas levels and assess acid-base balance. Serum osmolality is used to monitor fluid and electrolyte balance.

A client with a suspected diagnosis of Hodgkin's disease is to have a lymph node biopsy. The nurse should make sure that personnel involved with the procedure do which of the following when obtaining the lymph node biopsy specimen for histologic examination for this client? 1. Maintain sterile technique. 2. Use a mask, gloves, and a gown when assisting with the procedure. 3. Send the specimen to the laboratory when someone is available to take it. 4. Ensure that all instruments used are placed in a sealed and labeled container.

1 The nurse must ensure that sterile technique is used when a biopsy is obtained because the client is at high risk for infection. In most cases, a lymph node biopsy is sent immediately to the laboratory once it is placed in a specific solution in a closed container. It is not necessary to wear a gown and mask when obtaining the specimen. It is not necessary to use special handling procedures for the instruments used.

The best indicator that the client has learned how to give an insulin self injection correctly is when the client can: 1. Perform the procedure safely and correctly. 2. Critique the nurse's performance of the procedure. 3. Explain all steps of the procedure correctly. 4. Correctly answer a posttest about the procedure

1 The nurse should judge that learning has occurred from the evidence of a change in the client's behavior. A client who performs a procedure safely and correctly demonstrates that he has acquired a skill. Evaluation of this skill acquisition requires performance of that skill by the client with observation by the nurse. The client must also demonstrate cognitive understanding, as shown by the ability to critique the nurse's performance. Explaining the steps demonstrates acquisition of knowledge at the cognitive level only. A posttest does not indicate the degree to which the client has learned a psychomotor skill.

Which of the following individuals is most at risk for acquiring acute lymphocytic leukemia (ALL)? The client who is: 1. 4 to 12 years. 2. 20 to 30 years. 3. 40 to 50 years. 4. 60 to 70 years.

1 The peak incidence of ALL is at 4 years of age. ALL is uncommon after 15 years of age. The median age at incidence of CML is 40 to 50 years. The peak incidence of AML occurs at 60 years of age. Two-thirds of cases of chronic lymphocytic leukemia occur in clients older than 60 years of age.

The nurse's best explanation for why the severely neutropenic client is placed in reverse isolation is that reverse isolation helps prevent the spread of organisms: 1. To the client from sources outside the client's environment. 2. From the client to health care personnel, visitors, and other clients. 3. By using special techniques to dispose of contaminated materials. 4. By using special techniques to handle the client's linens and personal items.

1 The primary purpose of reverse isolation is to reduce transmission of organisms to the client from sources outside the client's environment.

Which of the following is a priority goal for the diabetic client who is taking insulin and has nausea and vomiting from a viral illness or influenza? 1. Obtaining adequate food intake. 2. Managing own health. 3. Relieving pain. 4. Increasing activity.

1 The priority goal for the client with diabetes mellitus who is experiencing vomiting with influenza is to obtain adequate nutrition. The diabetic client should eat small, frequent meals of 50 g of carbohydrate or food equal to 200 cal every 3 to 4 hours. If the client cannot eat the carbohydrates or take fluids, the health care provider should be called or the client should go to the emergency department. The diabetic client is in danger of complications with dehydration, electrolyte imbalance, and ketoacidosis. Increasing the client's health management skills is important to lifestyle behaviors, but it is not a priority during this acute illness of influenza. Pain relief may be a need for this client, but it is not the priority at this time; neither is increasing activity during the illness.

Which of the following findings is the best indication that fluid replacement for the client in hypovolemic shock is adequate? 1. Urine output greater than 30 mL/h. 2. Systolic blood pressure greater than 110 mm Hg. 3. Diastolic blood pressure greater than 90 mm Hg. 4. Respiratory rate of 20 breaths/min.

1 Urine output provides the most sensitive indication of the client's response to therapy for hypovolemic shock. Urine output should be consistently greater than 35 mL/h. Blood pressure is a more accurate reflection of the adequacy of vasoconstriction than of tissue perfusion. Respiratory rate is not a sensitive indicator of fluid balance in the client recovering from hypovolemic shock.

The nurse is planning care for a client who is diagnosed with peripheral vascular disease (PVD) and has a history of heart failure. The nurse should develop a plan of care that is based on the fact that the client may have a low tolerance for exercise related to: 1. Decreased blood flow. 2. Increased blood flow. 3. Decreased pain. 4. Increased blood viscosity.

1 A client with PVD and heart failure will experience decreased blood flow. In this situation, low exercise tolerance (oxygen demand becomes greater than the oxygen supply) may be related to less blood being ejected from the left ventricle into the systemic circulation. Decreased blood supply to the tissues results in pain. Increased blood viscosity may be a component, but it is of much less importance than the disease processes.

A client with heart failure has bilateral +4 edema of the right ankle that extends up to midcalf. The client is sitting in a chair with the legs in a dependent position. Which of the following goals is the priority? 1. Decrease venous congestion. 2. Maintain normal respirations. 3. Maintain body temperature. 4. Prevent injury to lower extremities.

1 Decreasing venous congestion in the extremities is a desired outcome for clients with heart failure. The nurse should elevate the client's legs above the level of the heart to achieve this goal. The client is not demonstrating difficulty breathing or being cold. The nurse should prevent injury to the swollen extremity; however, this is not the priority.

A client with type 1 diabetes mellitus has influenza. The nurse should instruct the client to: 1. Increase the frequency of self-monitoring (blood glucose testing). 2. Reduce food intake to diminish nausea. 3. Discontinue that dose of insulin if unable to eat. 4. Take half of the normal dose of insulin

1 Colds and influenza present special challenges to the client with diabetes mellitus because the body's need for insulin increases during illness. Therefore, the client must take the prescribed insulin dose, increase the frequency of blood glucose testing, and maintain an adequate fluid intake to counteract the dehydrating effect of hyperglycemia. Clear fluids, juices, and Gatorade are encouraged. Not taking insulin when sick, or taking half the normal dose, may cause the client to develop ketoacidosis.

A 68-year-old client on day 2 after hip surgery has no cardiac history but reports having chest heaviness. The first nursing action should be to: 1. Inquire about the onset, duration, severity, and precipitating factors of the heaviness. 2. Administer oxygen via nasal cannula. 3. Offer pain medication for the chest heaviness. 4. Inform the physician of the chest heaviness.

1 Further assessment is needed in this situation. It is premature to initiate other actions until further data have been gathered. Inquiring about the onset, duration, location, severity, and precipitating factors of the chest heaviness will provide pertinent information to convey to the physician.

The nurse is caring for a client who recently experienced a myocardial infarction and has been started on clopidogrel (Plavix). The nurse should develop a teaching plan that includes which of the following points? Select all that apply. 1. The client should report unexpected bleeding or bleeding that lasts a long time. 2. The client should take Plavix with food. 3. The client may bruise more easily and may experience bleeding gums. 4. Plavix works by preventing platelets from sticking together and forming a clot. 5. The client should drink a glass of water after taking Plavix.

1,3,4 Plavix is generally well absorbed and may be taken with or without food; it should be taken at the same time every day and, while food may help prevent potential GI upset, food has no effect on absorption of the drug. Bleeding is the most common adverse effect of Plavix; the client must understand the importance of reporting any unexpected, prolonged, or excessive bleeding including blood in urine or stool. Increased bruising and bleeding gums are possible side effects of Plavix; the client should be aware of this possibility. Plavix is an antiplatelet agent used to prevent clot formation in clients that have experienced or are at risk for myocardial infarction, ischemic stroke, peripheral artery disease, or acute coronary syndrome. It is not necessary to drink a glass of water after taking Plavix.

The client who does not respond adequately to fluid replacement has a prescription for an IV infusion of dopamine hydrochloride at 5 mcg/kg/min. To determine that the drug is having the desired effect, the nurse should assess the client for: 1. Increased renal and mesenteric blood flow. 2. Increased cardiac output. 3. Vasoconstriction. 4. Reduced preload and afterload.

2 At medium doses (4 to 8 mcg/kg/min), dopamine hydrochloride slightly increases the heart rate and improves contractility to increase cardiac output and improve tissue perfusion. When given at low doses (0.5 to 3.0 mcg/kg/min), dopamine increases renal and mesenteric blood flow. At high doses (8 to 10 mcg/kg/min), dopamine produces vasoconstriction, which is an undesirable effect. Dopamine is not given to affect preload and afterload.

A client is receiving dopamine hydrochloride for treatment of shock. The nurse should: 1. Administer pain medication concurrently. 2. Monitor blood pressure continuously. 3. Evaluate arterial blood gases at least every 2 hours. 4. Monitor for signs of infection.

2 The client who is receiving dopamine hydrochloride requires continuous blood pressure monitoring with an invasive or noninvasive device. The nurse may titrate the IV infusion to maintain a systolic blood pressure of 90 mm Hg. Administration of a pain medication concurrently with dopamine hydrochloride, which is a potent sympathomimetic with dose-related alpha-adrenergic agonist, beta 1-selective adrenergic agonist, and dopaminergic blocking effects, is not an essential nursing action for a client who is in shock with already low hemodynamic values. Arterial blood gas concentrations should be monitored according to the client's respiratory status and acidbase balance status and are not directly related to the dopamine hydrochloride dosage. Monitoring for signs of infection is not related to the nursing action for the client receiving dopamine hydrochloride.

The nurse explains to the client with Hodgkin's disease that a bone marrow biopsy will be taken after the aspiration. What should the nurse explain about the biopsy? 1. "Your biopsy will be performed before the aspiration because enough tissue may be obtained so that you won't have to go through the aspiration." 2. "You will feel a pressure sensation when the biopsy is taken but should not feel actual pain; if you do, tell the doctor so that you can be given extra numbing medicine." 3. "You may hear a crunch as the needle passes through the bone, but when the biopsy is taken, you will feel a suction-type pain that will last for just a moment." 4. "You will be shaved and cleaned with an antiseptic agent, after which the doctor will inject a needle without making an incision to aspirate out the bone marrow."

2 A biopsy needle is inserted through a separate incision in the anesthetized area. The client will feel a pressure sensation when the biopsy is taken but should not feel actual pain. The client should be instructed to inform the physician if pain is felt so that more anesthetic agent can be administered to keep the client comfortable. The biopsy is performed after the aspiration and from a slightly different site so that the tissue is not disturbed by either test. The client will feel a suction-type pain for a moment when the aspiration is being performed, not the biopsy. A small incision is made for the biopsy to accommodate the larger-bore needle. This may require a stitch

Prior to administering tissue plasminogen activator (t-PA), the nurse should assess the client for which of the following contradictions to administering the drug? 1. Age greater than 60 years. 2. History of cerebral hemorrhage. 3. History of heart failure. 4. Cigarette smoking.

2 A history of cerebral hemorrhage is a contraindication to administration of t- PA because the risk of hemorrhage may be further increased. Age greater than 60 years, history of heart failure, and cigarette smoking are not contraindications.

The nurse instructs the unlicensed nursing personnel (UAP) on how to provide oral hygiene for clients who cannot perform this task for themselves. Which of the following techniques should the nurse tell the UAP to incorporate into the client's daily care? 1. Assess the oral cavity each time mouth care is given and record observations. 2. Use a soft toothbrush to brush the client's teeth after each meal. 3. Swab the client's tongue, gums, and lips with a soft foam applicator every 2 hours. 4. Rinse the client's mouth with mouthwash several times a day.

2 A soft toothbrush should be used to brush the client's teeth after every meal and more often as needed. Mechanical cleaning is necessary to maintain oral health, stimulate gingiva, and remove plaque. Assessing the oral cavity and recording observations is the responsibility of the nurse, not the nursing assistant. Swabbing with a safe foam applicator does not provide enough friction to clean the mouth. Mouthwash can be a drying irritant and is not recommended for frequent use.

A client with thyrotoxicosis says to the nurse, "I am so irritable. I am having problems at work because I lose my temper very easily." Which of the following responses by the nurse would give the client the most accurate explanation of her behavior? 1. "Your behavior is caused by temporary confusion brought on by your illness." 2. "Your behavior is caused by the excess thyroid hormone in your system." 3. "Your behavior is caused by your worrying about the seriousness of your illness."

2 A typical sign of thyrotoxicosis is irritability caused by the high levels of circulating thyroid hormones in the body. This symptom decreases as the client responds to therapy. Thyrotoxicosis does not cause confusion. The client may be worried about her illness, and stress may influence her mood; however, irritability is a common symptom of thyrotoxicosis and the client should be informed of that fact rather than blamed.

A client has driven himself to the emergency department. He is 50 years old, has a history of hypertension, and informs the nurse that his father died from a heart attack at age 60. The client has indigestion. The nurse connects him to an electrocardiogram monitor and begins administering oxygen at 2 L/min per nasal cannula. The nurse's next action should be to: 1. Call for the physician. 2. Start an IV infusion. 3. Obtain a portable chest radiograph. 4. Draw blood for laboratory studies

2 Advanced cardiac life support recommends that at least one or two IV lines be inserted in one or both of the antecubital spaces. Calling the physician, obtaining a portable chest radiograph, and drawing blood for the laboratory are important but secondary to starting the IV line.

The nurse is evaluating the client's learning about combination chemotherapy. Which of the following statements by the client about reasons for using combination chemotherapy indicates the need for further explanation? 1. "Combination chemotherapy is used to interrupt cell growth cycle at different points." 2. "Combination chemotherapy is used to destroy cancer cells and treat side effects simultaneously." 3. "Combination chemotherapy is used to decrease resistance." 4. "Combination chemotherapy is used to minimize the toxicity from using high doses of a single agent."

2 Combination chemotherapy does not mean two groups of drugs, one to kill the cancer cells and one to treat the adverse effects of the chemotherapy. Combination chemotherapy means that multiple drugs are given to interrupt the cell growth cycle at different points, decrease resistance to a chemotherapy agent, and minimize the toxicity associated with use of a high dose of a single agent (ie, by using multiple agents with different toxicities).

Which of the following is an expected outcome when a client is receiving an IV administration of furosemide? 1. Increased blood pressure. 2. Increased urine output. 3. Decreased pain. 4. Decreased premature ventricular contractions.

2 Furosemide is a loop diuretic that acts to increase urine output. Furosemide does not increase blood pressure, decrease pain, or decrease arrhythmias.

The nurse is completing a health assessment of a 42-year-old female with suspected Graves' disease. The nurse should assess this client for: 1. Anorexia. 2. Tachycardia. 3. Weight gain. 4. Cold skin.

2 Graves' disease, the most common type of thyrotoxicosis, is a state of hypermetabolism. The increased metabolic rate generates heat and produces tachycardia and fine muscle tremors. Anorexia is associated with hypothyroidism. Loss of weight, despite a good appetite and adequate caloric intake, is a common feature of hyperthyroidism. Cold skin is associated with hypothyroidism.

The client is a survivor of non-Hodgkin's lymphoma. Which of the following statements indicates the client needs additional information? 1. "Regular screening is very important for me." 2. "The survivor rate is directly proportional to the incidence of second malignancy." 3. "The survivor rate is indirectly proportional to the incidence of second malignancy." 4. "It is important for survivors to know the stage of the disease and their current treatment plan."

2 It is incorrect that the survivor rate is directly proportional to the incidence of second malignancy. The survivor rate is indirectly proportional to the incidence of second malignancy, and regular screening is very important to detect a second malignancy, especially acute myeloid leukemia or myelodysplastic syndrome. Survivors should know the stage of the disease and their current treatment plan so that they can remain active participants in their health care.

Which of the following clinical manifestations does the nurse most likely observe in a client with Hodgkin's disease? 1. Difficulty swallowing. 2. Painless, enlarged cervical lymph nodes. 3. Difficulty breathing. 4. A feeling of fullness over the liver.

2 Painless and enlarged cervical lymph nodes, tachycardia, weight loss, weakness and fatigue, and night sweats are signs of Hodgkin's disease. Difficulty swallowing and breathing may occur, but only with mediastinal node involvement. Hepatomegaly is a late-stage manifestation.

Which of the following is the most appropriate diet for a client during the acute phase of myocardial infarction? 1. Liquids as desired. 2. Small, easily digested meals. 3. Three regular meals per day. 4. Nothing by mouth

2 Recommended dietary principles in the acute phase of MI include avoiding large meals because small, easily digested foods are better tolerated. Fluids are given according to the client's needs, and sodium restrictions may be prescribed, especially for clients with manifestations of heart failure. Cholesterol restrictions may be prescribed as well. Clients are not prescribed diets of liquids only or restricted to nothing by mouth unless their condition is very unstable.

A client has peripheral vascular disease (PVD) of the lower extremities. The client tells the nurse, "I've really tried to manage my condition well." Which of the following routines should the nurse evaluate as having been appropriate for this client? 1. Resting with the legs elevated above the level of the heart. 2. Walking slowly but steadily for 30 minutes twice a day. 3. Minimizing activity. 4. Wearing antiembolism stockings at all times when out of bed

2 Slow, steady walking is a recommended activity for clients with peripheral vascular disease because it stimulates the development of collateral circulation. The client with PVD should not remain inactive. Elevating the legs above the heart or wearing antiembolism stockings is a strategy for alleviating venous congestion and may worsen peripheral arterial disease

The nurse is assessing the client's use of medications. Which of the following medications may cause a complication with the treatment plan of a client with diabetes? 1. Aspirin. 2. Steroids. 3. Sulfonylureas. 4. Angiotensin-converting enzyme (ACE) inhibitors

2 Steroids can cause hyperglycemia because of their effects on carbohydrate metabolism, making diabetic control more difficult. Aspirin is not known to affect glucose metabolism. Sulfonylureas are oral hypoglycemic agents used in the treatment of diabetes mellitus. ACE inhibitors are not known to affect glucose metabolism.

Which is a priority assessment for the client in shock who is receiving an IV infusion of packed red blood cells and normal saline solution? 1. Fluid balance. 2. Anaphylactic reaction. 3. Pain. 4. Altered level of consciousness.

2 The client who is receiving a blood product requires astute assessment for signs and symptoms of allergic reaction and anaphylaxis, including pruritus (itching), urticaria (hives), facial or glottal edema, and shortness of breath. If such a reaction occurs, the nurse should stop the transfusion immediately, but leave the IV line intact, and notify the physician. Usually, an antihistamine such as diphenhydramine hydrochloride (Benadryl) is administered. Epinephrine and corticosteroids may be administered in severe reactions. Fluid balance is not an immediate concern during the blood administration. The administration should not cause pain unless it is extravasating out of the vein, in which case the IV administration should be stopped. Administration of a unit of blood should not affect the level of consciousness.

When assessing a client for early septic shock, the nurse should assess the client for which of the following? 1. Cool, clammy skin. 2. Warm, flushed skin. 3. Increased blood pressure. 4. Hemorrhage.

2 Warm, flushed skin from a high cardiac output with vasodilation occurs in warm shock or the hyperdynamic phase (first phase) of septic shock. Other signs and symptoms of early septic shock include fever with restlessness and confusion; normal or decreased blood pressure with tachypnea and tachycardia; increased or normal urine output; and nausea and vomiting or diarrhea. Cool, clammy skin occurs in the hypodynamic or cold phase (later phase). Hemorrhage is not a factor in septic shock.

Which of the following indicates a potential complication of diabetes mellitus? 1. Inflamed, painful joints. 2. Blood pressure of 160/100 mm Hg. 3. Stooped appearance. 4. Hemoglobin of 9 g/dL (90 g/L).

2 The client with diabetes mellitus is especially prone to hypertension due to atherosclerotic changes, which leads to problems of the microvascular and macrovascular systems. This can result in complications in the heart, brain, and kidneys. Heart disease and stroke are twice as common among people with diabetes mellitus as among people without the disease. Painful, inflamed joints accompany rheumatoid arthritis. A stooped appearance accompanies osteoporosis with narrowing of the vertebral column. A low hemoglobin concentration accompanies anemia, especially iron deficiency anemia and anemia of chronic disease.

A client with peripheral vascular disease has undergone a right femoral-popliteal bypass graft. The blood pressure has decreased from 124/80 to 94/62. What should the nurse assess first? 1. IV fluid solution. 2. Pedal pulses. 3. Nasal cannula flow rate. 4. Capillary refill

2 With each set of vital signs, the nurse should assess the dorsalis pedis and posterior tibial pulses. The nurse needs to ensure adequate perfusion to the lower extremity with the drop in blood pressure. IV fluids, nasal cannula setting, and capillary refill are important to assess; however, priority is to determine the cause of drop in blood pressure and that adequate perfusion through the new graft is maintained.

The nurse identifies deficient knowledge when the client undergoing induction therapy for leukemia makes which of the following statements? 1. "I will pace my activities with rest periods." 2. "I can't wait to get home to my cat!" 3. "I will use warm saline gargle instead of brushing my teeth." 4. "I must report a temperature of 100°F (37.7°C)."

2 The nurse identifies that the client does not understand that contact with animals must be avoided because they carry infection and the induction therapy will destroy the client's white blood cells (WBCs). The induction therapy will cause anemia, and the client will experience fatigue and will have to pace activities with rest periods. Platelet production will be decreased, and the client will be at risk for bleeding tendencies; oral hygiene will have to be provided by using a warm saline gargle instead of brushing the teeth and gums. The client will be at risk for infection owing to the decrease in WBC production and should report a temperature of 100°F (37.8°C) or higher.

The nurse is teaching a client about risk factors associated with atherosclerosis and how to reduce the risk. Which of the following is a risk factor that the client is not able to modify? 1. Diabetes. 2. Age. 3. Exercise level. 4. Dietary preferences

2 Age is a nonmodifiable risk factor for atherosclerosis. The nurse instructs the client to manage modifiable risk factors such as comorbid diseases (eg, diabetes), activity level, and diet. Controlling serum blood glucose levels, engaging in regular aerobic activity, and choosing a diet low in saturated fats can reduce the risk of developing atherosclerosis.

Ciprofloxacin hydrochloride is prescribed to a client with a urinary tract infection. The nurse provides instruction about the medication. What does the nurse tell the client about how best to take the medication? With milk With an antacid 2 hours after meals With aluminum hydroxide

2 hours after meals Rationale: Ciprofloxacin hydrochloride is an anti-infective in the fluoroquinolone family. It may be taken without regard to meals, but the best dosing time is 2 hours after a meal. Milk may affect absorption. Antacids (here, aluminum hydroxide) may reduce absorption and should be administered 2 hours apart from the ciprofloxacin hydrochloride.

A 34-year-old female is diagnosed with hypothyroidism. The nurse should assess the client for which of the following? Select all that apply. 1. Rapid pulse. 2. Decreased energy and fatigue. 3. Weight gain of 10 lb (4.5 kg). 4. Fine, thin hair with hair loss. 5. Constipation. 6. Menorrhagia.

2, 3, 5, 6. Clients with hypothyroidism exhibit symptoms indicating a lack of thyroid hormone. Bradycardia, decreased energy and lethargy, memory problems, weight gain, coarse hair, constipation, and menorrhagia are common signs and symptoms of hypothyroidism.

A client reports vomiting every hour for the past 8 to 10 hours. The nurse should assess the client for risk of which of the following? Select all that apply. 1. Metabolic acidosis. 2. Metabolic alkalosis. 3. Hypokalemia. 4. Hyperkalemia. 5. Hyponatremia.

2,3 Gastric acid contains a substantial amount of potassium, hydrogen ions, and chloride ions. Frequent vomiting can induce an excessive loss of these acids leading to alkalosis. Excessive loss of potassium produces hypokalemia. Frequent vomiting does not lead to the condition of too much potassium (hyperkalemia) or too little sodium

The nurse is assessing the lower extremities of the client with peripheral vascular disease (PVD). During the assessment, the nurse should expect to find which of the following clinical manifestations of PVD? Select all that apply. 1. Hairy legs. 2. Mottled skin. 3. Pink skin. 4. Coolness. 5. Moist skin.

2,4 Reduction of blood flow to a specific area results in decreased oxygen and nutrients. As a result, the skin may appear mottled. The skin will also be cool to the touch. Loss of hair and dry skin are other signs that the nurse may observe in a client with PVD of the lower extremities.

A primary health care provider writes a prescription for 1000 mL of 0.9% normal saline solution to be administered intravenously (IV) to a client over 10 hours. The drop factor for the infusion set is 15 gtt/mL. At what drip rate does the nurse set the infusion? Type answer in the box provided. _______________ gtt/min

25

A client with advanced Hodgkin's disease is admitted to hospice because death is imminent. The goal of nursing care at this time is to: 1. Reduce the client's fear of pain. 2. Support the client's wish to discontinue further therapy. 3. Prevent feelings of isolation. 4. Help the client overcome feelings of social inadequacy.

3 Terminally ill clients most often describe feelings of isolation because they tend to be ignored, they are often left out of conversations (especially those dealing with the future), and they sense the attitudes of discomfort that many people feel in their presence. Helpful nursing measures include taking the time to be with the client, offering opportunities to talk about feelings, and answering questions honestly.

If a client displays risk factors for coronary artery disease, such as smoking cigarettes, eating a diet high in saturated fat, or leading a sedentary lifestyle, techniques of behavior modification may be used to help the client change the behavior. The nurse can best reinforce new adaptive behaviors by: 1. Explaining how the risk factor behavior leads to poor health. 2. Withholding praise until the new behavior is well established. 3. Rewarding the client whenever the acceptable behavior is performed. 4. Instilling mild fear into the client to extinguish the behavior.

3 A basic principle of behavior modification is that behavior that is learned and continued is behavior that has been rewarded. Other reinforcement techniques have not been found to be as effective as reward.

The client with Hodgkin's disease develops B symptoms. These manifestations indicate which of the following? 1. The client has a low-grade fever (temperature lower than 100°F [37.8°C]). 2. The client has a weight loss of 5% or less of body weight. 3. The client has night sweats. 4. The client probably has not progressed to an advanced stage.

3 A temperature higher than 100.4°F (38°C), profuse night sweats, and an unintentional weight loss of 10% of body weight represent the cluster of clinical manifestations known as the B symptoms. Forty percent of clients with Hodgkin's disease have B symptoms, and B symptoms are more common in advanced stages of the disease.

Which of the following lipid abnormalities is a risk factor for the development of atherosclerosis and peripheral vascular disease? 1. Low concentration of triglycerides. 2. High levels of high-density lipid (HDL) cholesterol. 3. High levels of low-density lipid (LDL) cholesterol. 4. Low levels of LDL cholesterol.

3 An increased LDL cholesterol concentration has been documented as a risk factor for the development of atherosclerosis. LDL cholesterol is not broken down in the liver but is deposited into the intima of the blood vessels. Low triglyceride levels are desirable. High HDL and low LDL levels are beneficial and are known to be protective for the cardiovascular system.

The client with Hodgkin's disease undergoes an excisional cervical lymph node biopsy under local anesthesia. After the procedure, which does the nurse assess first? 1. Vital signs. 2. The incision. 3. The airway. 4. Neurologic signs.

3 Assessing for an open airway is always first. The procedure involves the neck; the anesthesia may have affected the swallowing reflex, or the inflammation may have closed in on the airway, leading to ineffective air exchange. Once a patent airway is confirmed and an effective breathing pattern established, the circulation is checked. Vital signs and the incision are assessed as soon as possible, but only after it is established that the airway is patent and the client is breathing normally. A neurologic assessment is completed as soon as possible after other important assessments.

. A middle-aged adult with a family history of CAD has the following: total cholesterol 198 (11 mmol/L); LDL cholesterol 120 (6.7 mmol/L); HDL cholesterol 58 (3.2 mmol/L); triglycerides 148 (8.2 mmol/L); blood sugar 102 (5.7 mmol/L); and Creactive protein (CRP) 4.2. The health care provider prescribes a statin medication and aspirin. The client asks the nurse why these medications are needed. Which is the best response by the nurse? 1. "The labs indicate severe hyperlipidemia and the medications will lower your LDL, along with a low-fat diet." 2. "The triglycerides are elevated and will not return to normal without these medications." 3. "The CRP is elevated indicating inflammation seen in cardiovascular disease, which can be lowered by the medications prescribed." 4. "These medications will reduce the risk of type 2 diabetes."

3 CRP is a marker of inflammation and is elevated in the presence of cardiovascular disease. The high sensitivity CRP (hs-CRP) is the blood test for greater accuracy in measuring the CRP to evaluate cardiovascular risk. The family history, postmenopausal age, LDL above optimum levels, and elevated CRP place the client at risk of CAD. Statin medications can decrease LDL, whereas statins and aspirin can reduce CRP and decrease the risk of MI and stroke. The blood sugar is within normal limits.

A client is receiving Cilostazol (Pletal) for peripheral arterial disease causing intermittent claudication. The nurse determines this medication is effective when the client reports which of the following? 1. "I am having fewer aches and pains." 2. "I do not have headaches anymore." 3. "I am able to walk further without leg pain." 4. "My toes are turning grayish black in color."

3 Cilostazol is indicated for management of intermittent claudication. Symptoms usually improve within 2 to 4 weeks of therapy. Intermittent claudication prevents clients from walking for long periods of time. Cilostazol inhibits platelet aggregation induced by various stimuli and improving blood flow to the muscles and allowing the client to walk long distances without pain. Peripheral arterial disease causes pain mainly of the leg muscles. "Aches and pains" does not specify exactly where the pain is occurring. Headaches may occur as a side effect of this drug, and the client should report this information to the health care provider. Peripheral arterial disease causes decreased blood supply to the peripheral tissues and may cause gangrene of the toes; the drug is effective when the toes are warm to the touch and the color of the toes is similar to the color of the body.

To help minimize the risk of postoperative respiratory complications after a hypophysectomy, during preoperative teaching, the nurse should instruct the client how to: 1. Use incentive spirometry. 2. Turn in bed. 3. Take deep breaths. 4. Cough.

3 Deep breathing is the best choice for helping prevent atelectasis. The client should be placed in the semi-Fowler's position (or as prescribed) and taught deep breathing, sighing, mouth breathing, and how to avoid coughing. Blow bottles are not effective in preventing atelectasis because they do not promote sustained alveolar inflation to maximal lung capacity. Frequent position changes help loosen lung secretions, but deep breathing is most important in preventing atelectasis. Coughing is contraindicated because it increases intracranial pressure and can cause cerebrospinal fluid to leak from the point at which the sella turcica was entered.

After a myocardial infarction, the hospitalized client is taught to move the legs while resting in bed. The expected outcome of this exercise is to: 1. Prepare the client for ambulation. 2. Promote urinary and intestinal elimination. 3. Prevent thrombophlebitis and blood clot formation. 4. Decrease the likelihood of pressure ulcer formation.

3 Encouraging the client to move the legs while in bed is a preventive strategy taught to all clients who are hospitalized and on bed rest to promote venous return. The muscular action aids in venous return and prevents venous stasis in the lower extremities. These exercises are not intended to prepare the client for ambulation. These exercises are not associated with promoting urinary and intestinal elimination. These exercises are not performed to decrease the risk of pressure ulcer formation

A client is undergoing a bone marrow aspiration and biopsy. What is the best way for the nurse to help the client and two upset family members handle anxiety during the procedure? 1. Allow the client's family to stay as long as possible. 2. Stay with the client without speaking. 3. Encourage the client to take slow, deep breaths to relax. 4. Allow the client time to express feelings.

3 Encouraging the client to take slow, deep breaths during uncomfortable parts of procedures is the best method of decreasing the stress response of tightening and tensing the muscles. Slow, deep breathing affects the level of carbon dioxide in the brain to increase the client's sense of well-being. Allowing the client's family to stay may be appropriate if the family has a calming effect on the client, but this family is upset and may contribute to the client's stress. Silence can be therapeutic, but when the client is faced with a potentially life-threatening diagnosis and a new, invasive procedure, taking deep breaths will be more effective in reducing the stress response. Expressing feelings is important, but deep breathing will promote relaxation; the nurse can encourage the client to express feelings when the procedure is completed.

A nurse is teaching a client with type 1 diabetes mellitus who jogs daily about the preferred sites for insulin absorption. What is the most appropriate site for a client who jogs? 1. Arms. 2. Legs. 3. Abdomen. 4. Iliac crest.

3 If the client engages in an activity or exercise that focuses on one area of the body, that area may cause inconsistent absorption of insulin. A good regimen for a jogger is to inject the abdomen for 1 week and then rotate to the buttock. A jogger may have inconsistent absorption in the legs or arms with strenuous running. The iliac crest is not an appropriate site due to a lack of loose skin and subcutaneous tissue in that area.

. Following diagnosis of angina pectoris, a client reports being unable to walk up two flights of stairs without pain. Which of the following measures would most likely help the client prevent this problem? 1. Climb the steps early in the day. 2. Rest for at least an hour before climbing the stairs. 3. Take a nitroglycerin tablet before climbing the stairs. 4. Lie down after climbing the stairs.

3 Nitroglycerin may be used prophylactically before stressful physical activities such as stair climbing to help the client remain pain free. Climbing the stairs early in the day would have no impact on decreasing pain episodes. Resting before or after an activity is not as likely to help prevent an activity-related pain episode.

Which of the following is the most important goal of nursing care for a client who is in shock? 1. Manage fluid overload. 2. Manage increased cardiac output. 3. Manage inadequate tissue perfusion. 4. Manage vasoconstriction of vascular beds

3 Nursing interventions and collaborative management are focused on correcting and maintaining adequate tissue perfusion. Inadequate tissue perfusion may be caused by hemorrhage, as in hypovolemic shock; by decreased cardiac output, as in cardiogenic shock; or by massive vasodilation of the vascular bed, as in neurogenic, anaphylactic, and septic shock. Fluid deficit, not fluid overload, occurs in shock

The client who experiences angina has been told to follow a low-cholesterol diet. Which of the following meals would be best? 1. Hamburger, salad, and milkshake. 2. Baked liver, green beans, and coffee. 3. Spaghetti with tomato sauce, salad, and coffee. 4. Fried chicken, green beans, and skim milk

3 Pasta, tomato sauce, salad, and coffee would be the best selection for the client following a low-cholesterol diet. Hamburgers, milkshakes, liver, and fried foods tend to be high in cholesterol.

Angiotensin-converting enzyme (ACE) inhibitors may be prescribed for the client with diabetes mellitus to reduce vascular changes and possibly prevent or delay development of: 1. Chronic obstructive pulmonary disease (COPD). 2. Pancreatic cancer. 3. Renal failure. 4. Cerebrovascular accident.

3 Renal failure frequently results from the vascular changes associated with diabetes mellitus. ACE inhibitors increase renal blood flow and are effective in decreasing diabetic nephropathy. Chronic obstructive pulmonary disease is not a complication of diabetes, nor is it prevented by ACE inhibitors. Pancreatic cancer is neither prevented by ACE inhibitors nor considered a complication of diabetes. Cerebrovascular accident is not directly prevented by ACE inhibitors, although management of hypertension will decrease vascular disease.

The nurse is planning care with a client with acute leukemia who has mucositis. The nurse should advise the client that after every meal and every 4 hours while awake the client should use: 1. Lemon-glycerin swabs. 2. A commercial mouthwash. 3. A saline solution. 4. A commercial toothpaste and brush

3 Simple rinses with saline or a baking soda and water solution are effective and moisten the oral mucosa. Commercial mouthwashes and lemon-glycerin swabs contain glycerin and alcohol, which are drying to the mucosa and should be avoided. Brushing after each meal is recommended, but every 4 hours may be too traumatic. During acute leukemia, the neutrophil and platelet counts are often low and a soft-bristle toothbrush, instead of the client's usual brush, should be used to prevent bleeding gums.

The client with acute leukemia and the health care team establish mutual client outcomes of improved tidal volume and activity tolerance. Which measure would be least likely to promote outcome achievement? 1. Ambulating in the hallway. 2. Sitting up in a chair. 3. Lying in bed and taking deep breaths. 4. Using a stationary bicycle in the room.

3 The client with acute leukemia experiences fatigue and deconditioning. Lying in bed and taking deep breaths will not help achieve the goals. The client must get out of bed to increase activity tolerance and improve tidal volume. Ambulating in the hall (using a HEPA filter mask if neutropenic) is a sensible activity and helps improve conditioning. Sitting up in a chair facilitates lung expansion. Using a stationary bicycle in the room allows the client to increase activity as tolerated.

Which of the following conditions is the most significant risk factor for the development of type 2 diabetes mellitus? 1. Cigarette smoking. 2. High-cholesterol diet. 3. Obesity. 4. Hypertension.

3 The most important factor predisposing to the development of type 2 diabetes mellitus is obesity. Insulin resistance increases with obesity. Cigarette smoking is not a predisposing factor, but it is a risk factor that increases complications of diabetes mellitus. A high-cholesterol diet does not necessarily predispose to diabetes mellitus, but it may contribute to obesity and hyperlipidemia. Hypertension is not a predisposing factor, but it is a risk factor for developing complications of diabetes mellitus.

The nurse in the intensive care unit is giving a report to the nurse in the post surgical unit about a client who had a gastrectomy. The most effective way to assure essential information about the client is reported is to: 1. Give the report face to face with both nurses in a quiet room. 2. Audiotape the report for future reference and documentation. 3. Use a printed checklist with information individualized for the client. 4. Document essential transfer information in the client's electronic health record

3 Using an individualized printed checklist assures that all key information is reported; the checklist can then serve as a record to which nurses can refer later. Giving a verbal report leaves room for error in memory; using an audiotape or an electronic health record requires nurses to spend unnecessary time retrieving information.

The nurse is caring for a client with peripheral artery disease who has recently been prescribed clopidogrel (Plavix). The nurse understands that more teaching is necessary when the client states which of the following: 1. "I should not be surprised if I bruise easier or if my gums bleed a little when brushing my teeth." 2. "It doesn't really matter if I take this medicine with or without food, whatever works best for my stomach." 3. "I should stop taking Plavix if it makes me feel weak and dizzy." 4. "The doctor prescribed this medicine to make my platelets less likely to stick together and help prevent clots from forming."

3 Weakness, dizziness, and headache are common adverse effects of Plavix and the client should report these to the physician if they are problematic; in order to decrease risk of clot formation, Plavix must be taken regularly and should not be stopped or taken intermittently. The main adverse effect of Plavix is bleeding, which often occurs as increased bruising or bleeding when brushing teeth. Plavix is well absorbed, and while food may help decrease potential gastrointestinal upset, Plavix may be taken with or without food. Plavix is an antiplatelet agent used to prevent clot formation in clients who have experienced or are at risk for myocardial infarction, ischemic stroke, peripheral artery disease, or acute coronary syndrome.

The nurse is unable to palpate the client's left pedal pulses. Which of the following actions should the nurse take next? 1. Auscultate the pulses with a stethoscope. 2. Call the physician. 3. Use a Doppler ultrasound device. 4. Inspect the lower left extremity

3 When pedal pulses are not palpable, the nurse should obtain a Doppler ultrasound device. Auscultation is not likely to be helpful if the pulse isn't palpable. Inspection of the lower extremity can be done simultaneously when palpating, but the nurse should first try to locate a pulse by Doppler. Calling the physician may be necessary if there is a change in the client's condition.

A client has had a pulmonary artery catheter inserted. In performing hemodynamic monitoring with the catheter, the nurse will wedge the catheter to gain information about which of the following? 1. Cardiac output. 2. Right atrial blood flow. 3. Left end-diastolic pressure. 4. Cardiac index

3 When wedged, the catheter is "pointing" indirectly at the left end-diastolic pressure. The pulmonary artery wedge pressure is measured when the tip of the catheter is slowing inflated and allowed to wedge into a branch of the pulmonary artery. Once the balloon is wedged, the catheter reads the pressure in front of the balloon. During diastole, the mitral valve is open, reflecting left ventricular end diastolic pressure. Cardiac output is the amount of blood ejected by the heart in 1 minute and is determined through thermodilution and not wedge pressure. Cardiac index is calculated by dividing the client's cardiac output by the client's body surface area, and is considered a more accurate reflection of the individual client's cardiac output. Right atrial blood pressure is not measured with the pulmonary artery catheter.

The nurse is assessing an older Caucasian male who has a history of peripheral vascular disease. The nurse observes that the man's left great toe is black. The discoloration is probably a result of: 1. Atrophy. 2. Contraction. 3. Gangrene. 4. Rubor.

3 The term gangrene refers to blackened, decomposing tissue that is devoid of circulation. Chronic ischemia and death of the tissue can lead to gangrene in the affected extremity. Injury, edema, and decreased circulation lead to infection, gangrene, and tissue death. Atrophy is the shrinking of tissue, and contraction is joint stiffening secondary to disuse. The term rubor denotes a reddish color of the skin

Lorazepam 1 mg by way of intravenous (IV) injection (IV push) is prescribed for a client for the management of anxiety. The nurse prepares the medication as prescribed. Over what period of time should the nurse administer this medication? 3 minutes 10 seconds 15 seconds 30 minutes

3 minutes Rationale: Lorazepam is a benzodiazepine. When administered by IV injection, each 2 mg or fraction thereof is administered over a period of 1 to 5 minutes. Ten seconds and 30 seconds are brief periods. Thirty minutes is a lengthy period.

When conducting a health history with a female client with thyrotoxicosis, the nurse should ask about which of the following changes in the menstrual cycle? 1. Dysmenorrhea. 2. Metrorrhagia. 3. Oligomenorrhea. 4. Menorrhagia.

3. A change in the menstrual interval, diminished menstrual flow (oligomenorrhea), or even the absence of menstruation (amenorrhea) may result from the hormonal imbalances of thyrotoxicosis. Oligomenorrhea in women and decreased libido and impotence in men are common features of thyrotoxicosis. Dysmenorrhea is painful menstruation. Metrorrhagia, blood loss between menstrual periods, is a symptom of hypothyroidism. Menorrhagia, excessive bleeding during menstrual periods, is a symptom of hypothyroidism.

A nurse is teaching a client with left-side weakness how to walk with the use of a quad-cane. What does the nurse ensure? The client places the cane on the left side The top of the cane is level with the client's waist 30-degree flexion of the client's elbow is maintained when the client is holding the cane The client leans on the cane and places as much weight as possible on the cane when moving it forward

30-degree flexion of the client's elbow is maintained when the client is holding the cane Rationale: The cane, which is placed on the unaffected side, should produce no more than 30 degrees of flexion of the elbow. A cane may be used if a client needs minimal support for an affected leg. A straight cane offers the least support. A hemi-cane or quad-cane provides a broader base for the cane and therefore more support. The top of the cane should be level with the greater trochanter of the femur

The nurse is instructing the client on insulin administration. The client is performing a return demonstration for preparing the insulin. The client's morning dose of insulin is 10 units of regular and 22 units of NPH. The nurse checks the dose accuracy with the client. The nurse determines that the client has prepared the correct dose when the syringe reads how many units?

32 units

A nurse has two middle-aged clients who have a prescription to receive a blood transfusion of packed red blood cells at the same time. The first client's blood pressure dropped from the preoperative value of 120/80 mm Hg to a postoperative value of 100/50. The second client is hospitalized because he developed dehydration and anemia following pneumonia. After checking the patency of their IV lines and vital signs, what should the nurse do next? 1. Call for both clients' blood transfusions at the same time. 2. Ask another nurse to verify the compatibility of both units at the same time. 3. Call for and hang the first client's blood transfusion. 4. Ask another nurse to call for and hang the blood for the second client.

4

Which of the following indicates hypovolemic shock in a client who has had a 15% blood loss? 1. Pulse rate less than 60 bpm. 2. Respiratory rate of 4 breaths/min. 3. Pupils unequally dilated. 4. Systolic blood pressure less than 90 mm Hg.

4 Typical signs and symptoms of hypovolemic shock include systolic blood pressure less than 90 mm Hg, narrowing pulse pressure, tachycardia, tachypnea, cool and clammy skin, decreased urine output, and mental status changes, such as irritability or anxiety. Unequal dilation of the pupils is related to central nervous system injury or possibly to a previous history of eye injury.

The nurse should caution the client with diabetes mellitus who is taking a sulfonylurea that alcoholic beverages should be avoided while taking these drugs because they can cause which of the following? 1. Hypokalemia. 2. Hyperkalemia. 3. Hypocalcemia. 4.Disulfiram (Antabuse)-like symptoms

4 A client with diabetes who takes any first- or second-generation sulfonylurea should be advised to avoid alcohol intake. Sulfonylureas in combination with alcohol can cause serious disulfiram (Antabuse)-like reactions, including flushing, angina, palpitations, and vertigo. Serious reactions, such as seizures and possibly death, may also occur. Hypokalemia, hyperkalemia, and hypocalcemia do not result from taking sulfonylureas in combination with alcohol.

The nurse is assessing a client with chronic myeloid leukemia (CML). The nurse should assess the client for: 1. Lymphadenopathy. 2. Hyperplasia of the gum. 3. Bone pain from expansion of marrow. 4. Shortness of breath

4 Although the clinical manifestations of CML vary, clients usually have confusion and shortness of breath related to decreased capillary perfusion to the brain and lungs. Lymphadenopathy is rare in CML. Hyperplasia of the gum and bone pain are clinical manifestations of AML.

The goal of nursing care for a client with acute myeloid leukemia (AML) is to prevent: 1. Cardiac arrhythmias. 2. Liver failure. 3. Renal failure. 4. Hemorrhage.

4 Bleeding and infection are the major complications and causes of death for clients with AML. Bleeding is related to the degree of thrombocytopenia, and infection is related to the degree of neutropenia. Cardiac arrhythmias rarely occur as a result of AML. Liver or renal failure may occur, but neither is a major cause of death in AML.

In assessing a client in the early stage of chronic lymphocytic leukemia (CLL), the nurse should determine if the client has: 1. Enlarged, painless lymph nodes. 2. Headache. 3. Hyperplasia of the gums. 4. Unintentional weight loss.

4 Clients with CLL develop unintentional weight loss; fever and drenching night sweats; enlarged, painful lymph nodes, spleen, and liver; decreased reaction to skin sensitivity tests (anergy); and susceptibility to viral infections. Enlarged, painless lymph nodes are a clinical manifestation of Hodgkin's lymphoma. A headache would not be one of the early signs and symptoms expected in CLL because CLL does not cross the blood-brain barrier and would not irritate the meninges. Hyperplasia of the gums is a clinical manifestation of AML.

Crackles heard on lung auscultation indicate which of the following? 1. Cyanosis. 2. Bronchospasm. 3. Airway narrowing. 4.Fluid-filled alveoli.

4 Crackles are auscultated over fluid-filled alveoli. Crackles heard on lung auscultation do not have to be associated with cyanosis. Bronchospasm and airway narrowing generally are associated with wheezing sounds.

Assessment of the diabetic client for common complications should include examination of the: 1. Abdomen. 2. Lymph glands. 3. Pharynx. 4. Eyes.

4 Diabetic retinopathy, cataracts, and glaucoma are common complications in diabetics, necessitating eye assessment and examination. The feet should also be examined at each client encounter, monitoring for thickening, fissures, or breaks in the skin; ulcers; and thickened nails. Although assessments of the abdomen, pharynx, and lymph glands are included in a thorough examination, they are not pertinent to common diabetic complications.

When assessing the lower extremities of a client with peripheral vascular disease (PVD), the nurse notes bilateral ankle edema. The edema is related to: 1. Competent venous valves. 2. Decreased blood volume. 3. Increase in muscular activity. 4. Increased venous pressure.

4 In PVD, decreased blood flow can result in increased venous pressure. The increase in venous pressure results in an increase in capillary hydrostatic pressure, which causes a net filtration of fluid out of the capillaries into the interstitial space, resulting in edema. Valves often become incompetent with PVD. Blood volume is not decreased in this condition. Decreased muscular action would contribute to the formation of edema in the lower extremities.

A client with diabetes begins to cry and says, "I just cannot stand the thought of having to give myself a shot every day." Which of the following would be the best response by the nurse? 1. "If you do not give yourself your insulin shots, you will die." 2. "We can teach your daughter to give the shots so you will not have to do it." 3. "I can arrange to have a home care nurse give you the shots every day." 4. "What is it about giving yourself the insulin shots that bothers you?"

4 The best response is to allow the client to verbalize her fears about giving herself a shot each day. Tactics that increase fear are not effective in changing behavior. If possible, the client needs to be responsible for her own care, including giving selfinjections. It is unlikely that the client's insurance company will pay for home-care visits if the client is capable of self-administration.

The client with type 1 diabetes mellitus is taught to take isophane insulin suspension NPH (Humulin N) at 5 PM each day. The client should be instructed that the greatest risk of hypoglycemia will occur at about what time? 1. 11 AM, shortly before lunch. 2. 1 PM, shortly after lunch. 3. 6 PM, shortly after dinner. 4.1 AM, while sleeping

4 The client with diabetes mellitus who is taking NPH insulin (Humulin N) in the evening is most likely to become hypoglycemic shortly after midnight because this insulin peaks in 6 to 8 hours. The client should eat a bedtime snack to help prevent hypoglycemia while sleeping.

. A client with acute myeloid leukemia (AML) reports overhearing one of the other clients say that AML had a very poor prognosis. The client has understood that the client's physician informed the client that his physician told him that he has a good prognosis. Which is the nurse's best response? 1. "You must have misunderstood. Who did you hear that from?" 2. "AML does have a very poor prognosis for poorly differentiated cells." 3. "AML is the most common nonlymphocytic leukemia." 4. "Your doctor stated your prognosis based on the differentiation of your cells."

4 The statement "Your doctor stated your prognosis based on the differentiation of your cells" addresses the client's situation on an individual basis. The nurse is clarifying that clients have different prognoses—even though they may have the same type of leukemia—because of the cell differentiation. Stating that the client misunderstood is inappropriate for an advocate of the client and serves no useful purpose. The other statements are true but do not address this client's individual concern.

Alteplase recombinant, or tissue plasminogen activator (t-PA), a thrombolytic enzyme, is administered during the first 6 hours after onset of myocardial infarction (MI) to: 1. Control chest pain. 2. Reduce coronary artery vasospasm. 3. Control the arrhythmias associated with MI. 4. Revascularize the blocked coronary artery.

4 The thrombolytic agent t-PA, administered intravenously, lyses the clot blocking the coronary artery. The drug is most effective when administered within the first 6 hours after onset of MI. The drug does not reduce coronary artery vasospasm; nitrates are used to promote vasodilation. Arrhythmias are managed by antiarrhythmic drugs. Surgical approaches are used to open the coronary artery and re-establish a blood supply to the area.

Which client is at greatest risk for coronary artery disease? 1. A 32-year-old female with mitral valve prolapse who quit smoking 10 years ago. 2. A 43-year-old male with a family history of CAD and cholesterol level of 158 (8.8 mmol/L). 3. A 56-year-old male with an HDL of 60 (3.3 mmol/L) who takes atorvastatin. 4. A 65-year-old female who is obese with an LDL of 188 (10.4 mmol/L).

4 The woman who is 65 years old, overweight, and has an elevated LDL is at greatest risk. Total cholesterol greater than 200 (11.1 mmol/L), LDL greater than 100 (5.5 mmol/L), HDL less than 40 (2.2 mmol/L) in men, HDL less than 50 (2.8 mmol/L) in women, men 45 years and older, women 55 years and older, smoking and obesity increase the risk of CAD. Atorvastatin reduces LDL and decreases risk of CAD. The combination of postmenopausal, obesity, and high LDL places this client at greatest risk.

A client is to have a transsphenoidal hypophysectomy to remove a large, invasive pituitary tumor. The nurse should instruct the client that the surgery will be performed through an incision in the: 1. Back of the mouth. 2. Nose. 3. Sinus channel below the right eye. 4.Upper gingival mucosa in the space between the upper gums and lip.

4 With transsphenoidal hypophysectomy, the sella turcica is entered from below, through the sphenoid sinus. There is no external incision; the incision is made between the upper lip and gums.

The nurse is assessing a 48-year-old client with a history of smoking during a routine clinic visit. The client, who exercises regularly, reports having pain in the calf during exercise that disappears at rest. Which of the following findings requires further evaluation? 1. Heart rate 57 bpm. 2. SpO2 of 94% on room air. 3. Blood pressure 134/82. 4. Ankle-brachial index of 0.65.

4 An Ankle-Brachial Index of 0.65 suggests moderate arterial vascular disease in a client who is experiencing intermittent claudication. A Doppler ultrasound is indicated for further evaluation. The bradycardic heart rate is acceptable in an athletic client with a normal blood pressure. The SpO2 is acceptable; the client has a smoking history.

When assessing an individual with peripheral vascular disease, which clinical manifestation would indicate complete arterial obstruction in the lower left leg? 1. Aching pain in the left calf. 2. Burning pain in the left calf. 3. Numbness and tingling in the left leg. 4. Coldness of the left foot and ankle

4 Coldness in the left foot and ankle is consistent with complete arterial obstruction. Other expected findings would include paralysis and pallor. Aching pain, a burning sensation, or numbness and tingling are earlier signs of tissue hypoxia and ischemia and are commonly associated with incomplete obstruction.

The nurse is obtaining the pulse of a client who has had a femoral-popliteal bypass surgery 6 hours ago. (See below) Which assessment provides the most accurate information about the client's postoperative status? 1. radial pulse 2. femoral pulse 3. apical pulse 4. dorsalis pedis pulse

4 The presence of a strong dorsalis pedis pulse indicates that there is circulation to the extremity distal to the surgery indicating that the graft between the femoral and popliteal artery is allowing blood to circulate effectively. Answer 1 shows the nurse obtaining the radial pulse; answer 2 shows the femoral pulse, which is proximal to the surgery site and will not indicate circulation distal to the surgery site. Answer 3 shows the nurse obtaining an apical pulse.

A client who has been taking warfarin has been admitted with severe acute rectal bleeding and the following laboratory results: International Normalized Ratio (INR), 8; hemoglobin, 11 g/dL (110 g/L); and hematocrit, 33% (0.33). In which order should the nurse implement the following physician prescriptions? 1. Give 1 unit fresh frozen plasma (FFP). 2. Administer vitamin K 2.5 mg by mouth. 3. Schedule client for sigmoidoscopy. 4. Administer IV dextrose 5% in 0.45% normal saline solution.

4,1,2,3 Analysis of the client's laboratory results indicate that an INR of 8 is increased beyond therapeutic ranges. The client is also experiencing severe acute rectal bleeding and has a hemoglobin level in the low range of normal and a hematocrit reflecting fluid volume loss. The nurse should first establish an IV line and administer the dextrose in saline. Next the nurse should administer the FFP. FFP contains concentrated clotting factors and provides an immediate reversal of the prolonged INR. Vitamin K 2.5 mg PO should be given next because it reverses the warfarin by returning the PT to normal values. However, the reversal process occurs over 1 to 2 hours. Last, the nurse can schedule the client for the sigmoidoscopy.

A registered nurse is planning client assisgnments for the day. There is a licensed practical nurse and an assistive personnel (AP) on the team. Which client is the most appropriate choice for the AP? A client with hemophilia who needs assistance with shaving A client with pneumonia who requires frequent oropharyngeal suctioning A client with rheumatoid arthritis who needs assistance with feeding and ambulation A client with heart failure who needs daily weights and monitoring of intake and output

A client with rheumatoid arthritis who needs assistance with feeding and ambulation Rationale: The most appropriate client for the AP is the client with rheumatoid arthritis who needs assistance with feeding and ambulation. When a nurse delegates aspects of a client's care to another staff member, the nurse assigning the task is responsible for ensuring that each task is appropriately assigned on the basis of the educational level and competency of the staff member. A client with hemophilia is at risk for bleeding and should be cared for by a licensed nurse. Suctioning, an invasive treatment, must be performed by a licensed nurse. Although a AP may weigh a client and monitor intake and output, this client has heart failure and should be monitored by a licensed nurse who will be able to listen to breath sounds and detect changes in the client's status. Feeding a client, turning and repositioning a client, assisting with hygiene measures, and ambulating a client are noninvasive interventions and that may be assigned to a AP.

The nurse is caring for a newborn who was born to a mother infected with HIV. The nurse knows what manifestations of treatment are included in the care of this neonate? Select all that apply. The newborn must be isolated from the mother. All neonates born to HIV-positive clients will acquire infection. Administer zidovudine to the newborn as prescribed for the first 6 weeks. For all neonates born to HIV-positive clients, an HIV culture is recommended at 1 and 4 months after birth. All HIV-exposed newborns should be treated with medication to prevent infection by Pneumocystis jiroveci. Neonates born to HIV-positive clients never test positive at birth because antibodies received from the mother may persist for 18 months after birth.

Administer zidovudine to the newborn as prescribed for the first 6 weeks. For all neonates born to HIV-positive clients, an HIV culture is recommended at 1 and 4 months after birth. All HIV-exposed newborns should be treated with medication to prevent infection by Pneumocystis jiroveci. Rationale: The nurse should administer zidovudine to the newborn as prescribed for the first 6 weeks of life. Also, for all neonates born to HIV-positive clients, an HIV culture is recommended at 1 and 4 months after birth. In addition, all HIV-exposed newborns should be treated with medication to prevent infection by Pneumocystis jiroveci. The newborn can room with the mother. All neonates acquire maternal antibody to HIV infection, but not all acquire infection. Neonates born to HIV-positive clients may test positive because antibodies received from the mother may persist for 18 months after birth.

An emergency department nurse is told that a client with carbon monoxide poisoning resulting from a suicide attempt is being brought to the hospital by emergency medical services. Which intervention will the nurse carry out as a priority upon arrival of the client? Administering 100% oxygen Having a crisis counselor available Instituting suicide precautions for the client Obtaining blood for determination of the client's carboxyhemoglobin level

Administering 100% oxygen Rationale: With a client with carbon monoxide poisoning, the priority is to treat the client with inhalation of 100% oxygen to shorten the half-life of carbon monoxide to around an hour. Hyperbaric oxygen may be required to reduce the half-life to minutes by forcing the carbon monoxide off the hemoglobin molecule. Because the poisoning occurred as a result of a suicide attempt, a crisis counselor should be consulted, but this is not the priority. Suicide precautions should be instituted once emergency interventions have been completed and the client has been admitted to the hospital. The diagnosis is confirmed with a measurement of the carboxyhemoglobin level in the client's blood. Obtaining a blood specimen to measure the carboxyhemoglobin level is a priority; however, the nurse would immediately administer 100% oxygen to the client.

Fluticasone propionate and albuterol, administered by inhalation twice daily, are prescribed for a client with asthma. The nurse provides information to the client about administration of the medication. What does the nurse tell the client about how to take the medications? Fluticasone propionate immediately after inhaling the albuterol Albuterol immediately after inhaling the fluticasone propionate Fluticasone propionate several minutes before inhaling the albuterol Albuterol several minutes before inhaling the fluticasone propionate

Albuterol several minutes before inhaling the fluticasone propionate Rationale: In a client receiving a bronchodilator by way of inhalation concomitantly with corticosteroid therapy, the client is instructed to use the bronchodilator several minutes before corticosteroid inhalation to enhance penetration of the corticosteroid into the bronchial tree. Albuterol is a bronchodilator, and fluticasone propionate is a corticosteroid. The other options are incorrect.

The nurse is caring for a client who just returned to the surgical unit after having a suprapubic prostatectomy. What type of medication does the nurse expect to be ordered? Phenothiazines Antispasmodics Antidyskinetics Benzodiazepines

Antispasmodics Rationale: Antispasmodics are prescribed for bladder spasms related to a suprapubic prostatectomy. This surgery involves removal of the prostate gland by an abdominal incision with a bladder incision. Phenothiazines are a class of antipsychotic medications. Antidyskinetics have an anticholinergic action and are used to treat Parkinson's disease and some of the acute movement disorders that may be caused by antipsychotic agents. Benzodiazepines are central nervous system (CNS) depressants and can cause sedation and psychomotor slowing. They can also intensify depression caused by other drugs. Benzodiazepines have some potential for abuse and should be used with caution in clients known to abuse alcohol or other psychoactive medications.

A nurse is caring for a client with sarcoidosis. The client is upset because he has missed work and worried about how he will care financially for his wife and three small children. On the basis of the client's concern, which problem does the nurse identify? Anxiety Powerlessness Disruption of thought processes Inability to maintain health

Anxiety Rationale: Anxiety is a vague, uneasy feeling of apprehension. Some related factors include a threat or perceived threat to physical or emotional integrity or self-concept, changes in function in one's role, and threats to or changes in socioeconomic status. The client experiencing powerlessness expresses feelings of having no control over a situation or outcome. Disruption of thought processes involves disturbance of cognitive abilities or thought. Inability to maintain health is being incapable of seeking out help needed to maintain health.

A nurse is preparing to care for a client who was admitted to the antepartum unit at 34 weeks' gestation after an episode of vaginal bleeding resulting from total placenta previa. In report, the nurse is told that the client's vital signs are stable, that the fetal heart rate is 140 beats/min with a reassuring pattern, and that both the client and her husband are worried about the condition of the fetus. On reviewing the client's plan of care, which client concern does the nurse identify as the priority at this time? Anxiety Premature grief Fluid volume loss Fluid volume overload

Anxiety Rationale: Anxiety is the priority client concern identified by the nurse. Anxiety is vague uneasiness or discomfort that warns of trouble and enables an individual to approach and deal with the threat. Fluid volume loss indicates a hypovolemic state, whereas fluid volume overload indicates a hypervolemic state. Premature grief is a state in which an individual grieves before an actual loss. There is no information in the question to indicate that fluid volume loss, fluid volume overload, or premature grief are factors for concern.

A nurse prepares to administer digoxin to a client with heart failure. Which vital sign must be checked before the medication is administered? Temperature Respirations Apical pulse Blood pressure

Apical pulse Rationale: The nurse would count the client's apical heart rate for 60 seconds before giving the medication. Digoxin is a cardiac glycoside used to treat heart failure and control the ventricular rate in clients with atrial fibrillation. If the heart rate is slower than 60 beats/min, the medication is withheld and the primary health care provider notified. Excessive slowing of the heart rate is one clinical sign of digoxin toxicity. Although the nurse might check other vital signs, the apical pulse is the most important in this context.

A client with emphysema is receiving theophylline. While providing dietary instructions, what does the nurse tell the client is acceptable to consume? Cola Coffee Hot cocoa Apple juice

Apple juice Rationale: The nurse tells the client that apple juice is acceptable to consume. Theophylline is a methylxanthine bronchodilator. Caffeine is a methylxanthine with pharmacologic properties like those of theophylline. For this reason, caffeine can intensify the adverse effects of theophylline on the central nervous system and heart. In addition, caffeine competes with theophylline for drug-metabolizing enzymes, thereby causing the theophylline level to increase. Because of these interactions, individuals taking theophylline should avoid caffeine-containing beverages such as cola, coffee, tea, and cocoa, as well as other caffeine-containing products.

An emergency department (ED) nurse is monitoring a client with suspected acute myocardial infarction (MI) who is awaiting transfer to the coronary intensive care unit. The nurse notes the sudden onset of premature ventricular contractions (PVCs) on the monitor, checks the client's carotid pulse, and determines that the PVCs are not perfusing. What is the nurse's most appropriate action? Document the findings Ask the ED primary health care provider to check the client Continue to monitor the client's cardiac status Inform the client that PVCs are expected after an MI

Ask the ED primary health care provider to check the client Rationale: The most appropriate action by the nurse would be to ask the ED health care provider to check the client. PVCs are a result of increased irritability of ventricular cells. Peripheral pulses may be absent or diminished with the PVCs themselves because the decreased stroke volume of the premature beats may in turn decrease peripheral perfusion. Because other rhythms also cause widened QRS complexes, it is essential that the nurse determine whether the premature beats are resulting in perfusion of the extremities. This is done by palpating the carotid, brachial, or femoral artery while observing the monitor for widened complexes or by auscultating for apical heart sounds. In the situation of acute MI, PVCs may be considered warning dysrhythmias, possibly heralding the onset of ventricular tachycardia or ventricular fibrillation. Therefore, the nurse would not tell the client that the PVCs are expected. Although the nurse will continue to monitor the client and document the findings, these are not the most appropriate actions of those provided.

A nurse on the evening shift checks a primary health care provider's prescriptions and notes that the dose of a prescribed medication is higher than the normal dose. The nurse calls the primary health care provider's answering service and is told that the primary health care provider is off for the night and will be available in the morning. What should the nurse do next? Call the nursing supervisor Ask the answering service to contact the on-call primary health care provider Withhold the medication until the primary health care provider can be reached in the morning Administer the medication but consult the primary health care provider when he becomes available

Ask the answering service to contact the on-call primary health care provider Rationale: The nurse has a duty to protect the client from harm. A nurse who believes that a primary health care provider's prescription may be in error is responsible for clarifying the prescription before carrying it out. Therefore the nurse would not administer the medication; instead, the nurse would withhold the medication until the dose can be clarified. The nurse would not wait until the next morning to obtain clarification. It is premature to call the nursing supervisor.

A nurse is preparing to care for a preschool-age child with sickle cell anemia who is experiencing vasoocclusive pain. Which method of assessing the degree of pain the child is experiencing is most appropriate? Asking the child to describe the intensity of the pain Asking the child to use a numeric rating scale of 0 to 100 Asking the child whether the patient-controlled analgesia (PCA) pump is relieving the pain Asking the child to point to the face on a spectrum ranging from smiling to very sad, that best describes the pain

Asking the child to point to the face on a spectrum ranging from smiling to very sad, that best describes the pain Rationale: A client of preschool age has the cognitive ability to recognize happy and sad faces and to correlate them with the level of pain he or she is experiencing. It may be too complicated for some preschoolers to come up with words to describe varying intensities of pain. Many preschool children are not yet able to count to 100 or to understand the value of numbers in relation to other numbers. A child of preschool age is too young to control a PCA pump.

A male client arrives at the emergency department and reports to the nurse, "I woke up this morning and couldn't move my arms." He also tells the nurse that he works in a factory and witnessed an accident 3 weeks ago in which a fellow employee's hands were severed by a machine. What is the priority response by the nurse? Assessing the client for organic causes of loss of arm movement Calling the crisis intervention team and asking them to assess the client Performing active and passive range-of-motion (ROM) exercises of the client's arms Asking the client to move his arms and documenting the loss of movement he has experienced

Assessing the client for organic causes of loss of arm movement Rationale: The priority is assessing the client for organic causes of loss of arm movement and ruling out any neurological disorders. After it has been determined that there is no physiological basis for the problem, further psychiatric evaluation can be done. Encouraging the client to move his arms and performing active and passive ROM exercises have no beneficial effect in this situation. In fact, either option could be harmful if there is a physiological basis for the client's problem.

The nurse is performing a sterile change of an abdominal dressing. Once the dressing has been removed and discarded in a waterproof bag, which action should the nurse take next? Assessing the wound Donning sterile gloves Cleansing the wound Setting up the sterile field

Assessing the wound Rationale: View video. The nurse next assesses the wound for size; redness; swelling; and amount, color, odor, and type of drainage, if drainage is present. When performing a dressing change, the nurse dons clean gloves and removes the old dressing; checks the dressing for drainage, noting the amount, color, and odor if drainage is present; and discards the gloves and dressing. Next the nurse washes the hands and sets up the sterile field; dons sterile gloves; cleanses the wound with solution as prescribed, moving from the least to the most contaminated area; and redresses the wound. If a drain is present, the nurse applies additional layers of gauze as needed.

A client is brought to the emergency department after sustaining smoke inhalation. Humidified oxygen is administered to the client by way of face mask, and arterial blood gases (ABGs) are measured. ABG analysis indicates arterial oxygenation (Pao2) of less than 60 mm Hg (7.95 kPa). On the basis of the ABG result, what does the nurse prepare to do? Continue monitoring the client Increase the amount of humidified oxygen Continue administering humidified oxygen Assist in intubating the client and beginning mechanical ventilation

Assist in intubating the client and beginning mechanical ventilation Rationale: A client who sustains smoke inhalation is immediately treated with 100% humidified oxygen, delivered by way of face mask. An arterial oxygenation (Pao2) of less than 60 mm Hg (7.95 kPa) is an indication for intubation and mechanical ventilation. Normal arterial oxygenation is 80-100 mm Hg (10.6-13.33 kPa). Also, endotracheal intubation with mechanical ventilation is needed if the client exhibits respiratory stridor, crowing, or dyspnea, all of which indicate airway obstruction.

A client who was involved in a high-speed motor vehicle crash is brought to the emergency department. Which finding indicates to the nurse that the client has sustained a flail chest? Asymmetrical chest movement Complaints of mild chest discomfort Increased breath sounds on auscultation Deep respirations, 18 breaths/min

Asymmetrical chest movement Rationale: The nurse specifically notes paradoxical breathing (asymmetrical chest movement); oscillation of the mediastinum; increasing dyspnea; rapid, shallow respirations; accessory muscle breathing; restlessness; decreased breath sounds on auscultation; cyanosis; and anxiety related to difficulty breathing. The client will also complain of severe chest pain. Flail chest is a thoracic injury resulting in paradoxical motion of the chest-wall segment. This causes the mediastinal structures to swing back and forth with respiration.

A client is found to have posttraumatic stress disorder (PTSD) after witnessing a terrorist attack that caused the deaths of hundreds of people. The nurse, developing a plan of care for the client, identifies posttrauma syndrome as a concern and identifies a client outcome that states, "The client will cope effectively with thoughts and feelings of the event." Which nursing interventions will assist the client in achieving this outcome? Select all that apply. Being honest, nonjudgmental, and empathetic Assessing the immediate posttraumatic reaction Encouraging the client to keep a journal focused on the trauma Asking the client about the use of alcohol and drugs before and since the event Promoting discussion of the reasons the client was responsible for the traumatic event Discouraging the use of support groups until the client is able to use effective coping techniques

Being honest, nonjudgmental, and empathetic Assessing the immediate posttraumatic reaction Encouraging the client to keep a journal focused on the trauma Asking the client about the use of alcohol and drugs before and since the event Rationale: An honest, nonjudgmental, and empathetic attitude helps the nurse build a trusting relationship with the client. The nurse would assess the immediate posttraumatic reaction and later coping. Numbing and denial are common reactions after a traumatic event, and knowing the range of the client's behavior can help the nurse assess the impact and meaning of the trauma. Writing about the trauma in a journal can lessen the intensity of the client's emotions and his/her preoccupation with the event over time. The nurse would ask the client about the use of alcohol and drugs before and since the event. It is important for the nurse to obtain this information, because attempts to self-medicate to reduce anxiety and induce sleep are common after a traumatic event. The client needs to understand that he/she is not responsible for the event, but the nurse should emphasize that the client is responsible for learning to cope. This strategy will assist the client in easing feelings of powerlessness. The nurse would encourage attendance at support groups so that the client can share experiences, feel understood, and begin to heal.

A client is receiving an intravenous infusion of alteplase. For which adverse effect of the medication does the nurse monitor the client most closely? Bleeding Hearing loss Decreased urine output Increased blood pressure

Bleeding Rationale: Alteplase is a thrombolytic agent used to dissolve existing thrombi. The nurse needs to monitor the client most closely for bleeding. It is the most common adverse effect, and the nurse must monitor the client for obvious or occult signs of bleeding. Hearing loss and decreased urine output are not associated with the use of this medication. The medication may also cause a decrease in blood pressure and an allergic reaction, denoted by a rash or wheezing.

A nurse reviews the results of a total serum calcium determination in a client with chronic kidney disease. The results indicate a level of 12.0 mg/dL (3 mmol/L). In light of this result, which finding does the nurse expect to note during assessment? Decreased urine output Hyperactive bowel sounds Bounding, full peripheral pulses Hyperactive deep tendon reflexes

Bounding, full peripheral pulses Rationale: The normal total serum calcium level ranges from 8.6-10.2 mg/dL (2.15-2.55 mmol/L). Hypercalcemia occurs when the total serum calcium level exceeds 10 mg/dL (2.5 mmol/L). Some of the key features of hypercalcemia are increased heart rate and blood pressure; bounding, full peripheral pulses; ineffective respiratory movement related to profound skeletal muscle weakness; disorientation; diminution or absence of deep tendon reflexes; increased urine output; and hypoactive bowel sounds.

Prednisone 5 mg/day has been prescribed for a client with a chronic respiratory disorder, and the nurse provides instructions to the client about the medication. What does the nurse tell the client to do? Take the medication on an empty stomach Take half of the daily dose if weight gain occurs Stop taking the medication if the ankles begin to swell Call the primary health care provider if a fever, sore throat, or muscle aches develop

Call the primary health care provider if a fever, sore throat, or muscle aches develop Rationale: Prednisone is a glucocorticoid that prevents or diminishes tissue response to an inflammatory process. The client taking this medication should notify the primary health care provider if fever, sore throat, muscle aches, or sudden weight gain or swelling occurs. The medication is taken with food or milk because it can cause gastrointestinal irritation. The client is instructed not to change the dose and to not stop taking the medication without contacting the primary health care provider. The medication must be tapered gradually under medical supervision.

A nurse has assisted a primary health care provider in inserting a central venous access device into a client with a diagnosis of severe malnutrition who will be receiving parenteral nutrition (PN). After insertion of the catheter what does the nurse immediately do? Call the radiography department to obtain a chest x-ray Check the client's blood glucose level to serve as a baseline measurement Hang the prescribed bag of PN and start the infusion at the prescribed rate Infuse normal saline solution through the catheter at a rate of 100 mL/hr to maintain patency

Call the radiography department to obtain a chest x-ray Rationale: The nurse should immediately make arrangements to have a chest x-ray done. One major complication associated with central venous catheter placement is pneumothorax, which may result from accidental puncture of the lung. After the catheter has been placed but before it is used for infusions, its placement must be checked with an x-ray. Hanging the prescribed bag of PN and starting the infusion at the prescribed rate and infusing normal saline solution through the catheter at a rate of 100 mL/hr to maintain patency are all incorrect because they could result in the infusion of solution into a lung if a pneumothorax is present. Although the nurse may obtain a blood glucose measurement to serve as a baseline, this action is not the priority.

An intravenous dose of adenosine is prescribed for a client to treat Wolff-Parkinson-White syndrome. Which piece of equipment does the nurse make a priority of obtaining before administering the medication? Pulse oximeter Cardiac monitor Blood-pressure cuff Suction catheter and suction machine

Cardiac monitor Rationale: Obtaining a cardiac monitor is the priority. Wolff-Parkinson-White syndrome is an abnormality of cardiac rhythm that is manifested as supraventricular tachycardia. Adenosine is an antidysrhythmic medication used to treat this dysrhythmia. It is administered intravenously. A pulse oximeter and blood pressure cuff will each provide information about the client's cardiovascular status, but neither is the priority. There is no information in the question to indicate that a suction catheter and suction machine are necessary.

The nurse is caring for a newborn recently diagnosed with gastroschisis. What does the nurse know about this condition? Select all that apply. Care similar to that for omphalocele Viscera are inside the abdominal cavity Surgery performed within several hours after birth Exposed bowel directly wrapped in saline-soaked pads Postoperatively most infants develop prolonged ileus Results from a defect in the abdominal wall

Care similar to that for omphalocele Surgery performed within several hours after birth Postoperatively most infants develop prolonged ileus Results from a defect in the abdominal wall Rationale: The nurse knows that gastroschisis results from a defect in the abdominal wall in which the viscera is outside, not inside, the abdominal cavity and are also not covered by a sac. Care of the newborn is similar to that for newborns with omphalocele. The exposed bowel is covered loosely in saline-soaked pads, not directly wrapped around the bowel. Surgery needs to be performed within several hours after birth, and postoperatively most newborns develop prolonged ileus.

The nurse is caring for a client who has just undergone esophagogastroduodenoscopy (EGD). The client says to the nurse, "I'm really thirsty — may I have something to drink?" Before giving the client a drink, what would the nurse do? Check the client's vital signs Check for the presence of a gag reflex Assess the client for the presence of bowel sounds Ask the client to gargle with a warm saline solution

Check for the presence of a gag reflex Rationale: After an EGD, the nurse places the highest priority on assessing the client for the return of the gag reflex. In preparation for EGD, the client's throat is usually sprayed with an anesthetic to dampen the gag reflex and permit the introduction of the endoscope used to visualize the gastrointestinal structures. No food or oral fluids are given to the client until the gag reflex is fully intact.Vital signs are checked frequently, but this action is not associated with giving the client oral fluids. The client may be asked to use throat lozenges or a saline gargle to relieve a sore throat after the test, but neither action is related to giving the client oral fluids; additionally, neither action would be taken until the gag reflex had been detected again. Bowel sounds are not affected by this test.

The nurse notes the presence of drainage on the mustache dressing of a client who has undergone transsphenoidal hypophysectomy. What should be the nurse's initial action? Contact the surgeon Change the dressing Document the findings Check the drainage for glucose

Check the drainage for glucose Rationale: The initial nursing action is to check the drainage for glucose. If glucose is present in the drainage, the surgeon must be notified. After hypophysectomy, the nurse assesses the client's nasal drainage for quantity, quality, and the presence of glucose, which indicates that the fluid is cerebrospinal fluid (CSF). The nurse would not change the dressing unless a specific prescription to do so has been given. The nurse would document the findings, but this would not be the initial action.

A nurse taking the vital signs of a client immediately after she has delivered a newborn notes that the client's heart rate is 110 beats/min. What should the nurse do first? Document the findings Offer the client oral fluids Recheck the heart rate in 1 hour Check the uterus and amount of lochia discharge

Check the uterus and amount of lochia discharge Rationale: If tachycardia is noted, the nurse should first assess the location and firmness of the uterus and amount of lochia. Additional assessments including blood pressure, estimated blood loss at delivery, and hemoglobin and hematocrit determinations should be carried out. After delivery, the normal heart rate ranges from 60 to 90 beats/min. Tachycardia may indicate excitement, fatigue, dehydration, hypovolemia, pain, or infection. Although the nurse would document the findings, it is most appropriate for the nurse to assess the client to determine the cause of the tachycardia. Oral fluids are important if the client is dehydrated, but further assessment of the problem is required and dehydration would first need to be confirmed. Rechecking the heart rate in 1 hour will delay necessary interventions.

The nurse is preparing to administer a tube feeding by way of a nasogastric tube. Which action does the nurse carry out as a priority before starting the flow of the solution? Flushing the tube with 30 mL of tap water Checking urine output in the previous 24 hours Scrubbing the port with povidone-iodine solution Checking for gastric residual volume and assessing tube placement

Checking for gastric residual volume and assessing tube placement Rationale: View video. When a client has a nasogastric tube, a priority concern is the risk for aspiration. The nurse would not administer a feeding or flush the tube unless residual volume and tube placement had been checked. To check residual volume, the nurse would draw up 30 mL of air in a catheter-tipped syringe, attach it to the feeding tube, inject the air, and then pull back the plunger to aspirate all of the gastric contents. The gastric contents are checked for amount and appearance and then the pH is checked for acidity to ensure accurate placement. It is not necessary to scrub the port with povidone-iodine solution; administering a tube feeding is a clean, not sterile procedure. Although intake and output may be monitored, it is not necessary to check urine output for the previous 24 hours before starting a feeding.

Methylergonovine is prescribed for a client to control postpartum bleeding. Which action does the nurse take before administering the medication? Checking the episiotomy site Palpating the client's bladder Checking the client's blood pressure Ensuring that the uterus is contracted

Checking the client's blood pressure Rationale: Because the medication causes arterial vasoconstriction and hypertension, the nurse checks the client's blood pressure before administering the medication. Methylergonovine is an oxytocic that stimulates contraction of the uterus and causes arterial vasoconstriction. It is used for the prevention and treatment of postpartum and postabortal hemorrhage caused by uterine atony or subinvolution. There is no information to indicate that the client has had an episiotomy. Although the nurse may palpate the client's bladder, this action is unrelated to the use of the medication.

Enalapril maleate is prescribed for a hospitalized client. Which assessment does the nurse perform as a priority before administering the medication? Checking the client's blood pressure Checking the client's peripheral pulses Checking the most recent potassium level Checking the client's intake-and-output record for the last 24 hours

Checking the client's blood pressure Rationale: Enalapril maleate is an angiotensin-converting enzyme (ACE) inhibitor used to treat hypertension. One common side effect is postural hypotension. Therefore the nurse would check the client's blood pressure immediately before administering each dose. Checking the client's peripheral pulses, the results of the most recent potassium level, and the intake and output for the previous 24 hours are not specifically associated with this mediation.

A nurse administers nitroglycerin sublingually to a client diagnosed with angina pectoris who reports chest pain. The medication is ineffective, so the nurse prepares to administer a second dose. Before administering the nitroglycerin, which action does the nurse make a priority? Checking the client's blood pressure Obtaining blood levels of cardiac enzymes Asking the client if experiencing headache Obtaining a 12-lead electrocardiogram (ECG)

Checking the client's blood pressure Rationale: Nitroglycerin is a nitrate that dilates the coronary arteries. One adverse effect of the medication is hypotension, and the nurse would assess the blood pressure and apical pulse before administration and periodically after the dose is given. Blood levels of cardiac enzymes are obtained if prescribed, but the priority is checking the client's blood pressure. Headache is a frequent side effect of the medication, mostly early in therapy and usually disappearing with continued treatment. It is not necessary to obtain a 12-lead ECG before administering a second dose of nitroglycerin unless this is prescribed by the primary health care provider. However, the client receiving intravenous nitroglycerin must have continuous ECG monitoring.

A nurse provides dietary instruction to a hospitalized client with chronic obstructive pulmonary disease (COPD). Which menu selection by the client tells the nurse that the client understands the instructions? Coffee Broccoli Cheeseburger Chocolate milk

Cheeseburger Rationale: The client with COPD is encouraged to eat a high-calorie, high-protein diet and to choose foods that are easy to chew and do not promote gas formation. Dry foods stimulate coughing, and foods such as milk and chocolate may increase the thickness of saliva and other secretions. The nurse advises the client to avoid these foods, as well as caffeinated beverages, which promote diuresis, contributing to dehydration, and may increase nervousness.

Vasopressin is prescribed to a client with diabetes insipidus. For which sign/symptom, indicative of an adverse effect of the medication, does the nurse monitor the client? Chest pain Constipation Loss of appetite Decreased urine output

Chest pain Rationale: Chest pain is a sign of an adverse effect of vasopressin, and if it occurs, the primary health care provider must be notified. Because of its powerful vasoconstrictive action, vasopressin can cause adverse cardiovascular effects. By constricting arteries of the heart, vasopressin can cause angina pectoris and even myocardial infarction, especially if the medication is administered to a client with existing coronary artery disease. Vasopressin may also cause gangrene by decreasing blood flow in the periphery. Decreased urine output is an expected effect. Neither constipation nor loss of appetite is associated with this medication.

A client with myasthenia gravis who has been taking pyridostigmine bromide for the treatment of the disorder comes to the emergency department complaining of severe muscle weakness, and cholinergic crisis is diagnosed. What muscles are involved first in a cholinergic crisis? Respiratory muscles Chewing and swallowing Pelvic girdle and leg muscles Shoulder girdle and upper extremities

Chewing and swallowing Rationale: Cholinergic crisis is defined as an over-stimulation at a neuromuscular junction due to an excess of acetylcholine (ACh) caused by an overdose of anticholinesterase drugs. The weakness first appears in the muscles involved in chewing and swallowing and is followed by weakness of the shoulder girdle and upper extremities, respiratory muscles, and pelvic girdle and leg muscles.

A client receiving parenteral nutrition (PN) suddenly experiences chest pain and dyspnea, and the nurse suspects an air embolism. The nurse immediately places the client in a lateral Trendelenburg position, on the left side. What action does the nurse take next? Auscultating heart sounds Clamping the intravenous catheter Checking the client's blood pressure Obtaining an arterial blood gas specimen

Clamping the intravenous catheter Rationale: The nurse next clamps the intravenous catheter and notifies the primary health care provider. The signs/symptoms of air embolism include chest pain, dyspnea, and hypoxia. Tachycardia and hypotension will also be present, and the client will experience anxiety. The nurse will also hear a loud churning sound on auscultation over the pericardium. The nurse immediately places the client in a lateral Trendelenburg position, on the left side. This position prevents air from flowing into the pulmonary veins. Although auscultating heart sounds, checking the client's blood pressure, and obtaining an arterial blood gas specimen may be appropriate interventions, none is the next action to take after positioning the client.

A client arrives at the clinic for her first prenatal assessment. The client tells the nurse that the first day of her last menstrual period (LMP) was September 25, 2017. Using Nagele's rule, what item of client information is needed for the nurse to accurately determine estimated date of delivery (EDD)? Client has never had an abortion Client has regular 28-day menstrual cycle Client was 14 years old when menses first started Client's menstrual periods never last longer than 3 days

Client has regular 28-day menstrual cycle Rationale: Accurate use of Nagele's rule is used to calculate the EDD. It requires that the woman have a regular 28-day menstrual cycle.

A client with a medical history of diabetes mellitus is found to have sarcoidosis and oral prednisone is prescribed. The nurse provides instructions to the client about the medication. What does the nurse tell the client? Eat foods that are high in sodium Decrease the daily dose of insulin Eat foods that are low in potassium Closely monitor the blood glucose level

Closely monitor the blood glucose level Rationale: The client with diabetes mellitus who is taking prednisone would be instructed to closely monitor the blood glucose level. When taken over an extensive period of time, prednisone may cause increased blood glucose and decreased potassium. It may also cause increased blood pressure and sodium retention. Sarcoidosis is a multisystem granulomatous disorder of unknown cause that can affect virtually any organ. Corticosteroids are the primary pharmacological therapy for sarcoidosis. The primary health care provider may prescribe an increased dose of insulin while the client is taking this medication.

An emergency department nurse assessing a client with Bell's palsy collects subjective and objective data. Which finding does the nurse expect to note? A symmetrical smile Tightening of all facial muscles Ability to wrinkle the forehead on request Complaints of inability to close the eye on the affected side

Complaints of inability to close the eye on the affected side Rationale: The onset of Bell's palsy is acute. Maximal paralysis occurs within 5 days in almost all clients. Pain behind the ear or on the face may precede paralysis by a few hours or days. The disorder is characterized by a drawing sensation and paralysis, not tightening, of all facial muscles on the affected side. The client cannot close the eye, wrinkle the forehead, smile, whistle, or grimace. The face appears masklike and sags. Taste is usually impaired to some degree, but this symptom seldom persists beyond the second week of paralysis. Loss of peripheral vision is not associated with Bell's palsy.

A nurse is monitoring a client receiving terbutaline by intravenous infusion to stop preterm labor. The nurse notes that the client's heart rate is 120 beats/min and that the fetal heart rate is 170 beats/min. What is the most appropriate action the nurse should take? Contact the primary health care provider Document the findings Continue to monitor the client Increase the rate of the infusion

Contact the primary health care provider Rationale: Although the nurse would document the findings, the most appropriate priority action in this scenario is to contact the primary health care provider. The nurse should monitor the client for adverse effects and notifiy the primary health care provider if the maternal heart rate is faster than 110 beats/min, respiration is faster than 24 breaths/min, systolic blood pressure is less than 90 mm Hg, the fetal heart rate is faster than 160 beats/min, or the client complains of chest pain or dyspnea. Terbutaline may be used to stop preterm labor. It stimulates beta-adrenergic receptors of the sympathetic nervous system, resulting in bronchodilation and inhibition of uterine muscle activity. Increasing the rate of infusion and continuing to monitor the client are inappropriate and delay necessary interventions.

A nurse preparing to administer digoxin to a client calls the laboratory for the result of the digoxin assay performed on a specimen that was drawn at 6 a.m. The laboratory reports that the result was 2.0 ng/mL (2.6 nmol/L). On the basis of this result, what should the nurse do? Administer the digoxin Contact the primary health care provider Wait for the primary health care provider to make rounds and report the result Check the client's apical heart rate and administer the digoxin if the rate is faster than 60 beats/min

Contact the primary health care provider Rationale: The nurse should contact the primary health care provider. The optimal therapeutic serum level for digoxin is 0.5 to 0.8 ng/mL (0.6 to 1.0 nmol/L). A digoxin concentration greater than this level indicates toxicity, and requires the nurse to contact the primary health care provider. The remaining options are inappropriate because they could delay necessary and immediate intervention, resulting in harm to the client.

A client in labor experiences spontaneous rupture of the membranes. The nurse immediately counts the fetal heart rate (FHR) for 1 full minute and then checks the amniotic fluid. The nurse notes that the fluid is yellow and has a strong odor. Which action should be the nurse's priority? Contact the primary health care provider Document the findings Check the fluid for protein Continue to monitor the client and the FHR

Contact the primary health care provider Rationale: The priority action is for the nurse to contact the primary health care provider. The FHR is assessed for at least 1 minute when the membranes rupture. The nurse also checks the quantity, color, and odor of the amniotic fluid. The fluid should be clear (often with bits of vernix) and have a mild odor. Fluid with a foul or strong odor, cloudy appearance, or yellow coloration suggests chorioamnionitis and warrants notifying the primary health care provider. A large amount of vernix in the fluid suggests that the fetus is preterm. Greenish, meconium-stained fluid may be seen in cases of postterm gestation or placental insufficiency. Checking the fluid for protein is not associated with the data in the question. The nurse would continue to monitor the client and the FHR and would document the findings.

A nurse provides home care instructions to a client who has undergone fluorescein angiography. What comment by the client helps the nurse determine that the client needs further instruction? Drink fluids to eliminate the dye Contact the primary health care provider if the skin appears yellow Expect that the urine will be bright green until the dye has been excreted Wear sunglasses and avoid direct sunlight until pupil dilation returns to normal

Contact the primary health care provider if the skin appears yellow Rationale: The client needs further instruction if the client plans to contact the primary health care provider if the skin appears yellow. The nurse also encourages rest and emphasizes that any yellow discoloration of the skin will disappear in a few hours. Fluorescein angiography provides a detailed image and permanent record of eye circulation. Photographs are taken in rapid succession after the intravenous administration of dye. After the test, the client may feel weak and nauseated, and the urine will be bright green until the dye has been excreted. Once the nausea has resolved, the client is encouraged to drink fluids to eliminate the dye, and instructed to avoid direct sunlight until pupil dilation returns to normal.

A school nurse observing a child diagnosed with Down syndrome is participating in a physical education class and notes that the child is experiencing a diminution in motor abilities. The nurse asks to see the child and conducts an assessment, during which the child complains of neck pain and loss of bladder control. What is the appropriate action by the nurse? Contacting the child's primary health care provider to report the findings Administering acetaminophen to the child to relieve the pain Asking that the child not attend the physical education class until the neck pain has subsided Teaching the child how to use peripads to prevent embarrassment resulting from loss of bladder control

Contacting the child's primary health care provider to report the findings Rationale: Children with Down syndrome who participate in sports that may involve stress on the neck should be evaluated radiologically for atlantoaxial instability. Signs/symptoms of the disorder include neck pain, weakness, and torticollis. Affected children are at risk for spinal cord compression. Signs/symptoms of spinal cord compression include persistent neck pain, loss of established motor skills and bladder or bowel control, and changes in sensation. If any of these signs/symptoms are noted, it must be reported immediately to the primary health care provider. Administering acetaminophen to relieve the pain and not allowing the child to attend physical education class until the pain has subsided are inappropriate actions that will delay necessary interventions. Teaching the child to use peripads to prevent embarrassment resulting from loss of bladder control may be appropriate in certain scenarios, but in this situation the child is exhibiting signs/symptoms of spinal cord compression, requiring immediate intervention.

A home care nurse visits a mother who delivered a healthy newborn 4 days ago and assesses how the mother is doing breastfeeding her infant. What does the nurse ask the mother to do to permit assessment of whether the infant is receiving an adequate amount of milk? Count the number of times that the infant swallows during a feeding Weigh the infant every day and check for a daily weight gain of 2 oz (60 ml) Count wet diapers to be sure that the infant is having at least six to 10 each day Pump the breasts, place the milk in a bottle, measure the amount, and then bottle-feed the infant

Count wet diapers to be sure that the infant is having at least six to 10 each day Rationale: The mother should be taught to count wet and soiled diapers to help determine whether the infant is receiving enough milk. Generally an infant should have at least 6 to 10 wet diapers (after the first 2 days of life) and at least 4 stools each day. The mother may also assess the swallowing and nutritive suckling of the infant, but this would not provide the best indication of adequate milk intake. Counting the number of times that the infant swallows during a feeding is an inadequate indicator of milk intake. The mother is not usually encouraged to weigh the infant at home, because this focuses too much attention on weight gain. Infants generally gain approximately 15 to 30 g (0.5 to 1 oz) each day after the early months of life. Pumping the breasts, placing the milk in a bottle, measuring the amount, and then bottle-feeding the infant constitute an assessment of the mother's bottle-feeding technique.

A nurse is developing a plan of care for a pregnant client with sickle-cell disease. Which concern does the nurse recognize as the priority? Inability to cope Decreased nutrition Decreased fluid volume Inability to tolerate activity

Decreased fluid volume Rationale: Decreased fluid volume is the priority concern in this situation, followed by decreased nutrition. Inability to tolerate activity and inability to cope compete for third priority, depending on the client's specific signs/symptoms at the time. Sickle cell disease is a genetic disorder that is manifested as chronic anemia, pain, disability, organ damage, increased risk for infection, and early death. In this disorder the red blood cells assume a sickle shape, become rigid, and clump together. Dehydration can precipitate sickling of the red blood cells. Sickling can lead to life-threatening consequences for the pregnant woman and the fetus, including interruption of blood flow to the respiratory system and placenta.

A nurse, conducting an assessment of a client being seen in the clinic for signs/symptoms of a sinus infection, asks the client about medications that he is taking. The client tells the nurse that he is taking nefazodone hydrochloride. On the basis of this information, the nurse determines that the client most likely has a history of what problem? Depression Diabetes mellitus Hyperthyroidism Coronary artery disease

Depression Rationale: The client is most likely suffering from depression. Nefazodone hydrochloride is an antidepressant used as maintenance therapy to prevent relapse of an acute depression. Diabetes mellitus, hypethyroidism, and coronary artery disease are not treated with this medication.

A nurse in a primary health care provider's office is talking to a client who underwent mastectomy of the right breast 2 weeks ago. The client says to the nurse, "I hate looking at this incision. I feel that I'm not even myself anymore." The nurse interprets this statement to mean that the client is experiencing which problem? Inability to cope Distorted body image Inability to care for self Inability to maintain health

Distorted body image Rationale: Distorted body image is characterized by negative verbalizations or feelings about a body part. This is a common response after mastectomy. The nurse supports the client and helps her work through these feelings. There is no information in the question to indicate that inability to care for self, inability to cope, or inability to maintain health is a problem.

A nurse is monitoring a pregnant woman in labor and notes this finding on the fetal-monitor tracing (see figure). Which action should the nurse take as a result of this observation? Reposition the mother Document the finding Notify the nurse-midwife Take the mother's vital signs

Document the finding Rationale: The nurse sees evidence of accelerations. Accelerations are transient increases in the fetal heart rate that often accompany contractions and are normally caused by fetal movement. Ths nurse should document the finding. Accelerations are thought to be a sign of fetal well-being and adequate oxygen reserve. Repositioning the mother, notifying the nurse-midwife, and taking the mother's vital signs are all unnecessary actions.

A nurse caring for a client 24 hours after a radical neck dissection notes the presence of serosanguineous drainage in the portable wound suction device attached to the surgical site. On the basis of this finding, what should the nurse do? Document the findings Contact the primary health care provider Ask the primary health care provider to remove the drains Increase the pressure on the wound suction device

Document the findings Rationale: Because the findings detailed in the question are expected, the nurse would document them. Immediately after radical neck dissection, the client will have a wound drain in the neck, attached to portable suction. The nurse places the client in the semi-Fowler position to minimize postoperative edema and monitors the neck drainage for volume and color. Sanguineous and serosanguineous drainage is expected in the 72 hours after surgery. Once drainage has stopped, the wound drains are removed.

A nurse in a primary health care provider's office is conducting a 2-week postpartum assessment of a client. During abdominal assessment, the nurse is unable to palpate the uterine fundus. This finding would prompt what action by the nurse? Document the findings Ask the primary health care provider to see the client immediately Ask another nurse to check for the uterine fundus Place the client in the supine position for 5 minutes, then recheck the abdomen

Document the findings Rationale: By the 10th to 14th day, the fundus is in the pelvic cavity and cannot be palpated abdominally. Involution is the progressive descent of the uterus into the pelvic cavity after delivery. Twenty-four hours after birth, descent of the fundus begins at a rate of approximately 1 fingerbreadth, or approximately 1 cm, per day. Asking the primary health care provider to see the client immediately, having another nurse check for the uterine fundus, and placing the client in the supine position for 5 minutes and rechecking the abdomen are all incorrect and unnecessary actions in light of the assessment finding.

A client who is taking lithium carbonate complains of mild nausea and voiding in large volumes. On assessment, the nurse notes that the client is also complaining of mild thirst. On the basis of these findings, what would the nurse do? Contact the primary health care provider Document the findings Institute seizure precautions Have a blood specimen drawn immediately for serum lithium testing

Document the findings Rationale: Lithium carbonate is a mood stabilizer that is used to treat manic-depressive illness. Side effects include polyuria, mild thirst, and mild nausea. Therefore, the nurse should simply document the findings. Because the client's complaints are side effects, not toxic effects, contacting the primary health care provider, instituting seizure precautions, and having a specimen drawn immediately for a serum lithium determination are all unnecessary. Vomiting, diarrhea, muscle weakness, tremors, drowsiness, and ataxia are signs/symptoms of toxicity and if these occur the primary health care provider needs to be notified.

The nurse assessing the deep tendon reflexes of a pregnant client notes that the reflexes are 1+ (i.e., reflex present, hypoactive). On the basis of this finding, what should the nurse do? Document the findings Contact the primary health care provider Ask the client to walk for 5 minutes, then recheck the reflexes Perform active and passive range-of-motion exercises of the client's lower extremities, then recheck the reflexes

Document the findings Rationale: The scale for rating deep tendon reflexes is as follows: 0 = absent; 1+ = present, hypoactive; 2+ = normal; 3+ = hyperactive; 4+ = hyperactive with clonus. Deep tendon reflexes should be 1+ or 2+. Reflexes that are brisker than average and hyperactive reflexes (3+ to 4+) suggest preeclampsia and must be reported to the primary health care provider. It is not necessary to contact the primary health care provider, because the finding is normal. Likewise, rechecking the client's reflexes after ambulation and performing active and passive ROM exercises incorrect and unnecessary actions.

The blood serum level of imipramine is determined in a client who is being treated for depression. The laboratory test indicates a concentration of 250 ng/mL. On the basis of this result, what should the nurse do? Contact the primary health care provider Hold the next dose of imipramine Document the laboratory result in the client's record Have another blood sample drawn and ask the laboratory to recheck the imipramine level

Document the laboratory result in the client's record Rationale: Imipramine is a tricyclic antidepressant that is often used to treat depression. The therapeutic blood serum level is between 225 and 300 ng/mL, so the nurse would simply document the laboratory result in the client's record. Asking the laboratory to recheck the level and withholding the next dose of the imipramine and contacting the primary health care provider are unnecessary.

A client diagnosed with adenocarcinoma of the ovary is scheduled to undergo chemotherapy with cyclophosphamide after total abdominal hysterectomy with bilateral salpingo-oophorectomy. What does the nurse instruct the client to do during chemotherapy? Select all that apply. Eat foods that are low in fat and protein Obtain pneumococcal and influenza vaccines Drink copious amounts of fluid and void frequently Avoid contact with any individual who has signs/symptoms of a cold Avoid contact with all individuals other than immediate family members

Drink copious amounts of fluid and void frequently Avoid contact with any individual who has signs/symptoms of a cold Rationale: Hemorrhagic cystitis is an adverse effect of this medication. The client is encouraged to drink copious amounts of fluid at least 24 hours before, during, and after chemotherapy, and avoid contact with individuals who are ill, have a cold, or have recently received a live-virus vaccine. The client is also encouraged to void frequently to prevent cystitis. The client is not to receive immunizations without the primary health care provider's approval, because they could diminish the body's resistance, putting the client at increased risk for infection. It is not necessary for the client to avoid contact with all individuals other than immediate family members. The client should, however, avoid contact with individuals who are ill, have a cold, or have recently received a live-virus vaccine. Encouraging adequate dietary intake is appropriate, but a low-protein or low-fat diet is not necessary.

A nurse is providing information about home care to a client with acute gout. Which measures does the nurse tell the client to take? Select all that apply. Drinking 2 to 3 L of fluid each day Applying heat packs to the affected joint Resting and immobilizing the affected area Consuming foods high in purines Performing range-of-motion exercise to the affected joint three times a day

Drinking 2 to 3 L of fluid each day Resting and immobilizing the affected area Rationale: Gout is a systemic disease in which urate crystals are deposited in the joints and other tissues, resulting in inflammation. In acute gout, rest and immobilization are recommended until the acute attack and inflammation have subsided. Local application of cold may help relieve the pain. The application of heat is avoided because it may worsen the inflammatory process. Dietary instructions include reducing or eliminating alcohol intake and avoiding excessive intake of foods containing purines (e.g., sweetbreads, yeast, heart, herring, herring roe, sardines). The client is encouraged to drink 2 to 3 L of fluid per day to help eliminate uric acid and to prevent the formation of renal calculi.

A nurse is performing an assessment of a newborn with a diagnosis of esophageal atresia (EA) and tracheoesophageal fistula (TEF). Which findings does the nurse expect to note in the infant? Select all that apply. Drooling Wheezing Hiccuping Short periods of apnea Excessive oral secretions Bowel sounds over the chest

Drooling Excessive oral secretions Rationale: EA with or without TEF results in excessive oral secretions, drooling, and feeding intolerance. EA and TEF, the most life-threatening anomalies of the esophagus, often occur together, although they may occur singly. EA is a congenital anomaly in which the esophagus ends in a blind pouch or narrows into a thin cord, thereby failing to form a continuous passageway to the stomach. TEF is an abnormal connection between the esophagus and trachea. When fed, the infant may swallow but will then cough and gag and return the fluid through the nose and mouth. Bowel sounds over the chest is a clinical manifestation associated with congenital diaphragmatic hernia. Hiccuping and spitting up after a meal are clinical manifestations of gastroesophageal reflux. Coughing, wheezing, and short periods of apnea are clinical manifestations of hiatal hernia.

Cyclobenzaprine is prescribed to a client with multiple sclerosis for the treatment of muscle spasms. For which common side effect of this medication does the nurse monitor the client? Diarrhea Drowsiness Abdominal pain Increased salivation

Drowsiness Rationale: Drowsiness, dizziness, and dry mouth are the most common side effects of cyclobenzaprine. Cyclobenzaprine is a centrally acting skeletal muscle relaxant used in the management of muscle spasm accompanying a variety of conditions. Rare side effects include fatigue, tiredness, blurred vision, headache, nervousness, confusion, nausea, constipation, dyspepsia, and an unpleasant taste in the mouth.

A nurse provides information to a client who will be undergoing endoscopic retrograde cholangiopancreatography (ERCP). What does the nurse tell the client? There is no need to fast (NPO status) before the procedure The gallbladder is easily removed during this procedure if gallstones are found The procedure is only performed to visualize the esophagus, stomach, and duodenum Dye may be injected during the procedure to permit visualization of the pancreatic and biliary ducts

Dye may be injected during the procedure to permit visualization of the pancreatic and biliary ducts Rationale: The nurse tells the client that dye may be injected to outline the pancreatic and biliary ducts. ERCP involves the oral insertion of an endoscope with a side-viewing tip and a cannula that can be maneuvered into the ampulla of Vater. The procedure may be combined with papillotomy to enlarge the sphincter and release gallstones. However, the gallbladder itself cannot be removed during this procedure. As with any endoscopic procedure, the client must remain NPO for 8 hours before the test.

A mother calls the emergency department and tells the nurse that her 3-year-old child drank ammonia from a bottle while the mother was cleaning house. What does the nurse tell the mother to immediately do? Induce vomiting Call the child's primary health care provider Bring the child to the emergency department Encourage the child to drink water or milk in small amounts

Encourage the child to drink water or milk in small amounts Rationale: The nurse tells the mother to immediately administer water or milk to dilute the toxic effects of acid or alkali ingestion. These substances, when ingested, may cause burning of tissue along the gastrointestinal tract. Because these caustic substances continue to cause damage until they are neutralized, induction of emesis is contraindicated. Although calling the child's primary health care provider and bringing the child to the emergency department may each be necessary, they are not the actions to be taken immediately, because they would delay necessary treatment.

A nurse is preparing to care for a client who has undergone abdominal hysterectomy for the treatment of endometrial cancer. What does the nurse determine is the priority in the 24 hours after surgery? Monitoring the client for signs of returning peristalsis Instructing the client in dietary changes to prevent constipation Encouraging the client to deep-breathe, cough, and use an incentive spirometer Encouraging the client to talk about the effects of the surgery on her femininity and sexuality

Encouraging the client to deep-breathe, cough, and use an incentive spirometer Rationale: The nurse determines that the priority in the 24 hours after surgery is to encourage the client to deep-breathe, cough, and use an incentive spirometer. Care after abdominal hysterectomy includes maintenance of a patent airway, promotion of circulation and oxygenation, promotion of comfort, monitoring of output and drainage, promotion of elimination, and discharge teaching with regard to medications and therapeutic regimens. The priority is the maintenance of a patent airway and promotion of oxygenation and circulation. Monitoring the client for signs/symptoms of returning peristalis, instructing her in dietary habits to prevent constipation, and encouraging her to talk about the effects of her surgery are also components of care after this surgery but are of lower priority than encouraging the client to deep-breathe, cough, and use an incentive spirometer.

A nurse provides information to a client diagnosed with peripheral vascular disease about ways to limit the disease's progression. Which measures does the nurse tell the client to take? Select all that apply. Crossing the legs at the ankles only Engaging in exercise such as walking on a daily basis Washing the feet daily with a mild soap and drying them well Inspecting the feet at least once a week for injuries, especially abrasions Using a heating pad on the legs to help keep the blood vessels dilated

Engaging in exercise such as walking on a daily basis Washing the feet daily with a mild soap and drying them well Rationale: Long-term management of peripheral vascular disease consists of measures that increase peripheral circulation. The client is instructed to exercise regularly and is encouraged to walk for 20 minutes each day. The client also needs to wash the feet daily with a mild soap, to dry the feet well, and to inspect the feet daily for injuries or abrasions. Crossing the legs at any level should be avoided because it promotes vasoconstriction. Keeping the extremities warm is important; however, heating pads and hot water bottles should not be placed on the extremity. Sensitivity may be diminished in the affected extremity, increasing the risk for burns. Also, direct application of heat increases the oxygen and nutritional requirements of the tissue even further.

An emergency department nurse is caring for a client in hypovolemic shock, a result of external hemorrhage caused by a gunshot wound. Which nursing interventions should the nurse take? Select all that apply. Maintaining the client in a high Fowler's position Checking the client's vital signs every hour until stable Ensuring that direct pressure is applied to the external hemorrhage site Ensuring a patent airway and supplying oxygen to the client as prescribed Inserting an intravenous (IV) catheter and administering fluids as prescribed Ensuring that the call bell is in place for the client's use when the nurse is out of the room

Ensuring that direct pressure is applied to the external hemorrhage site Ensuring a patent airway and supplying oxygen to the client as prescribed Inserting an intravenous (IV) catheter and administering fluids as prescribed Rationale: When caring for a client in hypovolemic shock, the nurse must first ensure a patent airway and supply oxygen to the client. Direct pressure must be applied to the site of external bleeding. The nurse would also insert an IV catheter if one is not already present and administer fluids as prescribed. The nurse would elevate the client's feet, keeping his or her head flat or elevated to a 30-degree angle. The nurse would take the client's vital signs every 5 minutes until they were stable. The nurse would not leave the client alone.

A nurse in a primary health care provider's office is reviewing the medical record of a child with a diagnosis of lactose intolerance. Which finding does the nurse expect to see documented in the child's record? Fatty stools Episodes of foul-smelling ribbon-like stools Episodes of profuse watery diarrhea and vomiting Episodes of cramping abdominal pain and excessive flatus

Episodes of cramping abdominal pain and excessive flatus Rationale: Manifestations of lactose intolerance include diarrhea that is frothy (but not fatty), abdominal distention, cramping abdominal pain, and excessive flatus. The presence of fatty stools may indicate a problem with bile flow. Foul-smelling ribbon-like stool is a clinical manifestation of Hirschsprung disease. Profuse watery diarrhea and vomiting is one clinical manifestation of celiac disease.

A client who has sustained an acute myocardial infarction (AMI) is receiving intravenous reteplase. For which adverse effect of the medication does the nurse monitor the client? Diarrhea Vomiting Epistaxis Epigastric pain

Epistaxis Rationale: Reteplase is a thrombolytic medication that promotes the fibrinolytic mechanism (i.e., conversion of plasminogen to plasmin, which destroys the fibrin in the blood clot). The thrombus, or blood clot, disintegrates when a thrombolytic medication is administered within 4 hours of an AMI. Necrosis resulting from blockage of the artery is prevented or minimized, and hospitalization may be shortened. Bleeding, a major adverse effect of thrombolytic therapy, may be superficial or internal and may be spontaneous. Epigastric pain, vomiting, and diarrhea are not adverse effects of this therapy.

A nurse is assigned to conduct an admission assessment of a client with a diagnosis of bipolar disorder. What does the nurse plan to do first? Perform the physical assessment Tell the client about the nursing unit rules Establish a trusting nurse-client relationship Tell the client that he/she will have to participate in self-care

Establish a trusting nurse-client relationship Rationale: Bipolar disorder, also known as manic-depressive illness, is a brain disorder that causes unusual shifts in mood, energy, activity levels, and the ability to carry out day-to-day tasks. The nurse should first develop a trusting relationship with the client. It is most important to establish a trusting relationship, which will indicate to the client that the client is important. After a therapeutic relationship has been established, other interventions may be carried out. The nurse would perform a physical assessment, but this would not be the first intervention. The client should be informed of the nursing unit's rules, but, again, this is not the first intervention. Telling the client that he/she will have to participate in self-care is inappropriate. The client with bipolar disorder requiring hospitalization is likely to need assistance with care.

Alendronate is prescribed for a client with postmenopausal osteoporosis. The nurse provides information on the medication to the client. When does the nurse tell the client to take the alendronate? At bedtime With orange juice, to enhance absorption at night Every morning before breakfast, with a full glass of water Every morning after breakfast, after which the client should lie down for 30 minutes

Every morning before breakfast, with a full glass of water Rationale: Alendronate should be taken in the morning before breakfast with a full glass of water and on an empty stomach to maximize its bioavailability. It is a medication used to treat postmenopausal osteoporesis, glucocorticoid-induced osteoporosis, and Paget's disease of bone. Proper administration is necessary to maximize bioavailability and minimize the risk of esophagitis. No food, including orange juice or coffee, should be consumed for at least 30 minutes after alendronate is taken. To minimize the risk of esophagitis, the client should take the medication with a full glass of water and remain upright (seated or standing) for at least 30 minutes. Therefore taking the medication at bedtime, with orange juice to help with absorption, and every morning after breakfast, followed by a 30-minute period of lying down, are all incorrect.

A nurse is gathering subjective and objective data from a client with suspected rheumatoid arthritis (RA). Which early manifestations of RA would the nurse expect to note? Select all that apply. Fatigue Anemia Weight loss Low-grade fever Joint deformities

Fatigue Low-grade fever Rationale: Early manifestations of RA include fatigue, low-grade fever, weakness, anorexia, and paresthesias. Rheumatoid arthritis is a chronic, progressive, systemic and inflammatory autoimmune disease process that affects the synovial joints, resulting in their destruction. Anemia, weight loss, and joint deformities are some of the late manifestations.

A client says to the nurse, "My doctor just left. He told me that my abdominal scan showed a mass in my pancreas and that it's probably cancer. Does this mean I'm going to die?" How does the nurse interpret the client's initial reaction? Fear Denial Acceptance Anger

Fear Rationale: The nurse interprets the client's initial reaction as fear. Fear is a response to a threat that is consciously recognized as a danger. In this situation, the client's reaction is one of fear, and the client verbalizes the object of fear (dying). There is no evidence of denial, acceptance, or anger in the client's statement.

A client is admitted to the nursing unit with a diagnosis of avoidant personality disorder. What are some characteristics of this disorder? Select all that apply. Neediness Perfectionism Feelings of inadequacy Feeling extremely shy Sensitivity to rejection Preoccupation with details Hypersensitivity to negative evaluation

Feelings of inadequacy Feeling extremely shy Sensitivity to rejection Hypersensitivity to negative evaluation Rationale: Avoidant personality disorder is a psychiatric condition in which a person feels extremely shy, inadequate, and sensitive to rejection. Other characteristics of avoidant personality disorder include excessive anxiety in social situations and hypersensitivity to negative evaluation. Neediness is a characteristic of dependent personality disorder. Perfectionism and preoccupation with details are characteristics of obsessive-compulsive disorder.

A nurse is caring for a client with community-acquired pneumonia who is being treated with levofloxacin. Which finding, indicating an adverse reaction to the medication, does the nurse monitor the client? Fever Dizziness Flatulence Drowsiness

Fever Rationale: Levofloxacin is an antibiotic of the fluoroquinolone class. Pseudomembranous colitis is an adverse reaction associated with the use of this medication. It is characterized by severe abdominal pain or cramps, severe watery diarrhea, and fever. Dizziness, flatulence, and drowsiness are side effects of the medication.

A nurse is reviewing the medical record of a client with a suspected systemic lupus erythematosus (SLE). Which manifestations of SLE would the nurse expect to find noted in the client's medical record? Select all that apply. Fever Vasculitis Weight gain Increased energy Abdominal pain

Fever Vasculitis Abdominal pain Rationale: Systemic lupus erythematosus is a chronic, progressive, inflammatory disorder of the connective tissue that can cause the failure of major organs and body systems. Manifestations include fever, fatigue, anorexia, weight loss, vasculitis, discoid lesions, and abdominal pain. Erythema, usually in a butterfly pattern (hence the nickname "butterfly rash"), appears over the cheeks and bridge of the nose. Other manifestations include nephritis, pericarditis, the Raynaud phenomenon (discoloration of fingers and/or toes after exposure to changes in temperature), pleural effusions, joint inflammation, and myositis.

A nurse reviews the medical record of a client with histoplasmosis. Which clinical manifestation of this infection does the nurse expect to see documented? Neurological deficits Cardiac dysrhythmias Gastrointestinal disturbances Flulike pulmonary signs/symptoms

Flulike pulmonary signs/symptoms Rationale: Histoplasmosis is a fungal infection of the lungs. The client typically experiences a flulike pulmonary illness with cough, chest pain, dyspnea, headache, fever, arthralgia, anorexia, erythema nodosum, hepatomegaly, and splenomegaly. Neurological disturbances, gastrointestinal disturbances, and cardiac dysrhythmias are not associated with this infection.

A client calls the emergency department and tells the nurse that he may have come in contact with poison ivy while trimming bushes in his yard. What does the nurse tell the client to immediately do? Contact the primary health care provider Report to the emergency department for treatment Get into the shower and rinse the skin for at least 15 minutes Go to the drugstore, purchase an over-the-counter topical corticosteroid, and rub it into the exposed skin

Get into the shower and rinse the skin for at least 15 minutes Rationale: If contact with poison ivy is suspected, signs/symptoms may be averted by immediately rinsing the skin for 15 minutes with running water to remove the resin before skin penetration occurs. Persons walking or working in areas where poison ivy grows should protect the skin by wearing appropriate clothing. The client is also instructed to remove clothing carefully to avoid skin contact. Although a topical over-the-counter corticosteroid may relieve some of the discomfort of the poison ivy rash, this is not the action that needs to be taken immediately. Contacting the primary health care provider and coming to the emergency department for treatment are unnecessary.

A client with agoraphobia will undergo systematic desensitization through graduated exposure. In explaining the treatment to the client, what does the nurse tell the client this technique involves? Having the client perform a healthy coping behavior Having the client perform a ritualistic or compulsive behavior Providing a high degree of exposure of the client to the stimulus that the client finds undesirable Gradually introducing the client to a phobic object or situation in a predetermined sequence of least to most frightening

Gradually introducing the client to a phobic object or situation in a predetermined sequence of least to most frightening Rationale: The technique of systematic desensitization involves gradually introducing the client to a phobic object or situation in a predetermined sequence of least to most frightening with the goal of defusing the phobia. Having the client perform a healthy coping behavior is the description of modeling. Performing ritualistic or compulsive behaviors is a behavior characteristic of clients with obsessive-compulsive disorder. Having the client perform a ritualistic or compulsive behavior may not be therapeutic; additionally, it is not associated with systematic desensitization. Providing a high degree of exposure to a stimulus that the client finds undesirable is the technique known as flooding.

A nurse is providing information to a client with diabetes insipidus who will be taking desmopressin acetate by way of the nasal route. For which occurrence does the nurse tell the client to contact the primary health care provider? Abdominal cramps Stuffy or runny nose Headache and nausea Decreased urine output

Headache and nausea Rationale: One adverse effect of desmopressin acetate is water intoxication. Early signs/symptoms of water intoxication include headache, nausea, shortness of breath, drowsiness, and listlessness. The primary health care provider is notified if these signs/symptoms occur. Desmopressin, a synthetic form of antidiuretic hormone, causes increased resorption of water and a resultant decrease in urine output (an expected outcome). Diabetes insipidus is a disorder of water metabolism caused by a deficiency of antidiuretic hormone (ADH). Abdominal cramping is a side effect, not an adverse effect, of the parenteral form of the medication. A runny or stuffy nose is a side effect, not an adverse effect, of the medication.

A client with cervical cancer is undergoing chemotherapy with cisplatin. For which adverse effect of cisplatin will the nurse assess the client? Nausea Bloody urine Hearing loss Electrocardiographic changes

Hearing loss Rationale: Cisplatin is a platinum-based agent used to treat various types of cancer. One adverse effect of cisplatin is ototoxicity, and the nurse would monitor the client for tinnitus and hearing loss. Nausea occurs with the use of several chemotherapeutic agents and is not necessarily an adverse effect. Cyclophosphamide causes hemorrhagic cystitis, evidenced by bloody urine. Doxorubicin (Adriamycin) causes cardiotoxicity.

A nurse is providing information on the glycosylated hemoglobin assay and its purpose to a client with diabetes mellitus. What does the nurse tell the client about this blood test? Is a measure of the client's hematocrit level Is a measure of the client's hemoglobin level Helps predict the risk for the development of chronic complications of diabetes mellitus Provides a determination of short-term glycemic control in the client with diabetes mellitus

Helps predict the risk for the development of chronic complications of diabetes mellitus Rationale: The nurse tells the client that the blood test is used to assess long-term glycemic control, as well as to predict the risk for the development of chronic complications. Glycosylated hemoglobin is the best indicator of the average blood glucose level. Because glucose attaches itself to the hemoglobin molecule, measurement of glycosylated hemoglobin indicates the average blood glucose level during the previous 120 days, the lifespan of the red blood cell.

The client who is scheduled to undergo chemotherapy asks the nurse, "Is my hair going to fall out?" What does the nurse tell the client? Her hair will definitely fall out She should not be worrying about her hair at this point Her hair may fall out but will regrow after the chemotherapy is discontinued Vigorous hair-brushing is important while the client is undergoing chemotherapy to prevent hair loss

Her hair may fall out but will regrow after the chemotherapy is discontinued Rationale: Some chemotherapeutic agents cause hair loss, and the client should be informed of this possibility. The client should also be reassured that chemotherapy-related hair loss is temporary and that the hair will regrow after the chemotherapy is discontinued, perhaps in a different shade, color, or texture. Telling the client that her hair will definitely fall out is incorrect and telling her that she should not be worrying about her hair at this point ignores the client's concern. Hair dyes, permanents, and vigorous hair-brushing are avoided to minimize thinning.

A client diagnosed with advanced chronic kidney disease (CKD) and oliguria has been taught about sodium and potassium restriction between dialysis treatments. The nurse determines that the client understands this restriction if the client states that what is acceptable to use? Salt substitutes Herbs and spices Salt with cooking only Processed foods as desired

Herbs and spices Rationale: Most clients with CKD retain sodium. The client with CKD is instructed not to add salt at the table or during food preparation. Herbs and spices may be used as an alternative to salt to enhance the flavor of food. The client with advanced CKD is instructed to limit potassium intake. The client is also instructed to avoid salt substitutes, many of which are composed of potassium chloride, if oliguria is present. Processed foods are discouraged because they are high in sodium.

A nurse has provided information about exercise to a client with a diagnosis of degenerative joint disease (osteoarthritis). Which type of exercise does the nurse tell the client to avoid? High-impact exercise Swimming and water exercise Daily range-of-motion exercises Regular exercise with warm-up and cool-down sessions

High-impact exercise Rationale: Vigorous or high-impact exercise may exacerbate signs/symptoms and may be damaging to articulating surfaces within joints. These type of exercises should be avoided by clients with osteoarthritis. Osteoarthritis is a joint disorder marked by joint pain that is worse after high-impact exercise or when the affected person puts weight on the joint and is relieved by rest. The other options are helpful activities/exercises in promoting joint mobility.

A nurse, providing information to a client who has just been diagnosed with diabetes mellitus, gives the client a list of signs/symptoms of hypoglycemia. Which answers by the client, on being asked to list the signs/symptoms, tells the nurse that the client understands the information? Select all that apply. Hunger Weakness Blurred vision Increased thirst Increased urine output

Hunger Weakness Blurred vision Rationale: Signs/symptoms of hypoglycemia include weakness, double vision, blurred vision, hunger, tachycardia, and palpitations. The manifestations of diabetes mellitus (hyperglycemia) include polydipsia, polyuria, and polyphagia.

A client with chronic back pain asks a nurse about the use of complementary and alternative therapies to treat the pain. What would the nurse initially do? Identify the client's treatment goals Share current research outcomes with the client Offer options that may be beneficial to the client Tell the client that the primary health care provider does not believe in these therapies

Identify the client's treatment goals. Rationale: If a client asks a nurse about complementary and alternative therapies, the nurse should initially identify the client's treatment goals, such as sign/symptom management. The nurse should respect the client's request and inquiry about these therapies. Offering options that may be beneficial to the client are appropriate but would not be the initial nursing action. Telling the client that the primary health care provider does not believe in these therapies is a violation of the client's right to be informed about treatment options.

A client with myasthenia gravis is taking neostigmine bromide. What does the nurse note that indicates the client is gaining a therapeutic effect from the medication? Bradycardia Increased heart rate Decreased blood pressure Improved swallowing function

Improved swallowing function Rationale: Neostigmine bromide, a cholinergic medication that prevents the destruction of acetylcholine, is used to treat myanthenia gravis. The nurse would monitor the client for a therapeutic response, which includes increased muscle strength, an easing of fatigue, and improved chewing and swallowing function. Bradycardia, increased heart rate, and decreased blood pressure are signs/symptoms of an adverse reaction to the medication.

A nurse is planning to teach a crutch gait to a client who will be using wooden axillary crutches. The nurse knows that what elements are related to the basic crutch stance? Select all that apply. Improves client's balance Hips and knees are extended Provides narrow base of support Axillae bear half of client's weight Incorrect Tripod position assumed before crutch walking Body alignment includes erect head and neck and straight vertebrae

Improves client's balance Hips and knees are extended Tripod position assumed before crutch walking Body alignment includes erect head and neck and straight vertebrae Rationale: The basic crutch stance is the tripod position, which the client assumes before crutch walking. It is formed when the crutches are placed 15 cm (6 inches) in front of and 15 cm (6 inches) to the side of each foot. This position improves the client's balance by providing a wider, not narrower, base of support. The body alignment of the client in the tripod position includes an erect head and neck, straight vertebrae, and extended hips and knees. The axillae should not bear any weight.

A home care nurse prefills syringes containing NPH (Humulin N) and regular (Humulin R) insulin for a client with diabetes mellitus who will be administering his/her own insulin but has difficulty seeing and accurately preparing doses. How does the nurse place the medication in the client's refrigerator with the syringes? Lying flat In a horizontal position In a vertical position with the needles pointing up In a vertical position with the needles pointing down

In a vertical position with the needles pointing up Rationale: The syringes should be stored vertically, with the needles pointing up to prevent clogging of the needle with the insulin. Mixtures of insulin in prefilled syringes may be stored in a refrigerator, where they will be stable for at least 1 to 2 weeks. Before administration of the medication, the syringe should be agitated gently to resuspend the insulin.

A client who has undergone abdominal hysterectomy asks the nurse when she will be able to resume sexual intercourse. What does the nurse tell the client about when sexual intercourse can be resumed? At any time after the surgery When menstruation resumes When pelvic sensation and response to stimuli return In about 6 weeks, when the vaginal vault is satisfactorily healed

In about 6 weeks, when the vaginal vault is satisfactorily healed Rationale: After abdominal hysterectomy, the client is instructed to avoid sexual intercourse until the vaginal vault is satisfactorily healed. This takes about 6 weeks. A woman who has undergone this procedure must adjust to changes in the nature of pelvic sensations and stimuli during sexual intercourse; however, this is not related to when sexual intercourse may be resumed. The client will not have menstrual periods after abdominal hysterectomy.

Oral prednisone 10 mg/day is prescribed for a client with an acute exacerbation of rheumatoid arthritis. The nurse is providing information to the client about the medication. What time of day does the nurse tell the client is best to take the medication? At bedtime With lunch In the evening, after 9 p.m. In the morning, before 9:00 a.m.

In the morning, before 9:00 a.m. Rationale: The nurse tells the client that it is best to take glucocorticoids before 9 a.m. This schedule helps minimize adrenal insufficiency and mimics the burst of glucocorticoids released naturally by the adrenals each morning. Prednisone, a glucocorticoid, may be prescribed to reduce inflammation and pain in acute exacerbations of rheumatoid arthritis. The time frames in the remaining options are incorrect.

A nurse provides instructions to a client who will be taking levothyroxine for hypothyroidism. When does the nurse tell the client that it is best to take the medication? With milk At bedtime With an antacid In the morning, before breakfast

In the morning, before breakfast Rationale: The nurse tells the client that the best way and time to take the medication is on an empty stomach and preferably in the morning, before breakfast. Levothyroxine, a synthetic preparation of thyroxine, a naturally occurring thyroid hormone, is used to treat clients who require hormone-replacement therapy.

A laxative has been prescribed for a client with diminished colonic motor response as a means of promoting defecation. The nurse provides information to the client about the medication. What does the nurse tell the client to do? Increase fluid intake Consume low-fiber foods Consume foods that are low in potassium Contact the primary health care provider if the urine turns yellow-brown

Increase fluid intake Rationale: The nurse encourages the client to increase fluid intake, to consume a high-fiber diet, and to exercise. Hypokalemia may result from use of a laxative, so the nurse encourages the client to consume foods high in potassium. The client's urine may turn pink-red, red-violet, red-brown, or yellow-brown, but the client is told that this is a temporary, harmless effect.

A client diagnosed with hypoparathyroidism is taking calcium gluconate to treat hypocalcemia. The client calls the clinic nurse and complains of becoming constipated since starting the medication. What should the nurse tell the client to do? Stop the medication Contact the primary health care provider immediately Increase intake of high-fiber foods Add a half-ounce (15 ml) of mineral oil to the daily diet

Increase intake of high-fiber foods Rationale: The nurse should tell the client to increase intake of high-fiber foods. Calcium gluconate is a calcium supplement, and constipation is a side effect of calcium gluconate. It is not necessary for the client to contact the primary health care provider immediately if constipation occurs. Rather, the nurse would suggest interventions to alleviate and prevent constipation (e.g., increased fluid intake, high-fiber diet, exercise). Clients taking calcium supplements should be instructed to avoid the use of mineral oil as a laxative because it reduces vitamin D absorption, and vitamin D is needed to assist in the absorption of calcium. The client should not stop taking the medication.

A nurse is monitoring a child with intussusception for signs/symptoms of peritonitis. For which finding, indicative of this complication, does the nurse notify the primary health care provider? Increased alertness Increased heart rate A sausage-shaped abdominal mass Diarrhea and the passage of bloody mucous stool

Increased heart rate Rationale: The nurse monitors the child closely for signs/symptoms of sepsis, peritonitis, and shock. Possible indicators of peritonitis include fever, increased heart rate, changes in the level of consciousness or in blood pressure, and respiratory distress. Intussusception is an invagination of a section of the intestine into the distal bowel. It is the most common cause of bowel obstruction in children ages 3 months to 6 years. The classic signs of intussusception are a sausage-shaped abdominal mass and the passage of bloody ("currant jelly") stool and diarrhea.

Laboratory studies are performed on a client diagnosed with suspected sickle cell disease, and electrophoresis reveals a large percentage of hemoglobin S (HbS). Which additional laboratory finding will the nurse expect to note that is a characteristic of this disease? Low reticulocyte count Low total bilirubin level Increased hematocrit count Increased white blood cell (WBC) count

Increased white blood cell (WBC) count Rationale: Sickle cell disease is a genetic disorder that results in chronic anemia, pain, disability, organ damage, increased risk for infection, and early death. The WBC count is usually higher than normal in clients with sickle cell disease. Also, the red blood cells assume a sickle shape, become rigid, and clump together. The main laboratory finding associated with sickle cell disease is the large percentage of HbS present on electrophoresis. The reticulocyte count is increased, indicating anemia of long duration, and the total bilirubin level is increased. The hematocrit level is low and decreases during crisis, because the bone marrow fails to produce cells during stressful periods. It is believed that this increase is related to chronic inflammation resulting from tissue hypoxia and ischemia.

A client diagnosed with HIV infection who has been found to have histoplasmosis is being treated with intravenous amphotericin B. Which parameter does the nurse check to detect the most common adverse effect of this medication? Temperature Blood pressure Peripheral pulses Intake and output

Intake and output Rationale: As a means of detecting renal injury, tests of kidney function should be performed weekly, and intake and output should be monitored closely. Amphotericin B, an antifungal medication, is highly toxic, and infusion reactions and renal damage occur, to varying degrees, in all clients. Other adverse effects include delirium, hypotension, hypertension, wheezing, and hypoxia. The remaining options are not associated with an adverse effect of the medication.

Desmopressin is prescribed to a client with diabetes insipidus. Which parameter does the nurse tell the client that it is important to monitor while taking the medication? Appetite Pulse rate Bowel pattern Intake and output

Intake and output Rationale: The client is instructed to monitor and record daily intake and output of fluid. Diabetes insipidus is a disorder of water metabolism caused by a deficiency of antidiuretic hormone. Desmopressin promotes renal conservation of water. The hormone accomplishes this by acting on the collecting ducts of the kidney to increase their permeability to water, resulting in increased water reabsorption. If the dose prescribed is adequate, urine volume should rapidly drop to normal. Appetite, pulse rate, and changes in bowel pattern are not associated with the use of this medication.

A nurse is preparing to provide information to a client who has been found to have stable angina. What does the nurse plan to tell the client about this type of angina? Requires surgical treatment Can be cured with medication Will eventually need to be treated with a coronary artery bypass graft Is often managed medically with medications such as calcium channel blockers and beta-blocking medications

Is often managed medically with medications such as calcium channel blockers and beta-blocking medications Rationale: The nurse tells the client that stable angina is often managed by medical means, such as calcium channel blockers and beta-blocking medications. Stable angina is chest discomfort that occurs with moderate to prolonged exertion in a pattern that is familiar to the client. It is usually relieved by nitroglycerin and rest. Rarely does it require aggressive treatment such as surgery. The disease can be managed but not cured.

A community health nurse is preparing a poster for a health fair that will include information about the ways to prevent ear infection or ear trauma. Which prevention measures does the nurse include on the poster? Select all that apply. Always sneeze with the mouth closed. Occlude one nostril when blowing the nose. Keep the volume of headphones at the lowest setting. Avoid environmental conditions involving rapid changes in air pressure. Clean the external ear and canal daily in the shower or while washing the hair. Be cautious when using cotton-tipped applicators to clean the external ear canal.

Keep the volume of headphones at the lowest setting. Avoid environmental conditions involving rapid changes in air pressure. Clean the external ear and canal daily in the shower or while washing the hair. Rationale: The client is instructed to wash the external ear and canal daily in the shower or while washing his or her hair. The client should also wear sound protection around loud or continuous noises, avoid activities with a high risk for ear trauma, keep the volume of headphones at the lowest setting, and avoid environmental conditions involving rapid changes in air pressure. The client should never use a small object such as a cotton-tipped applicator to clean the external ear canal. The client should always sneeze with the mouth open (not closed), and also be instructed to blow the nose gently and not occlude a nostril when blowing the nose.

Aneurysm precautions are prescribed for a client with a cerebral aneurysm. Which interventions does the nurse implement? Select all that apply. Keeping the room slightly darkened Placing the client in a room with a quiet roommate Encouraging isometric exercises if bed rest is prescribed Monitoring the client for changes in alertness or mental status Restricting visits to close family members and significant others and keeping visits short

Keeping the room slightly darkened Monitoring the client for changes in alertness or mental status Restricting visits to close family members and significant others and keeping visits short Rationale: The room is kept slightly darkened, and bright lighting is avoided. The client is placed in a quiet private room without a telephone. The client is monitored for changes in alertness or mental status. Visitors are restricted to close family members and significant others, and visits are kept short. Any contact with visitors who upset or excite the client is avoided. A cerebral aneurysm is a thin-walled outpouching or dilation of an artery of the brain. When an aneurysm ruptures, bleeding into the subarachnoid space usually ensues. Aneurysm precautions are implemented to maintain a stable perfusion pressure and help prevent rupture. Stool softeners are administered to help keep the client from straining during defecation. Isometric exercises and use of the Valsalva maneuver are avoided because both increase intrathoracic and intraabdominal pressure. Bed rest with the head of the bed elevated 30 degrees may be prescribed. Some primary health care providers permit bathroom privileges for selected clients. If the client is allowed out of bed, the nurse stresses the importance of not bending over.

A nurse is caring for a client who is immobilized in skeletal traction after sustaining a leg fracture in a motor vehicle crash. The nurse notes that the client is restless, and the client complains of being bored. Which problem does the nurse identify on the basis of this information? Lack of control Lack of physical mobility Lack of adequate diversional activity Lack of energy to bathe and feed self

Lack of adequate diversional activity Rationale: A characteristic of lack of adequate diversional activity is the expression of boredom by the client. The question does not identify client difficulties with coordination, range of motion, or muscle strength, which would be related to lack of physical mobility. Nor does the question address client's lack of energy to perform activities of daily living (bathing/hygiene self-care deficit) or lack of control.

A pediatric nurse is caring for a hospitalized toddler. Which activity does the nurse deem the most appropriate for the toddler? Singing games Watching videos Simple board games Large building blocks

Large building blocks Rationale: The toddler is developing the use of motor skills and enjoys manipulating small objects such as blocks, push-pull toys, and toy people, cars, and animals. Therefore large building blocks are appropriate for a client of this age. Singing games, simple board games, and videos are appropriate for the preschooler.

A nurse is monitoring a client who was brought to the emergency department in an unresponsive state and is now being treated for hyperglycemic nonketotic syndrome (HNS). Which finding indicates to the nurse that fluid replacement is inadequate? Increased urine output Blood pressure of 128/80 mm Hg Potassium level of 3.6 mEq/L (3.6 mmol/L) Level of consciousness remains unchanged

Level of consciousness remains unchanged Rationale: Lack of any improvement in the level of consciousness may indicate an inadequate rate of fluid replacement or a reduction in plasma osmolarity. The initial objective for fluid replacement in HNS is to increase circulating blood volume. Increased urine output indicates an improvement in the client's status. Potassium level of 3.6 mEq/L (3.6 mmol/L) and a blood pressure of 128/80 mm Hg are normal findings.

A client who has undergone knee-replacement surgery will be self-administering enoxaparin sodium at home. The nurse teaches the client about the medication. What does the nurse tell the client? Store the medication in the refrigerator Lie down to administer the subcutaneous injection Inject the medication in the upper outer aspect of the arm Discard the medication if the solution appears pale yellow

Lie down to administer the subcutaneous injection Rationale: The client is instructed to lie down to administer the injection and to introduce the entire length of the needle (½ inch [1.25 cm]) into a skin fold held between the thumb and forefinger. Enoxaparin sodium is an anticoagulant that is administered by way of subcutaneous injection. It is injected into the abdominal wall. The solution, which appears clear and colorless to pale yellow, is stored at room temperature.

Alprazolam is prescribed for a client to treat an anxiety disorder. Which side effect does the nurse warn the client of? Headache Urine retention Lightheadedness Increased salivation

Lightheadedness Rationale: Alprazolam is a benzodiazepine that is used to manage anxiety disorders. Common side effects include muscle incoordination, lightheadedness, transient mild drowsiness, and slurred speech. The other options are not side effects of this medication.

Although previously well controlled with glyburide, a client's fasting blood glucose has been running 180 to 200 mg/dL (10 to 11.1 mmol/L). On reviewing the client's record, which medication, recently added to the client's regimen, does the nurse recognize as a possible contributor to the hyperglycemia? Atenolol Phenelzine Allopurinol Lithium carbonate

Lithium carbonate Rationale: Glyburide is a hypoglycemic medication. If the client takes a corticosteroid, thiazide diuretic, or lithium carbonate concurrently, the effect of the glyburide is diminished. Lithium carbonate, an antimanic medication, may increase the blood glucose level. Phenelzine is a monoamine oxidase inhibitor. Atenolol is a beta-blocker. Allopurinol is a xanthine oxidase inhibitor. These medications may amplify the effects of an oral hypoglycemic.

The emergency room nurse is caring for a client who sustained extensive burn injuries. What are the nurse's primary goals for this client? Select all that apply. Administer oxygen Insert a Foley catheter Maintain a patent airway Contact the client's family Preserve vital organ functioning Administer IV fluids to prevent hypovolemic shock

Maintain a patent airway Preserve vital organ functioning Administer IV fluids to prevent hypovolemic shock Rationale: The primary goals for the client with a burn injury are to maintain a patent airway, administer intravenous (IV) fluids to prevent hypovolemic shock, and preserve vital organ functioning.

A nurse is caring for a client with a cuffed endotracheal tube who is undergoing mechanical ventilation. Which intervention to prevent a tracheoesophageal fistula, a complication of this type of tube, does the nurse implement? Frequent suctioning Maintaining cuff pressure Maintaining mechanical ventilation settings Alternating the use of a cuffed tube with a cuffless tube on a daily basis

Maintaining cuff pressure Rationale: Necrosis of the tracheal wall caused by pressure of the cuff of an endotracheal tube can lead to the development of an opening between the posterior trachea and esophagus, a complication known as tracheoesophageal fistula. The fistula allows air to escape into the stomach, resulting in abdominal distention. It also leads to the aspiration of gastric contents. To prevent this complication, the nurse must maintain cuff pressure, monitor the amount of air needed for cuff inflation, and help the client progress to a deflated cuff or cuffless tube as soon as possible as prescribed by the primary health care provider. Suctioning should be performed only as needed; frequent suctioning can cause mucosal damage. Maintenance of mechanical ventilation settings ensures that the client is adequately oxygenated, but this intervention is not a measure for the prevention of tracheoesophageal fistula. Alternating the use of a cuffed tube and a cuffless tube on a daily basis is incorrect, because the endotracheal tube would not be removed and replaced on a daily basis.

A client being seen in the clinic complains of fatigue and weakness. Laboratory studies are performed because the primary health care provider suspects iron-deficiency anemia. After reviewing the laboratory results, which finding indicative of this type of anemia does the nurse expect to note?. An increased RBC count An increased hematocrit level An increased hemoglobin level Microcytic red blood cells (RBCs)

Microcytic red blood cells (RBCs) Rationale: The nurse expects to note a low RBC count and microcytic (small) RBCs. In iron-deficiency anemia, laboratory testing will reveal low hemoglobin and hematocrit levels. In iron-deficiency anemia, iron stores are depleted first, followed by hemoglobin stores.

The nurse is reviewing the record of a client admitted with Cushing's syndrome due to ingestion of large doses of long-term glucocorticoids used to treat nonendocrine conditions. What signs/symptoms does the nurse expect to note? Select all that apply. Moon face Dizziness Osteoporosis Hyperkalemia Loss of weight Edema in the feet

Moon face Dizziness Osteoporosis Edema in the feet Rationale: Signs/symptoms of clients with Cushing's syndrome include moon face, dizziness, osteoporosis, and swelling of the feet. Other signs/symptoms include puffy eyelids, increased bruising, bleeding, and menstrual irregularities. Hyperkalemia, not hypokalemia, also occurs, as well as gaining, not losing, weight.

A nurse is assessing a client with hepatitis for signs/symptoms of jaundice. Which area does the nurse check, knowing that it will provide the best data regarding the presence of jaundice? Lips Soles Palms Mucous membranes

Mucous membranes Rationale: Assessment of the skin, sclera, and mucous membranes provides the best data regarding the presence of jaundice. The color of the lips provides data regarding the presence of cyanosis. Although assessment of the skin provides adequate data regarding jaundice, the soles and palms are not the best areas of skin for assessment. Jaundice needs to be distinguished from yellow or green skin color resulting from carotenemia or quinacrine (drug used for treatment of giardiasis), The resultant yellow skin color is differentiated from jaundice by the absence of yellow color in mucous membranes and sclerae, the normal urine color, and the accentuation of yellow-brown carotenoid pigment in the palms, soles, and nasolabial folds.

Which finding in a client's history indicates the greatest risk of cervical cancer to the nurse? Nulliparity Early menarche Multiple sexual partners Hormone-replacement therapy

Multiple sexual partners Rationale: Risk factors for cervical cancer include multiple sexual partners, a history of human papillomavirus infection, first sexual intercourse before the age of 16, cigarette smoking, environmental tobacco smoke exposure, and use of oral contraceptives for more than 5 years. Nulliparity, early menarche, and the use of hormone-replacement therapy are risk factors for ovarian rather than cervical cancer.

A nurse is caring for a client with open-angle glaucoma. The nurse knows that what agents are used to treat this condition? Myotic agents Mydriatric agents Cycloplegic agents Anticholinergic agents

Myotic agents Rationale: Miotic agents are used to treat glaucoma. Mydriatic medications dilate the pupil and can cause an increase in intraocular pressure in the eye. Besides mydriatic agents, cycloplegic and anticholinergic agents are contraindicated in clients with glaucoma.

An emergency department nurse is assessing a client with acute closed-angle glaucoma. Which characteristic of the disorder does the nurse expect the client to exhibit? Select all that apply. Nausea Eye pain Vomiting Headache Diminished central vision Increased light perception

Nausea Eye pain Vomiting Headache Rationale: In acute closed-angle glaucoma, the onset of signs/symptoms is acute and the client complains of sudden excruciating pain around the eyes that radiates over the sensory distribution of the fifth cranial nerve. Headache or brow ache, nausea, vomiting, and abdominal discomfort may also occur. Other signs/symptoms of glaucoma include seeing colored halos around lights, sudden blurred vision with decreased light perception, and loss of peripheral vision.

Phenelzine sulfate is prescribed for a client with depression. The nurse provides information to the client about the adverse effects of the medication and tells the client to contact the primary health care provider immediately if he/she experiences what sign/symptom? Dry mouth Restlessness Feelings of depression Neck stiffness or soreness

Neck stiffness or soreness Rationale: The client is taught to immediately contact the primary health care provider if the client experiences any occipital headache radiating frontally and neck stiffness or soreness, which could be the first sign of a hypertensive crisis. Phenelzine sulfate, a monoamine oxidase inhibitor (MAOI), is an antidepressant and is used to treat depression. Hypertensive crisis, an adverse effect of this medication, is characterized by hypertension, frontally radiating occipital headache, neck stiffness and soreness, nausea, vomiting, sweating, fever and chills, clammy skin, dilated pupils, and palpitations. Tachycardia, bradycardia, and constricting chest pain may also be present. Dry mouth and restlessness are common side effects of the medication.

An emergency department (ED) nurse is monitoring a client who sustained a severe inhalation burn injury during a fire in which the client was trapped in an enclosed space. The nurse auscultates the client's trachea and notes that the previously heard wheezing sounds have disappeared. What is the most appropriate action the nurse should take? Continue monitoring the client Notify the emergency department (ED) primary health care provider Document the client's improvement in the medical record Remove the oxygen mask and fit the client with a nasal

Notify the emergency department (ED) primary health care provider Rationale: The most appropriate action by the nurse would be to notify the ED primary health care provider immediately. A client with a severe inhalation injury may sustain such progressive obstruction that within a short time he/she cannot force air through the narrowed airways. As a result, the wheezing sounds disappear. This finding indicates impending airway obstruction and demands immediate intubation. A client with an inhalation burn injury is at risk for respiratory complications. Upper-airway edema and inhalation injury are most notable in the trachea and main stem bronchi. Auscultation of these areas reveals wheezes, which are a sign/symptom of obstruction. Continuing to monitor the client, documenting the client's improvement in the medical record, and removing the oxygen mask and fitting the client with a nasal cannula are all incorrect and would delay necessary interventions.

The night nurse is caring for a client who just had a craniotomy. The nurse is monitoring the client's Jackson-Pratt drain that is being maintained on suction. The nurse notes that a total of 200 mL of red drainage has drained from the Jackson-Pratt (J-P) tube in the last 8 hours. What action should the nurse take? Document the amount in the client's record. Discontinue the Jackson-Pratt drain from suction. Continue to monitor the amount and color of the drainage. Notify the primary health care provider immediately of the amount of drainage.

Notify the primary health care provider immediately of the amount of drainage. Rationale: The nurse must immediately notify the primary health care provider of this excessive amount of drainage. The primary health care provider must also be immediately notified of any saturated head dressings. The normal amount of drainage from a Jackson-Pratt drain is 30 to 50 mL per shift. Discontinuing the suction from the J-P drain is not an option and is not done. Also, just documenting the amount in the client's record is not correct even though the nurse would document that the primary health care provider was notified of the total drain amount. Just continuing to monitor the amount of drainage is also not an option.

A nurse is caring for a client undergoing skeletal traction of the left leg. The client complains of severe pain in the leg. The nurse checks the client's alignment in bed and notes that proper alignment is being maintained. Which action should the nurse take next? Providing pin care Medicating the client Notifying the primary health care provider Removing some weight from the traction

Notifying the primary health care provider Rationale: The nurse realigns the client and, if this is ineffective in relieving the pain, should next notify the primary health care provider. A client in traction who complains of severe pain may require realignment or may have traction weights that are too heavy. Severe leg pain, once traction has been established, indicates a problem. Provision of pin care is not related to the problem as described. The client should be medicated after an attempt has been made to determine and treat the cause; the cause of the severe pain should be investigated first. The nurse should never remove the weights from the traction without a specific prescription to do so.

Ergotamine is prescribed to a client with cluster headaches. Which occurrence does the nurse tell the client to report to the primary health care provider if she experiences them while taking the medication? Cough Fatigue and lethargy Dizziness and fatigue Numbness and tingling of the fingers or toes

Numbness and tingling of the fingers or toes Rationale: Ergotamine is an antimigraine medication. Prolonged administration or an excessive dosage may produce ergotamine poisoning (ergotism). Signs/symptoms include nausea, vomiting, weakness in the legs, pain in the limb muscles, and numbness and tingling of the fingers and toes. The client is instructed to report these signs/symptoms to the primary health care provider if they occur. Cough, fatigue, lethargy, and dizziness are side effects and not adverse effects of the medication.

A female client admitted to the mental health unit tells the nurse that she cannot leave the house without checking to be sure that she has shut off the coffee maker and unplugged her curling iron. The client states that she even leaves the house, gets into her car, and then has to go back into the house to check these appliances again and that these behaviors are interfering with her work and social commitments. Which anxiety disorder does the nurse associate this client's symptoms? Agoraphobia Avoidant personality disorder Obsessive-compulsive disorder Dependent personality disorder

Obsessive-compulsive disorder Rationale: Obsessive-compulsive disorder is an anxiety disorder characterized by intrusive thoughts that produce uneasiness, apprehension, fear, or worry; by repetitive behaviors aimed at reducing anxiety; or by a combination of such thoughts (obsessions) and behaviors (compulsions). The client is inflexible and rigid, and is highly critical of self and others. The characteristics of dependent personality disorder include neediness and self-sacrificing and submissive behaviors. The client with avoidant personality disorder is extremely shy, feels inadequate, and is sensitive to rejection. Agoraphobia is the fear of open spaces.

A client diagnosed with depression is anorexic. Which measure does the nurse take to assist the client in meeting nutritional needs? Providing food and fluid as the client requests Offering high-calorie and high-protein foods and fluids frequently throughout the day Completing the dietary menu for the client to ensure that adequate nutrition is provided Weighing the client daily so that the client may determine whether the nutritional plan is working

Offering high-calorie and high-protein foods and fluids frequently throughout the day Rationale: The client should be offered high-calorie and high-protein foods and fluids frequently throughout the day. Small, frequent snacks are more easily tolerated than large plates of food when the client is anorexic. The client should be offered choices of foods and fluids he/she likes, because the client is more likely to consume foods he/she has selected. The client should be weighed weekly, not daily. Weight gain may not be noted daily, which may cause the client to view the interventions to improve nutritional status as useless.

A client is found to have iron-deficiency anemia, and ferrous sulfate is prescribed. What does the nurse tell the client is best to take the medication with? Milk Apple juice Orange juice Scrambled eggs

Orange juice Rationale: Ferrous sulfate is an iron preparation, and the client is instructed to take the medication with orange juice or another vitamin C-containing product to increase absorption of the iron. Milk and eggs inhibit the absorption of iron. Orange juice is higher in vitamin C than apple juice.

A nurse in the cardiac care unit (CCU) is told that a client with a diagnosis of myocardial infarction (MI) will be admitted from the emergency department (ED). Which item does the nurse give priority to placing at the client's bedside? Bedside commode Suctioning equipment Electrocardiography machine Oxygen cannula and flowmeter

Oxygen cannula and flowmeter Rationale: The oxygen cannula and flowmeter are the priority. The client will require oxygen therapy after myocardial infarction to improve oxygen supply to the myocardium and ease the pain resulting from ischemia. Suctioning equipment is not the priority item but may be needed if a complication occurs. An electrocardiogram machine and bedside commode may be necessary but are not the priority items.

A child with a diagnosis of Wilms' tumor is being admitted to the pediatric unit. The nurse prepares the room for the child and places a sign at the child's bedside. What does this sign tell staff to avoid? Palpating the abdomen Taking temperatures rectally Turning the child to the right side Measuring blood pressure in the right arm

Palpating the abdomen Rationale: The nurse would place a sign at the child's bedside warning against palpation of the child's abdomen. Wilms' tumor, or nephroblastoma, is the most common renal tumor in children. Arising from the renal parenchyma of the kidney, this tumor grows very rapidly. It may be unilateral and localized or bilateral and sometimes involves metastasis to other organs. The tumor mass should not be palpated because of the risk that the protective capsule will rupture. Excessive manipulation can result in seeding of the tumor and the spread of cancerous cells. Taking temperatures rectally, turning the child to the right side, and measuring blood pressure in the right arm are interventions that do not need to be avoided.

A client is found to have hypoxemic respiratory failure. Which finding does the nurse expect to note on review of the results of the client's arterial blood gas (ABG) analysis? Pao2 of 73 mm Hg, Paco2 of 62 mm Hg Pao2 of 58 mm Hg, Paco2 of 35 mm Hg Pao2 of 60 mm Hg, Paco2 of 45 mm Hg Pao2 of 49 mm Hg, Paco2 of 32 mm Hg

Pao2 of 49 mm Hg, Paco2 of 32 mm Hg Rationale: Hypoxemic respiratory failure is characterized by a low Pao2 (less than 55 mm Hg) and a normal or low Paco2. The normal Pao2 is 80 to 100 mm Hg and the normal Paco2 is 35 to 45 mm Hg. Respiratory failure may be classified, according to the underlying pathophysiology, as hypoxemic respiratory failure or hypoxemic-hypercapnic respiratory failure. Hypoxemic-hypercapnic respiratory failure is characterized by a low Pao2 (less than 55 mm Hg) and an increased Paco2 (greater than 50 mm Hg). A Pao2 of 49 mm Hg and Paco2 of 32 mm Hg is the only option that characterizes hypoxemic respiratory failure.

A nurse is assessing a pregnant woman for the presence of edema. The nurse places a thumb on the top of the client's foot, then exerts pressure and releases it and notes that the thumb has left a persistent depression. On the basis of this finding, what does the nurse conclude? No edema is present The client is dehydrated Pitting edema is present Blood is not pooling in the extremities

Pitting edema is present Rationale: Edema in the lower extremities reflects pooling of blood, which results in a shift of intravascular fluid into the interstitial spaces. Dehydration is not likely to cause pitting edema. When pressure exerted with a finger or thumb leaves a persistent depression, the client is said to have "pitting edema." Therefore, the other options identify incorrect interpretations.

A clinic nurse is assessing a client who has had a cast applied to the lower left arm 1 week ago. The client tells the nurse that the skin is being irritated by the edges of the cast. What is the appropriate action on the part of the nurse? Bivalve the cast Use a nail file to smooth the rough edges Ask the primary health care provider to reapply the cast Place small pieces of tape over the rough edges of the cast

Place small pieces of tape over the rough edges of the cast Rationale: The appropriate action by the nurse is to petal (place small pieces of tape over) the rough edges of the cast to minimize the irritation. Bivalving is performed if the limb swells and the cast becomes too tight. Using a nail file to smooth the rough edges could cause pieces of the cast to fall into the cast, possibly resulting in the disruption of skin integrity. It is not necessary to contact the primary health care provider, and there is no reason to reapply the cast.

The wife of a client diagnosed with diabetes mellitus calls the nurse and reports that her husband's blood glucose level is 60 mg/dL (3.3 mmol/L) and that her husband is awake but groggy. What does the nurse tell the client's wife to immediately do? Call the primary health care provider Administer glucagon hydrochloride Call an ambulance to bring her husband to the emergency department Place some honey in her husband's mouth, between his gums and cheek

Place some honey in her husband's mouth, between his gums and cheek Rationale: The nurse tells the client's wife to immediately place some honey in her husband's mouth, between his gums and cheek. Once the sugar has been absorbed through the oral mucosa, the client can usually be aroused sufficiently to take a glass of juice, milk, or sugar-sweetened coffee or tea. The client and his spouse should be educated about the signs/symptoms of hypoglycemia (blood glucose level of 60 mg/dL [3.3 mmol/L] or lower). There is no reason at this time to call the primary health care provider or to call an ambulance to bring the client to the emergency department. Glucagon hydrochloride is used to treat hypoglycemic coma.

A nurse developing a nursing care plan for a client with abruptio placentae includes initial nursing measures to be implemented in the event of the development of shock. After contacting the primary health care provider, which does the nurse specify as the first action in the event of shock? Checking the client's urine output Inserting an intravenous (IV) line Obtaining informed consent for a cesarean delivery Placing the client in a lateral position with the bed flat

Placing the client in a lateral position with the bed flat Rationale: If the client exhibits signs/symptoms of hypovolemic shock, the client would first be placed in a lateral position, with the head of the bed flat to increase cardiac return. It is necessary for the nurse to take this initial action to minimize the effects of hypovolemic shock and promote tissue oxygenation and thus increase circulation and oxygenation of the placenta and other vital organs. The nurse would also contact the primary health care provider and monitor fetal status closely. After positioning the client, the nurse would insert IV lines in accordance with the primary health care provider's prescriptions and hospital protocols so that blood and replacement fluids may be administered. Quick preparation of the client for cesarean delivery may be necessary, but obtaining informed consent for the procedure is not the first action. Urine output is monitored to ensure an output of at least 30 mL/hr but, again, this is not the first action.

A postoperative client with deep-vein thrombosis is at risk for pulmonary embolism. For which characteristic sign/symptom of this complication does the nurse monitor the client? Pleuritic chest pain Slowed heart rate Chills and a high fever Decreased respiratory rate

Pleuritic chest pain Rationale: The characteristic signs/symptoms of pulmonary embolism are dyspnea, tachypnea, tachycardia, and pleuritic chest pain (sharp, stabbing pain on inspiration). Pulmonary embolism, whch results in blockage of the main artery of the lung or one of its branches, occurs when an object, such as a clot, or substance travels from elsewhere in the body through the bloodstream (embolism). The client may become apprehensive and restless. A low-grade fever may occur. Chills and a high fever are characteristics of an infection.

A nurse reviews the laboratory results of a hospitalized pregnant client with a diagnosis of sepsis who is at risk for disseminated intravascular coagulopathy (DIC). Which laboratory finding would indicate to the nurse that DIC has developed in the client? Increased platelet count Shortened prothrombin time Positive result on d-dimer study Decreased fibrin-degradation products

Positive result on d-dimer study Rationale: The d-dimer study is used to confirm the presence of fibrin split products; a positive result is indicative of DIC. DIC is a life-threatening defect in coagulation. As plasma factors are consumed, the circulating blood becomes deficient in clotting factors and unable to clot. Even as anticoagulation is occurring, inappropriate coagulation is also taking place in the microcirculation, and tiny clots form in the smallest blood vessels, blocking blood flow to the organs and causing ischemia. Laboratory studies help establish a diagnosis. The fibrinogen value and platelet count are usually decreased, prothrombin and activated partial thromboplastin times may be prolonged, and levels of fibrin degradation products (the most sensitive measurement) are increased.

A client diagnosed with chronic kidney disease who requires dialysis three times a week for the rest of his life says to the nurse, "Why should I even bother to watch what I eat and drink? It doesn't really matter what I do if I'm never going to get better!" On the basis of the client's statement, the nurse determines that the client is experiencing which problem? Anxiety Powerlessness Ineffective coping Disturbed body image

Powerlessness Rationale: Powerlessness is present when a client believes that he or she has no control over the situation or that his or her actions will not affect an outcome in any significant way. Anxiety is a vague uneasy feeling of apprehension. Some factors in anxiety include a threat or perceived threat to physical or emotional integrity or self-concept, changes in role function, and a threat to or change in socioeconomic status. Ineffective coping is present when the client exhibits impaired adaptive abilities or behaviors in meeting the demands or roles expected. Disturbed body image is diagnosed when there is an alteration in the way the client perceives his or her own body image.

The nurse discovers that a client receiving heparin sodium by way of continuous intravenous (IV) infusion has removed the IV tubing from the infusion pump to change his hospital gown. After assessing the client and placing the tubing back in the infusion pump, which medication does the nurse check for in the medication room in case a heparin overdose has occurred? Enoxaparin Phytonadione Protamine sulfate Aminocaproic acid

Protamine sulfate Rationale: The nurse checks for protamine sulfate in the medication room in case a heparin overdose occurs. If the IV tubing is removed from an infusion pump and the tubing is not clamped, the client will receive a bolus of the solution of the medication contained in the solution. Heparin is an anticoagulant, and the client who receives a bolus dose of heparin is at risk for bleeding. The nurse would notify the primary health care provider. A blood sample for partial thromboplastin time (PTT) would be drawn and the results of testing evaluated. If the PTT is too high, the infusion may be stopped for a time, or a dose of protamine sulfate, the antidote for heparin, may be prescribed. Enoxaparin is an anticoagulant. Phytonadione is the antidote for warfarin sodium.Aminocaproic acid is an antifibrinolytic, inhibiting clot breakdown.

A client is brought to the emergency department after sustaining smoke inhalation injury during a fire in the client's home. What should the nurse plan to do first? Check for a patent IV line Provide emotional support to the client Provide the client with 100% oxygen by mask Administer intravenous (IV) fluids as prescribed

Provide the client with 100% oxygen by mask Rationale: The nurse should first provide the client with 100% oxygen by mask. When smoke is inhaled, carbon monoxide binds with hemoglobin, displacing oxygen. A high carboxyhemoglobin level impairs tissue oxygenation, resulting in tissue asphyxia. Providing the client with 100% oxygen by mask reverses this condition. The nurse would next ensure that the client has a patent IV line and then administer fluids as prescribed. The nurse would also maintain body temperature, provide wound care as needed, and provide comfort and emotional support.

A client diagnosed with heart failure suddenly experiences profound dyspnea, pallor, audible wheezing, and cyanosis, and the nurse suspects pulmonary edema. What should the nurse do first? Obtain a pulse oximetry reading Raise the head of the client's bed Administer a dose of morphine sulfate Obtain a specimen for an arterial blood gas determination

Raise the head of the client's bed Rationale: The nurse would first raise the head of the client's bed and position the client to maximize chest expansion to ease the air hunger that the client is experiencing. Acute pulmonary edema is characterized by profound dyspnea, pallor, audible wheezing, and cyanosis. An arterial blood gas or pulse oximetry reading will reveal the need for supplemental oxygen. Morphine sulfate may be administered because it blunts the sympathetic response and promotes peripheral vasodilation. However, the nurse also needs to contact the primary health case provider to prescribe that medication. On the basis of the options provided, however, the initial action is placing the client in a head-elevated position.

A nurse is preparing to insert a nasogastric tube into a client. In which position does the nurse place the client before inserting the tube?

Rationale: A nasogastric tube is inserted through the nose and into the stomach for the purpose of gastric decompression or feeding the client. The client is placed in the Fowler position before insertion of the tube to promote comfort and easy insertion. A flat position may be used for clients who are hypotensive. In the reverse Trendelenburg position, the entire bed frame is tilted with the foot of the bed down and may be used to promote gastric emptying or prevent esophageal reflux. A trendelenburg position is one in which the entire bed frame is tilted with the head of the bed down and may be used for postural drainage or to facilitate venous return in clients with poor peripheral perfusion.

A nurse notes documentation in the client's medical record indicating that the client has a stage II pressure ulcer. On the basis of this information, which finding does the nurse expect to note?

Rationale: A stage I ulcer is characterized by intact skin that is red and does not blanch under external pressure. A stage II ulcer is characterized by nonintact skin. There is partial-thickness skin loss, and the wound may appear as an abrasion, a shallow crater, or a blister. A stage III ulcer is characterized by full-thickness skin loss, and the subcutaneous tissue may be damaged or necrotic. The damage extends down to but not through the underlying tissues. A deep crater-like appearance or eschar is present. A stage IV ulcer is characterized by full-thickness skin loss with extensive destruction, tissue necrosis, or damage to muscle, bone, or supporting structures. Sinus tracts may develop.

A nurse reviewing the medical record of a client with a diagnosis of infiltrating ductal carcinoma of the breast notes documentation of the presence of peau d'orange skin. On the basis of this notation, which finding would the nurse expect to note on assessment of the client's breast?

Rationale: Peau d'orange (French for "orange peel") is the term used to describe skin dimpling, resembling the skin of an orange, at the location of a breast mass. This change, along with increased vascularity, nipple retraction, or ulceration, may indicate advanced disease. Erythema, or reddening, of the breast indicates inflammation such as that resulting from cellulitis or a breast abscess. Paget's disease is a rare type of breast cancer that is manifested as a red, scaly nipple; discharge; crusting lasting more than a few weeks. In nipple retraction, the nipple is pointed or pulled in an abnormal direction. It is suggestive of malignancy.

A nurse is reviewing the record of a client scheduled for electroconvulsive therapy (ECT). Which diagnosis, if noted on the client's record, would indicate a need to contact the primary health care provider who is scheduled to perform the ECT? Recent stroke Hypothyroidism History of glaucoma Peripheral vascular disease

Recent stroke Rationale: Several conditions pose risks in the client scheduled for ECT. Among them are recent myocardial infarction or stroke and cerebrovascular malformations or intracranial lesions. Hypothyroidism, glaucoma, and peripheral vascular disease are not contraindications to this treatment.

A nurse in the postpartum unit is caring for a client who delivered a healthy newborn 12 hours ago. The nurse checks the client's temperature and notes that it is 100.4° F (38° C). On the basis of this finding, what should the nurse do? Notify the primary health care provider Recheck the temperature in 4 hours Encourage the client to breastfeed the newborn Institute strict bedrest for the client and notify the primary health care provider

Recheck the temperature in 4 hours Rationale: The nurse would recheck the temperature in 4 hours. A temperature of 100.4° F (38° C) is common during the 24 hours after childbirth and may be the result of dehydration or normal postpartum leukocytosis. If the increased temperature persists for more than 24 hours or exceeds 100.4° F (38° C), infection is a possibility, and the fever is reported. There is no reason to restrict place the client to strict bedrest or to notify the primary health care provider. Although the client would be encouraged to breastfeed her newborn, this action is unrelated to the client's temperature.

A postpartum nurse provides information about normal and abnormal characteristics of lochia to a client who has delivered a healthy newborn. Which finding does the nurse tell the client to report to the primary health care provider? Pink lochia on postpartum day 4 White lochia on postpartum day 11 Bloody lochia on postpartum day 2 Reddish lochia on postpartum day 8

Reddish lochia on postpartum day 8 Rationale: Reddish lochia on postpartum day 8 is an abnormal finding and would be reported to the primary health care provider. Lochia is the postdelivery vaginal discharge from the uterus consisting of blood from the vessels of the placental site and debris from the deciduas. Rubra is the bright-red lochial discharge that appears from delivery day to day 3. Serosa is the brownish-pink lochial discharge that appears on days 4 to 10. Alba is the white lochial discharge that appears on days 10 to 14.

While being seen by a primary health care provider, a client reports persistent fever, malaise, and night sweats. On physical examination, the primary health care provider palpates enlarged lymph nodes, and the client states that the nodes are painless. Hodgkin's lymphoma is suspected, and several diagnostic studies are performed. Which characteristic of this type of lymphoma does the nurse expect to note while reviewing the results of the diagnostic studies? Blast cells in the bone marrow Epstein-Barr virus in the blood Increased blood urea nitrogen (BUN) Reed-Sternberg cells on biopsy of a lymph node

Reed-Sternberg cells on biopsy of a lymph node Rationale: Hodgkin's lymphoma is a neoplasm of lymphatic tissue. The presence of giant multinucleated cells (Reed-Sternberg cells) is the hallmark of this disease. The presence of blast cells in the bone marrow is indicative of leukemia. Viral infections have been associated with Hodgkin's lymphoma but do not confirm the presence of the disease. An increased BUN level is indicative of a renal system disorder.

A client is brought to the emergency department by ambulance, and diabetic ketoacidosis is suspected. Blood samples are taken, and the nurse obtains supplies that will be needed to treat the client. Which type of insulin does the nurse take from the medication supply room for intravenous (IV) administration? NPH (Humulin N) Lente (Humulin L) Regular (Humulin R) NPH/regular 50%/50% (Humulin 50/50)

Regular (Humulin R) Rationale: Ketoacidosis, the most severe manifestation of insulin deficiency, is a life-threatening emergency. IV insulin is a primary component of treatment. Regular insulin, which has a rapid onset and short duration of action, is the only insulin that may be given IV. Because regular insulin forms a true solution, it is safe for IV use. NPH, Lente, NPH/regular 50%/50% are intermediate-acting insulins.

After delivering a normal, healthy newborn, a client complains of severe pelvic pain and a feeling of extreme fullness in the vagina, and uterine inversion is suspected. For which immediate intervention does the nurse prepare the client? Hysterectomy Insertion of an indwelling catheter Administration of oxytocin Replacement of the uterus through the vagina into a normal position

Replacement of the uterus through the vagina into a normal position Rationale: If uterine inversion is suspected, the immediate intervention by the nurse is to prepare the client for replacement of the uterus through the vagina. If this is not possible or effective, laparotomy with replacement is performed. Hysterectomy may be required. Intravenous lines are established to allow rapid fluid and blood replacement. A tocolytic medication or general anesthesia is usually needed to relax the uterus enough to replace it. Once the uterus has been replaced and the placenta removed, oxytocin is given to induce uterine contraction and control blood loss. To help prevent trapping of the inverted fundus in the cervix, oxytocin is not given until the uterus has been repositioned. An indwelling catheter is often inserted to aid monitoring of fluid balance and keep the bladder empty so that the uterus can contract fully, but this is not the immediate action taken by the nurse.

The nurse is explaining the diagnosis of congenital diaphragmatic hernia (CDH) related to a left posterolateral defect (also called Bochdalek hernia) to a postpartum client. What body system does the nurse emphasize is the system most at risk for problems? Respiratory system Genitourinary system Musculoskeletal system Gastrointestinal system

Respiratory system Rationale: With a congenital diaphragmatic hernia, abdominal contents herniate through an opening in the diaphragm. If caused by a left posterolateral defect, the respiratory system is most at risk for problems. This serious defect requires prompt recognition (usually in utero) and aggressive treatment to reduce its high mortality. Intestines and other abdominal structures, such as the liver, can enter the thoracic cavity, compressing the lung. Lung hypoplasia may occur on the affected side and to a lesser degree on the contralateral side. Ventilation is further compromised by hypoplasia and compression of the lung, including the airways and blood vessels. In addition to the anatomic defect, pulmonary hypoplasia and pulmonary hypertension have also been recently recognized as components in the pathology of CDH.

A nurse is providing instruction to a client with osteoporosis regarding appropriate foods to include in the diet. What one food item high in calcium does the nurse tell the client to eat? Corn Cocoa Peaches Sardines

Sardines Rationale: Foods high in calcium include milk and milk products, dark-green leafy vegetables, tofu and other soy products, sardines, and hard water. Osteoporosis is a chronic metabolic disease in which bone loss results in decreased density and sometimes fractures. Corn, cocoa, and peaches do not contain appreciable amounts of calcium.

A nurse is preparing a poster for a health fair booth promoting primary prevention of skin cancer. Which recommendations does the nurse include on the poster? Select all that apply. Seek medical advice if you find a skin lesion. Use sunscreen with a low sun protection factor (SPF). Avoid sun exposure before 10 a.m. and after 4 p.m. Wear a hat, opaque clothing, and sunglasses when out in the sun. Examine the body every 6 months for possibly cancerous or precancerous lesions.

Seek medical advice if you find a skin lesion. Wear a hat, opaque clothing, and sunglasses when out in the sun. Wear a hat, opaque clothing, and sunglasses when out in the sun.

A nurse is reviewing the laboratory results of a client in the emergency department with diabetic ketoacidosis (DKA). Which laboratory result would the nurse expect to note? Creatinine 1.0 mg/dL (88.3 μmol/L) Serum bicarbonate of 12 mEq/L (12 mmol/L) Blood urea nitrogen (BUN) of 15 mg/dL (5.325 mmol/L) Negative results on urinary ketone testing

Serum bicarbonate of 12 mEq/L (12 mmol/L) Rationale: In DKA, the serum bicarbonate concentration is less than 15 mEq/L(15 mmol/L). The BUN and creatinine levels may be increased because of dehydration, and urinary ketones are present. Additionally, the serum glucose is greater than 300 mg/dL (16.6 mmol/L) and the serum pH is less than 7.35. A BUN reading of 15 mg/dL(5.325 mmol/L), a creatinine level of 1.0 mg/dL (88.3 μmol/L), and a negative results on urinary ketone testing are normal findings.

An adult client with an ileostomy is admitted to the hospital with a diagnosis of isotonic dehydration. What findings does the nurse expect to note during the admission assessment? Select all that apply. Skin tenting Flat neck veins Weak peripheral pulses Moist oral mucous membranes A heart rate of 88 beats/min A respiratory rate of 18 breaths/min

Skin tenting Flat neck veins Weak peripheral pulses Rationale: Isotonic dehydration decreases circulating blood volume (hypovolemia), leading to inadequate tissue perfusion. The nurse would expect to note tachycardia, tachypnea, and dry oral mucous membranes. The oral mucous membranes may be covered with a thick, sticky, pastelike coating and may exhibit fissures. The client may also experience weight loss, lethargy or headache, sunken eyes, poor skin turgor (e.g., tenting), flat neck and peripheral veins, and low blood pressure. Peripheral pulses are weak, difficult to find, and easily obstructed with light pressure.

A nurse who will be staffing a booth at a health fair is preparing pamphlets containing information regarding the risk factors for osteoporosis. Which risk factors does the nurse include in the pamphlet? Select all that apply. Smoking A high-calcium diet High alcohol intake White or Asian ethnicity Participation in physical activities that promote flexibility and muscle strength

Smoking High alcohol intake White or Asian ethnicity Rationale: Osteoporosis is a chronic metabolic disease in which bone loss results in decreased density and sometimes fractures. Risk factors include being 65 years or older in women, 75 years or older in men, family history of the disorder, history of fracture after age 50, white or Asian ethnicity, low body weight and slender build, chronically low calcium intake, a history of smoking, high alcohol intake, and lack of physical exercise or prolonged immobility.

A nurse is monitoring a pregnant client with suspected partial placenta previa who is experiencing vaginal bleeding. Which finding would the nurse expect to note on assessment of the client? Painful vaginal bleeding Sustained tetanic contractions Complaints of abdominal pain Soft, relaxed, nontender uterus

Soft, relaxed, nontender uterus Rationale: Partial placenta previa is incomplete coverage of the internal os by the placenta. One characteristic of placenta previa is painless vaginal bleeding. The abdominal assessment would reveal a soft, relaxed, nontender uterus with normal tone. Vaginal bleeding and uterine pain and tenderness accompany placental abruption, especially with a central abruption and blood trapped behind the placenta. In placental abruption, the abdomen feels hard and boardlike on palpation as the blood penetrates the myometrium, resulting in pain and uterine irritability. A sustained tetanic contraction may occur if the client is in labor and the uterine muscle cannot relax.

Propylthiouracil has been prescribed for a client with Graves disease, and the nurse provides instructions to the client about the medication. For which occurrence does the nurse tell the client to contact the primary health care provider? Fatigue Diaphoresis Sore throat Heat intolerance

Sore throat Rationale: "Propylthiouracil is an antithyroid medication that blocks thyroid hormone production. One adverse effect of propylthiouracil is agranulocytosis. Agranulocytosiss is a serous condition that occurs when the body does not make enough granulocytes (type of white blood cell). The immune system is affected, so a person with this condition is at risk of developing a severe or even life-threatening infection." The client is instructed to report signs/symptoms of infection such as a sore throat. Graves disease (hyperthyroidism) occurs as a result of excessive thyroid hormone secretion. Fatigue may or may not occur in this disorder; however, it is not an adverse effect of the medication. One manifestation of Graves disease is heat intolerance, and the client may experience diaphoresis even when the environmental temperature is comfortable for others. Agranulocytos is a serious that occurs when the body does not make enough granulocytes (type of white blood cell). The immune system is affected, so a person with this condition is at risk of developing a severe or even life-threatening infection.

A client has been given a prescription for lovastatin. Which food does the nurse instruct the client to limit consumption of while taking this medication? Steak Spinach Chicken Oranges

Steak Rationale: Lovastatin is a lipid-lowering agent. The client is instructed to consume foods that are low in fat, cholesterol, and complex sugars. The item highest in fat here is steak; therefore the client should limit the intake of steak. Fruits, vegetables, and chicken are low in fat.Test-Taking Strategy: Focus on the subject

A nurse is monitoring a client with bronchogenic carcinoma for signs/symptoms of superior vena cava syndrome. For which early sign of this oncological emergency does the nurse assess the client? Dyspnea Cyanosis Hypotension Stokes sign

Stokes sign Rationale: In superior vena cava syndrome, the superior vena cava is compressed or obstructed by tumor growth. Early signs/symptoms, which generally occur in the early morning, include edema of the face, especially around the eyes (periorbital edema), and complaints of tightness of a shirt or blouse collar (Stokes sign). As the compression worsens, the client experiences edema of the hands and arms, dyspnea, erythema of the upper body, and epistaxis (nosebleeds). Late (and life-threatening) signs/symptoms include hemorrhage, cyanosis, mental status changes resulting from lack of blood to the brain, decreased cardiac output, and hypotension (low blood pressure). Death may result if the compression is not relieved.

A client is receiving an intravenous infusion of oxytocin to stimulate labor. The nurse monitoring the client notes uterine hypertonicity. What does the nurse immediately do? Stop the oxytocin infusion Check the vagina for crowning Encourage the client to take short, deep breaths Increase the rate of the oxytocin infusion and call the primary health care provider

Stop the oxytocin infusion Rationale: The nurse would immediately stop the oxytocin infusion and increase the rate of the nonadditive solution, position the client in a side-lying position, and administer oxygen with the use of a snug face mask at 8 to 10 L/min. If uterine hypertonicity or a nonreassuring fetal heart rate pattern is detected, the nurse must intervene to reduce uterine activity and increase fetal oxygenation. The nurse would also notify the primary health care provider. Oxytocin is a synthetic compound identical to the natural hormone secreted from the posterior pituitary gland. It is used to induce or augment labor at or near term. The nurse monitors uterine activity for the establishment of an effective labor pattern and for complications associated with the use of the medication. Checking the vagina for crowning; encouraging the client to take short, deep breaths; and increasing the rate of the oxytocin infusion are not the immediate actions.

A hospitalized client scheduled for surgery is told by the primary health care provider that he/she is extremely anemic and will need a blood transfusion. The client, a Jehovah's Witness, refuses the transfusion. What is the most appropriate initial nursing action? Supporting the client's decision to refuse the transfusion Teaching the client ways to increase dietary intake of iron Telling the client about the importance of the blood transfusion Telling the client that if he/she refuses the blood transfusion, the surgery will have to be canceled

Supporting the client's decision to refuse the transfusion Rationale: The most appropriate initial nursing action is to be supportive of a Jehovah's Witness' refusal to receive a blood transfusion. Religious beliefs can influence individual and family responses to health and illness, and religion can be a source of support and comfort for a client. Awareness of a client's religious beliefs enables the nurse to appropriately support the client's decisions regarding care. The nurse might need to instruct the client in how to increase dietary intake of iron, but this would not be the initial action. Telling the client about the importance of blood transfusions and telling her that the surgery will need to be canceled if she refuses the blood transfusion are both inappropriate.

A nurse is caring for a client with a diagnosis of abruptio placentae. For which early signs/symptoms of hypovolemic shock does the nurse closely monitor the client? Select all that apply. Tachycardia Cool, clammy skin Decreased respiratory rate Diminished peripheral pulses Urine output of less than 30 mL/hr

Tachycardia Diminished peripheral pulses Rationale: When hypovolemic shock develops, the body attempts to compensate for decreased blood volume and to maintain oxygenation of essential organs by increasing the rate and effort of the heart and lungs by shunting blood from less essential organs, such as the skin and extremities, to more essential ones, such as the brain and kidneys. This compensatory mechanism results in the early signs/symptoms of hypovolemic shock, which include tachycardia, diminished peripheral pulses, normal or slightly decreased blood pressure, increased respiratory rate, and cool, pale skin and mucous membranes. The compensatory mechanism fails if hypovolemic shock progresses and there is insufficient blood to perfuse the brain, heart, and kidneys. Later signs/symptoms of hypovolemic shock include decreasing blood pressure, pallor, cold and clammy skin, and urine output of less than 30 mL/hr.

A nurse working the evening shift is helping clients get ready for sleep. A female client diagnosed with mania is hyperactive and pacing the hallway. What is the most appropriate action the nurse can take? Stay with the client and observe her behavior Take the client to the bathroom and provide her with a warm bath Tell the client that it is time for sleep and that she needs to go to her room Tell the client that other clients are trying to sleep and that she is being disruptive

Take the client to the bathroom and provide her with a warm bath Rationale: At bedtime, the nurse should take the client to the bathroom and provide warm baths, soothing music, and medication when indicated. For the client with mania, the nurse needs to promote relaxation, rest, and sleep and to minimize manic behavior. The nurse should encourage frequent rest periods during the day and keep the client in areas of low stimulation. The client should not consume products containing caffeine. Staying with the client and observing her behavior, telling the client that it is time to go to sleep and to go to her room, and telling the client that other clients are trying to sleep and that she is being disruptive do not address the client's needs and are not measures that will help the client relax and sleep.

A nurse is preparing medication instructions for a client who will be taking a daily oral dose of digoxin 0.25 mg in the treatment of heart failure (HF). Which instructions should the nurse include on the list? Select all that apply. Take your pulse before taking each dose. Avoid eating foods that contain potassium. Take the digoxin at the same time each day. Take the digoxin with a chewable antacid to prevent nausea. If you forget to take your daily dose, double the dose on the next day. Notify the primary health care provider if you experience loss of appetite, muscle weakness, or visual disturbances.

Take your pulse before taking each dose. Take the digoxin at the same time each day. Notify the primary health care provider if you experience loss of appetite, muscle weakness, or visual disturbances. Rationale: Loss of appetite, muscle weakness, and visual disturbances are signs/symptoms of digoxin toxicity, and the primary health care provider must be notified if any of these occur. Digoxin is a cardiac glycoside that increases the force of myocardial contraction. It is used to treat HF and to control the ventricular rate in clients with atrial fibrillation. The client is instructed to take the medication at the same time each day and to check the pulse rate before taking the medication. If the pulse rate is slower than 60 beats/min or faster than 100 beats/min, the primary health care provider is notified. The medication is not taken with an antacid, because the antacid will affect absorption of the medication. If the client forgets to take a dose, it needs to be taken as soon as remembered. The dose is never doubled. Hypokalemia predisposes a client to digoxin toxicity. A client is not instructed to avoid foods that contain potassium unless specifically instructed to do so by the primary health care provider.

A nurse is caring for a client who has had a stroke and is experiencing hemianopsia. Which measure does the nurse take in the care of the client? Approaching the client from the side of nonintact vision Teaching the client to move the head from side to side (scan) when eating Placing objects needed for self-care within the client's nonintact visual field Positioning the client in the room so that his nonintact visual field faces the door

Teaching the client to move the head from side to side (scan) when eating Rationale: Hemianopsia is the loss of vision in a portion of the visual field. Approaching the client from the side of nonintact vision, placing objects needed for self-care within the client's nonintact visual field, and positioning the client so that the damaged part of the visual field faces the door are incorrect because the client depends on and needs to use the intact portion of the visual field for sensory input. The client is taught to move the head from side to side (scan) to compensate for a diminished visual field. Scanning is also important when the client is eating.

A client on the mental health unit says to the nurse, "Everything is contaminated." The client scrubs his/her hands if forced to touch any object. While planning care, what does the nurse remember about compulsive behavior? Temporarily eases anxiety in the client Is an attempt on the client's part to punish self Is an attempt on the client's part to seek the attention of others Is a response by the client to voices saying that everything is contaminated and that he/she must engage in this behavior

Temporarily eases anxiety in the client Rationale: Compulsions are ritualistic behaviors that an individual feels driven to perform in an attempt to reduce anxiety. Obsessions are thoughts, impulses, or images that persist and recur so that they cannot be dismissed from the mind. The other options identify interpretations of the client's obsessive behavior.

A client with multiple sclerosis (MS) has been started on baclofen for muscle spasms. The client calls the primary health care provider's office 1 week after beginning the medication and tells the nurse that she/he feels extremely drowsy. What does the nurse most appropriately tell the client? That she/he will need to be seen by the primary health care provider That the medication will need to be discontinued That drowsiness usually diminishes with continued therapy To stop the medication for 2 days and then resume it at the prescribed dosage

That drowsiness usually diminishes with continued therapy Rationale: Baclofen is a centrally acting skeletal muscle relaxant. Side effects of baclofen include drowsiness, dizziness, weakness, and nausea. The nurse most appropriately tells the client that drowsiness usually diminishes with continued therapy. However, the client is told to avoid activities that require alertness until the response to the medication has been established. Therefore it is not necessary to call the primary health care provider or discontinue or stop the medication.

A nurse is providing home care instructions to a client diagnosed with coronary artery disease (CAD) who will be discharged home and will be taking 1 aspirin daily. What does the nurse tell the client? To stop the aspirin if nausea occurs To take the aspirin on an empty stomach That ringing in the ears is a sign of toxicity That the aspirin is a short-term treatment and will probably be discontinued in 2 weeks

That ringing in the ears is a sign of toxicity Rationale: Ringing in the ears (tinnitus) is a sign of toxicity; if it occurs, the client should contact the primary health care provider. Aspirin, an antiplatelet agent, helps prevent blood clotting. A single low daily dose of aspirin may be a long-term component of the standard treatment regimen for the client with CAD. Gastric irritation may occur with the use of aspirin, and the client is advised to take the daily dose with food to prevent this. The client would not stop the medication if nausea occurs but would instead take the medication with food to relieve this discomfort.

The mother of an adolescent diagnosed with type 1 diabetes mellitus tells the nurse that her child is a member of the school soccer team and expresses concern about her child's participation in sports. What does the nurse tell the mother after providing information to the mother about diet, exercise, insulin, and blood glucose control? To always administer less insulin on the days of soccer games That it is best not to encourage the child to participate in sports activities That the child should eat a carbohydrate snack about a half-hour before each soccer game To administer additional insulin before a soccer game if the blood glucose level is 240 mg/dL (13.3 mmol/L) or higher and ketones are present.

That the child should eat a carbohydrate snack about a half-hour before each soccer game Rationale: The child with diabetes mellitus who is active in sports requires additional food intake in the form of a carbohydrate snack about a half-hour before the anticipated activity. Additional food will need to be consumed, often as frequently as every 45 minutes to 1 hour, during prolonged periods of activity. If the blood glucose level is increased (240 mg/dL [13.3 mmol/L] or more) and ketones are present before planned exercise, the activity should be postponed until the blood glucose has been controlled. Moderate to high ketone values should be reported to the primary health care provider. There is no reason for the child to avoid participating in sports.

Disulfiram is prescribed to a client with an alcohol abuse problem. The nurse provides information about the medication. What does the nurse tell the client? That driving is prohibited while the client is taking the medication To take the medication immediately if the desire to drink alcohol occurs That the effect of the medication ends as soon as the client stops taking the medication That the medication cannot be started until at least 12 hours has elapsed since the client's last ingestion of alcohol

That the medication cannot be started until at least 12 hours has elapsed since the client's last ingestion of alcohol Rationale: Disulfiram is an alcohol abuse deterrent prescribed to motivated clients who have shown the ability to stay sober. Driving is not prohibited; however, the client is instructed to use caution when driving and performing other tasks that require alertness. The medication is taken daily (not just when the client has a desire to drink alcohol), and the effects of the medication last 5 days to 2 weeks after the last dose is taken. The medication cannot be started until at least 12 hours has elapsed since the client's last ingestion of alcohol. Otherwise, an alcohol-disulfiram reaction will occur, with effects consisting of facial flushing, sweating, a throbbing headache, neck pain, tachycardia, respiratory distress, a potentially serious decrease in blood pressure, and nausea and vomiting. This reaction may last 30 to 120 minutes.

A nurse is monitoring a client who has undergone subtotal thyroidectomy for signs/symptoms of postoperative complications. Which finding would be a matter of concern for the nurse as an indication of hypocalcemia? The client's temperature is 100.6˚ F (38.1˚ C). The client's voice is hoarse and weak. The client's heart rate is 92 beats/min. The client complains of a tingling sensation around the mouth.

The client complains of a tingling sensation around the mouth. Rationale: The parathyroid glands may be damaged or their blood supply impaired during thyroid surgery. Hypocalcemia and tetany result when the level of parathyroid hormone decreases. The nurse assesses the client for signs/symptoms of tingling around the mouth or of the toes and fingers and muscle twitching, signs/symptoms of calcium deficiency. Calcium gluconate or calcium chloride for intravenous administration must be available in an emergency situation (severe hypocalcemia). Hoarseness and a weak voice may result if the laryngeal nerve is damaged during surgery; however, the client is reassured that the hoarseness is usually temporary. Tachycardia and a fever are signs/symptoms of thyroid storm or thyroid crisis, a complication of thyroid surgery. Postoperative thyroid storm is rare because clients receive antithyroid medications, beta-blockers, corticosteroids, and iodides before thyroid surgery.

A nurse is conducting a preoperative psychosocial assessment of a client who is scheduled for a mastectomy. Which finding would cause the nurse to conclude that the client is at risk for poor sexual adjustment after the mastectomy? The client reports a history of sexual abuse by her father. The client reports irregular menses relationship with her spouse. The client reports a satisfying intimate relationship with her spouse. The client reports that her and her spouse have never been able to conceive children.

The client reports a history of sexual abuse by her father. Rationale: Clients at risk for self-esteem problems and poor sexual adjustment after mastectomy include those who report a lack of support from a spouse or partner; the existence of an unhappy, unstable intimate relationship; or a history of sexual problems or of sexual abuse, such as rape or incest. Clients with problems involving intimate relationships and sexuality should be referred for counseling. The remaining options are unrelated to the problem of poor sexual adjustment.

Risperidone is prescribed for a client hospitalized in the mental health unit for the treatment of a psychotic disorder. Which finding in the client's medical record would prompt the nurse to contact the prescribing primary health care provider before administering the medication? The client has a history of cataracts. The client has a history of hypothyroidism. The client takes a prescribed antihypertensive. The client is allergic to acetylsalicylic acid (aspirin).

The client takes a prescribed antihypertensive. Rationale: Risperidone is an antipsychotic medication. Contraindications to the use of risperidone include cardiac disorders, cerebrovascular disease, dehydration, hypovolemia, and therapy with antihypertensive agents. Risperidone is used with caution in clients with a history of seizures. History of cataracts, hypothyroidism, or allergy to aspirin does not affect the administration of this medication.

Levothyroxine is prescribed to a client with hypothyroidism. One week after beginning the medication, the client calls the primary health care provider's office and tells the nurse that the medication has not helped. What does the nurse most appropriately tell the client? The medication will need to be changed The full therapeutic effect may take 4 weeks An additional medication will have to be added to the regimen The blood levels of the client's thyroid hormones will need to be rechecked

The full therapeutic effect may take 4 weeks Rationale: The nurse most appropriately tells the client that full therapeutic effect may take 1 month. For this reason, changing the medication and the addition of a medication are not necessary. Levothyroxine is used in the treatment of hypothyroidism. Therapy with levothyroxine may begin with small doses that are gradually increased. Blood levels of thyroid hormones are checked during therapy with levothyroxine, obtaining these levels after 1 week of therapy will not provide useful information.

A nurse is assessing a client who is experiencing chest pain. Which observation indicates to the nurse that the pain is most likely a result of angina? The pain is relieved by rest and nitroglycerin. The pain is relieved by the administration of an antacid. The pain is relieved by the administration of an antiinflammatory medication. The pain is relieved with an upright sitting position and the administration of an analgesic.

The pain is relieved by rest and nitroglycerin. Rationale: Anginal pain may radiate to the left shoulder, arm, neck, or jaw. It is often precipitated by exertion or stress, has few associated signs/symptoms, and is relieved by rest and nitroglycerin. Antacids provide relief from chest pain (squeezing heartburn pain) caused by esophageal or gastric disorders. An upright sitting position, the administration of an analgesic, or the administration of an antiinflammatory medication often relieves chest pain (sudden, sharp, and stabbing) caused by pericarditis.

A client with rheumatoid arthritis is taking 3.6 g of acetylsalicylic acid daily in a divided dose. At the primary health care provider's office, the client tells the nurse that she has been experiencing ringing in the ears over the past few days. What should the nurse tell the client? This is expected and nothing to be concerned about It is important to drink at least 10 glasses of water a day to prevent ringing in the ears This is a sign/symptom of toxicity, so the aspirin will be discontinued and replaced with a nonsteroidal antiinflammatory medication The primary health care provider will probably withhold the aspirin until the signs/symptoms have subsided, then resume the aspirin at a lower dosage

The primary health care provider will probably withhold the aspirin until the signs/symptoms have subsided, then resume the aspirin at a lower dosage Rationale: The nurse should tell the client that the primary health care provider will probably withhold the aspirin until the signs/symptoms have subsided, then resume the aspirin at a lower dosage. Salicylism is a syndrome that begins to develop when the aspirin level climbs just slightly above the therapeutic level. Overt signs/symptoms include tinnitus (ringing in the ears), sweating, headache, and dizziness. If salicylism develops, aspirin is withheld until the signs/symptoms have subsided; therapy should then be resumed with a small reduction in dosage. Tinnitus is not an expected finding. Drinking water will not prevent tinnitus. A nonsteroidal antiinflammatory medication will not be prescribed, because these medications' chemical properties are similar to those of aspirin.

A nurse assessing the wound of a client with a stage 3 pressure ulcer notes that the wound bed is pale. What does the nurse interpret this finding as a possible indication of? The wound is healthy The wound is improving Necrotic tissue is present The client's hemoglobin level is low

The wound is improving Rationale: A pale wound bed may reflect a low hemoglobin value. Healthy full-thickness wounds have a granular beefy-red appearance. The color of a wound provides information about vascular supply, infection, the presence of healthy or necrotic tissue, and nutritional status. Necrotic tissue is white, yellow, gray, or black. Pseudomonas infection can produce greenish drainage in a wound bed.

Empyema develops in a client with an infected pleural effusion, and the nurse prepares the client for thoracentesis. The nurse is assisting the primary health care provider with the procedure. What characteristics of the fluid removed during thoracentesis should the nurse expect to note? Clear and yellow Thick and opaque White and odorless Clear, with a foul odor

Thick and opaque Rationale: Empyema is the accumulation of pus in the pleural space. Empyema fluid is thick, opaque, exudative, and intensely foul-smelling. Clear and yellow, white and odorless, and clear and foul-smelling are incorrect descriptions of the fluid that occurs in this disorder.

A child with growth hormone deficiency will be receiving somatropin. The nurse provides information to the mother about the medication. Which of the following laboratory values does the nurse tell the mother will require monitoring? Creatinine Hemoglobin Blood urea nitrogen (BUN) Thyroid-stimulating hormone (TSH)

Thyroid-stimulating hormone (TSH) Rationale: TSH is the laboratory value the nurse tells the mother to monitor. Somatropin is a growth hormone. One adverse reaction to somatropin is hypothyroidism. Thyroid function is assessed before treatment and periodically thereafter. Creatinine and BUN are used to evaluate renal function, and hemoglobin reflects hematologic activity.

A nurse is providing instruction in how to perform Kegel exercises to a client with stress incontinence. What does the nurse tell the client to do? Always perform the exercises while lying down Expect an improvement in the control of urine in about 1 week Tighten the pelvic muscles for as long as 5 minutes, three or four times a day Tighten the pelvic muscles for a slow count of 10, then relax for a slow count of 10

Tighten the pelvic muscles for a slow count of 10, then relax for a slow count of 10 Rationale: Kegel exercises strengthen the muscles of the pelvic floor. To perform the exercises, the client is taught to tighten the pelvic muscles to a slow count of 10, then relax to a slow count of 10. The client is also instructed to do this exercise 15 times while lying down, sitting up, and standing (a total of 45 repetitions). The client is told that an improvement in the control of urine will be noticed after several weeks of the exercises; some individuals report that improvement takes as long as 3 months.

A nurse is preparing a pregnant client in the third trimester for an amniocentesis. What does the nurse tell the client is the reason amniocentesis is often performed during the third trimester? To know the sex of the fetus To discover genetic characteristics To establish an accurate age for the fetus To assess the degree of fetal lung maturity

To assess the degree of fetal lung maturity Rationale: Amniocentesis is the aspiration of fluid from the amniotic sac for examination. Common indications for amniocentesis during the third trimester include assessment of fetal lung maturity and evaluation of fetal condition when the woman has Rh isoimmunization. A common purpose of amniocentesis in the second trimester is to examine fetal cells in the amniotic fluid to identify chromosomal abnormalities. Other methods of genetic analysis, such as those for metabolic defects in the fetus, may be performed on the cells as well. The sex and age of the fetus are not determined with the use of amniocentesis.

Iodine solution is prescribed to a client who is scheduled for subtotal thyroidectomy. The client calls the nurse at the clinic and complains of a burning sensation in the mouth and soreness of the gums and teeth. What does the nurse most appropriately tell the client? To contact the primary health care provider That these are expected side effects of the medication That these discomforts will resolve with continued therapy To stop the medication for the next 24 hours and then continue as prescribed

To contact the primary health care provider Rationale: The client should most appropriately be instructed to notify the primary health care provider if the client experiences burning sensation in the mouth and soreness of the gums and teeth. Iodine can produce toxicity (iodism). The client should be instructed about the signs/symptoms of iodism, which includes a brassy taste and burning sensation in the mouth, soreness of the gums and teeth, frontal headache, coryza, salivation, and skin eruptions. The client's signs/symptoms are not expected side effects of the medication and will not resolve with continued therapy; in fact, continued therapy will worsen the client's condition. Stopping the medication for 24 hours and then continuing it as prescribed will also worsen the client's condition.

Ferrous sulfate is prescribed for a client. What does the nurse tell the client is best to take the medication with? Milk Water Any meal Tomato juice

Tomato juice Rationale: Ferrous sulfate is an iron product. Absorption of iron is best promoted when the supplement is taken with orange juice or tomato juice another food source of vitamin C or ascorbic acid. Calcium and phosphorus in milk decrease iron absorption. Water has no effect on the absorption of vitamin C. Telling the client to take the medication with any meal of the day does not guarantee that the iron will be taken with a food source of vitamin C or ascorbic acid. Additionally, it is best to take the iron supplement between meals with a drink high in ascorbic acid.

A client who has been undergoing long-term therapy with an antipsychotic medication is admitted to the inpatient mental health unit. Which finding does the nurse, knowing that long-term use of an antipsychotic medication can cause tardive dyskinesia, monitor in the client? Fever Diarrhea Hypertension Tongue protrusion

Tongue protrusion Rationale: The clinical manifestations include abnormal movements (dyskinesia) and involuntary movements of the mouth, tongue ("flycatcher tongue"), and face. Tardive dyskinesia is a severe reaction associated with long-term use of antipsychotic medications. In its most severe form, tardive dyskinesia involves the fingers, arms, trunk, and respiratory muscles. When this occurs, the medication is discontinued. Fever, diarrhea, and hypertension are not characteristics of tardive dyskinesia.

A nurse is preparing the room of a client in skeletal traction who will be admitted to the nursing unit. Which item for use by the client does the nurse identify as most important? Telephone Television Trapeze bar Bedside commode

Trapeze bar Rationale: The most important item for the client in skeletal traction to use is a trapeze bar. This bar is a triangular device that hangs from a securely fastened overhead bar that is attached to the bed frame. It allows the client to pull up with the upper extremities to raise the trunk off the bed. It is a useful device for helping increase independence, maintain upper-body strength, and reducing the shearing action that results when the client slides across or up and down in the bed. The client in traction would not be allowed to get out of bed to use a bedside commode; rather, a bedpan or fracture pan would be used. Although a telephone and a television are helpful for diversion and maintaining social contact, they are not the most important items.

A nurse provides home care instructions to a client with mild preeclampsia. What does the nurse tell the client? Sodium intake is restricted Fluid intake must be limited to 1 quart (1 litres) each day Urine output must be measured and the primary health care provider should be notified if output is less than 500 mL in a 24-hour period Urinary protein must be measured and the primary health care provider should be notified if the results indicate a trace amount of protein

Urine output must be measured and the primary health care provider should be notified if output is less than 500 mL in a 24-hour period Rationale: Urine output of less than 500 mL/24 hr should prompt the client to notify the primary health care provider. Preeclampsia is considered mild when the diastolic blood pressure does not exceed 100 mm Hg, proteinuria is no more than 500 mg/day (trace to 1+), and signs/symptoms such as headache, visual disturbances, and abdominal pain are absent. The diet should provide ample protein and calories, and fluid and sodium should not be limited. The disease is considered severe when the blood pressure is higher than 160/110 mm Hg, proteinuria is greater than 5 g/24 hr (3+ or more), and oliguria is present (500 mL or less in 24 hours).

A nurse develops a list of home care instructions for a client who is wearing a halo fixation device after sustaining a cervical fracture. Which instructions should the nurse include? Select all that apply. Use a straw to drink. Avoid sexual activity while the vest is in place. Apply powder under the vest to prevent irritation. Use caution when leaning forward or backward. Wear snug clothing to prevent the device from shifting. Do not drive, because full range of vision is impaired with the device.

Use a straw to drink. Use caution when leaning forward or backward. Use caution when leaning forward or backward. Rationale: Straws are used to drink, and meat and other foods are cut into small pieces to facilitate swallowing. The weight of the halo device alters balance; therefore the client should use caution when leaning forward or backward. The client is also told not to drive, because full range of vision is impaired with the device. A halo fixation (stabilization) device is used to prevent the head and neck from moving after a neck injury. The halo fixation device is not removed. Sexual activity does not have to be avoided; the client is instructed to use a position of comfort. Powders and lotions are used sparingly or not at all to prevent buildup of moisture and subsequent skin breakdown. The client is instructed to wear loose clothing with a large neck. The halo fixation device should not shift; if it does, the primary health care provider must be notified. The client is taught to sleep with the head supported with a small pillow to prevent unnecessary pressure and discomfort.

A nurse caring for a woman in labor is reading the fetal monitor tracing (see figure). How does the nurse interpret this finding? Umbilical cord compression Pressure on the fetal head during a contraction Uteroplacental insufficiency during a contraction Inadequate pacemaker activity of the fetal heart

Uteroplacental insufficiency during a contraction Rationale: The observation that the nurse noted in this tracing is late decelerations. Late decelerations constitute an ominous pattern in labor because they suggest uteroplacental insufficiency, possibly associated with a contraction. Early decelerations result from pressure on the fetal head during a contraction. Variable decelerations suggest umbilical cord compression. The term short-term variability refers to the difference between successive heartbeats, indicating that the natural pacemaker function of the fetal heart is working properly.

A nurse provides dietary instructions about foods that will promote healing to a client diagnosed with osteoporosis who has sustained a fracture. The nurse tells the client to consume foods high in what nutrient? Fats Vitamin C Carbohydrates Concentrated sugar

Vitamin C Rationale: If a fracture has occurred, the nurse encourages the client to eat foods high in vitamin C, protein, and iron, because these nutrients will promote healing. Dietary therapy for the client with osteoporosis includes foods high in calcium. Although fats and carbohydrates should be included in the daily diet, they are not specifically related to the healing process. Foods containing concentrated sugar do not promote healing.

A nurse is reviewing the medical record of an infant in whom hypertrophic pyloric stenosis (HPS) is suspected. Which characteristics associated with the disorder does the nurse expect to see documented in the infant's medical record? Select all that apply. Weight loss Facial edema Metabolic acidosis Projectile vomiting Distended upper abdomen

Weight loss Projectile vomiting Distended upper abdomen Rationale: HPS occurs when the circular muscle of the pylorus becomes thickened, causing constriction of the pylorus and obstruction of the gastric outlet. Clinical manifestations include projectile vomiting, a hungry infant who eagerly accepts a second feeding after the vomiting episode, weight loss, signs/symptoms of dehydration, and a distended upper abdomen. A readily palpable olive-shaped mass in the epigastrium just to the right of the umbilicus is noted, and gastric peristaltic waves, moving from left to right across the epigastrium, are visible. Laboratory findings include metabolic alkalosis, a result of the vomiting that occurs in this disorder. Facial edema and metabolic acidosis do not occur in this disorder.

The nurse, auscultating the breath sounds of a client, hears these sounds. What are they? Rhonchi Crackles Wheezes Vesicular

Wheezes Rationale: Wheezes are continuous squeaky, musical sounds, the result of air rushing through narrowed airways. They are associated with inflammation, bronchospasm, edema, secretions, or pulmonary vessel engorgement. Rhonchi are continuous low-pitched, coarse snoring sounds associated with thick, tenacious secretions; sputum production; or obstruction by a foreign body. Crackles are discontinuous popping sounds caused by air moving into previously deflated airways (they sound like air is being rolled between the fingers near the ear). Vesicular sounds are rustling sounds heard over the peripheral lung fields. They are a normal finding.

A nurse is reviewing the laboratory results of a female client with ovarian cancer who is undergoing chemotherapy. Which finding indicates to the nurse that the client is experiencing an adverse effect of the chemotherapy? Sodium 140 mEq/L (140 mmol/L) Hemoglobin 12.5 g/dL (125 g/L) Blood urea nitrogen (BUN) 20 mg/dL (7.1 mmol/L) White blood cell count of 2.5 × 103/μL (2.5 × 109/L)

White blood cell count of 2.5 × 103/μL (2.5 × 109/L) Rationale: A white blood cell count of 2.5 × 103/μL (2.5 × 109/L) is low and puts the client at risk for infection. The normal white blood cell count ranges from 4.0-11.0 × 103/μL (4.0-11.0 × 109/L). All of the other values are within normal limits. The normal sodium level is 135-145 mEq/L (135-145 mmol/L). The normal hemoglobin level for a female ranges from 13.2-17.3 g/dL (132-173 g/L). The normal BUN concentration ranges from 6-20 mg/dL (2.1-7.1 mmol/L).

A client with acute gouty arthritis is being started on medication therapy with indomethacin. The nurse provides medication instructions to the client. How does the nurse tell the client to take the medication? At bedtime With food 1 hour before meals On an empty stomach

With food Rationale: The client is instructed to take the medication with food. Indomethacin is a nonsteroidal antiinflammatory medication that produces analgesic and antiinflammatory effects by inhibiting prostaglandin synthesis. Adverse effects include ulceration of the esophagus, stomach, duodenum, and small intestine.

A nurse transcribing the prescriptions of a client admitted to the nursing unit notes that metformin daily has been prescribed. The nurse makes a note in the client's medication record. When should the medication be administered? At noon With supper With the morning meal With the midafternoon snack

With the morning meal Rationale: Metformin is an antihyperglycemic medication used as an adjunct to diet in the management of type 2 diabetes mellitus. When it is prescribed for daily use, it is administered with the morning meal. Giving the medication at noon, with supper, or with a midafternoon snack is incorrect because the medication is needed to maintain a controlled and consistent blood glucose level. If the prescribed metformin is an extended-release form, the primary health care provider may prescribe a daily dose of the medication with the evening meal, because this timing may enhance absorption as a result of slower nighttime gastrointestinal transit time.

A client living in a long-term care facility shouts at the nurse, "Get out of my room! I don't need your help!" What is the most appropriate way for the nurse to document this occurrence in the client's record? Writing that the client is very agitated Writing that the client yelled at the nurse Writing that the client is able to perform his/her own care Writing down the client's words and placing them in quotation marks

Writing down the client's words and placing them in quotation marks Rationale: Documenting the client's words verbatim and placing them in quotations ensures accurate data. An objective description is the result of direct observation and measurement. Documenting inferences without supporting factual data is not acceptable, because a client's statements may be misunderstood. The remaining options do not provide objective descriptions.

A client diagnosed with tuberculosis will be taking pyrazinamide, and the nurse provides instructions about the adverse effects of the medication. For which occurrence does the nurse tell the client to contact the primary health care provider? Headache Yellow skin Difficulty sleeping Nasal congestion

Yellow skin Rationale: The nurse instructs the client to contact the primary health care provider if he/she experiences jaundice (yellow skin or eyes); unusual tiredness; fever; loss of appetite; or hot, painful, or swollen joints. Adverse effects include hepatotoxicity, thrombocytopenia, and anemia. Pyrazinamide is an antitubercular medication that is given in conjunction with other antitubercular medications. Headache, nasal congestion, and difficulty sleeping are not associated with the use of this medication.

A nurse provides skin care instructions to a client with acne vulgaris. Which statement by the client indicates a need for further instruction? "I should use oil-based cosmetics." "I shouldn't leave make-up on overnight." "I should avoid rubbing my face vigorously." "I should wash my face two or three times a day with a mild cleanser."

"I should use oil-based cosmetics." Rationale: There is a need for further instruction if the client states, "I should use oil-based cosmetics." The client with acne is instructed to use water-based cosmetics and to avoid exposure to skin products that contain oil, because products that are oily may cause flare-ups. The statements in the other options are correct.

A client with peripheral vascular disease returns to the surgical care unit after having femoral-popliteal bypass grafting. Indicate in which order the nurse should conduct assessment of this client. 1. Postoperative pain. 2. Peripheral pulses. 3. Urine output. 4. Incision site.

(2,4,3,1) Because assessment of the presence and quality of the pedal pulses in the affected extremity is essential after surgery to make sure that the bypass graft is functioning, this step should be done first. The nurse should next ensure that the dressing is intact, and then that the client has adequate urine output. Lastly, the nurse should determine the client's level of pain.

A primary health care provider prescribes a dose of morphine sulfate 2.5 mg stat to be administered intravenously to a client in pain. The nurse preparing the medication notes that the label on the vial of morphine sulfate solution for injection reads "4 mg/mL." How many milliliters (mL) must the nurse draw into a syringe for administration to the client? Type the answer in the space provided. _____ mL

0.625

Following a transsphenoidal hypophysectomy, the nurse should assess the client for: 1. Cerebrospinal fluid (CSF) leak. 2. Fluctuating blood glucose levels. 3. Cushing's syndrome. 4. Cardiac arrhythmias

1 A major focus of nursing care after transsphenoidal hypophysectomy is the prevention of and monitoring for a CSF leak. CSF leakage can occur if the patch or incision is disrupted. The nurse should monitor for signs of infection, including elevated temperature, increased white blood cell count, rhinorrhea, nuchal rigidity, and persistent headache. Hypoglycemia and adrenocortical insufficiency may occur. Monitoring for fluctuating blood glucose levels is not related specifically to transsphenoidal hypophysectomy. The client will be given IV fluids postoperatively to supply carbohydrates. Cushing's disease results from adrenocortical excess, not insufficiency. Monitoring for cardiac arrhythmias is important, but arrhythmias are not anticipated following a transsphenoidal hypophysectomy.

A client is scheduled for an arteriogram. The nurse should explain to the client that the arteriogram will confirm the diagnosis of occlusive arterial disease by: 1. Showing the location of the obstruction and the collateral circulation. 2. Scanning the affected extremity and identifying the areas of volume changes. 3. Using ultrasound to estimate the velocity changes in the blood vessels. 4. Determining how long the client can walk.

1 An arteriogram involves injecting a radiopaque contrast agent directly into the vascular system to visualize the vessels. It usually involves computed tomographic scanning. The velocity of the blood flow can be estimated by duplex ultrasound. The client's ankle-brachial index is determined, and then the client is requested to walk. The normal response is little or no drop in ankle systolic pressure after exercise.

An overweight client taking warfarin (Coumadin) has dry skin due to decreased arterial blood flow. What should the nurse instruct the client to do? Select all that apply. 1. Apply lanolin or petroleum jelly to intact skin. 2. Follow a reduced-calorie, reduced-fat diet. 3. Inspect the involved areas daily for new ulcerations. 4. Instruct the client to limit activities of daily living (ADLs). 5.Use an electric razor to shave

1,2,3,5 Maintaining skin integrity is important in preventing chronic ulcers and infections. The client should be taught to inspect the skin on a daily basis. The client should reduce weight to promote circulation; a diet lower in calories and fat is appropriate. Because the client is receiving Coumadin, the client is at risk for bleeding from cuts. To decrease the risk of cuts, the nurse should suggest that the client use an electric razor. The client with decreased arterial blood flow should be encouraged to participate in ADLs. In fact, the client should be encouraged to consult an exercise physiologist for an exercise program that enhances the aerobic capacity of the body.

Which of the following is an indication of a complication of septic shock? 1. Anaphylaxis. 2. Acute respiratory distress syndrome (ARDS). 3. Chronic obstructive pulmonary disease (COPD). 4. Mitral valve prolapse.

2

A client is scheduled to have an arteriogram. During the arteriogram, the client reports having nausea, tingling, and dyspnea. The nurse's immediate action should be to: 1. Administer epinephrine. 2. Inform the physician. 3. Administer oxygen. 4. Inform the client that the procedure is almost over.

2 .Clients may have an immediate or a delayed reaction to the radiopaque dye. The physician should be notified immediately because the symptoms suggest an allergic reaction. Treatment may involve administering oxygen and epinephrine. Explaining that the procedure is over does not address the current symptoms

The nurse is evaluating a client with hyperthyroidism who is taking Propylthiouracil (PTU) 100 mg/day in three divided doses for maintenance therapy. Which of the following statements from the client indicates the desired outcome of the drug? 1. "I have excess energy throughout the day." 2. "I am able to sleep and rest at night." 3. "I have lost weight since taking this medication." 4. "I do perspire throughout the entire day."

2. PTU is a prototype of thioamide antithyroid drugs. It inhibits production of thyroid hormones and peripheral conversion of T4 to the more active T3. A client taking this antithyroid drug should be able to sleep and rest well at night since the level of thyroid hormones is reduced in the blood. Excess energy throughout the day, loss of weight and perspiring through the day are symptoms of hyperthyroidism indicating the drug has not produced its outcome.

Which nursing intervention is most important in preventing septic shock? 1. Administering IV fluid replacement therapy as prescribed. 2. Obtaining vital signs every 4 hours for all clients. 3. Monitoring red blood cell counts for elevation. 4. Maintaining asepsis of indwelling urinary catheters.

4

An adult client with chronic kidney disease who is oliguric and undergoing hemodialysis is under a fluid restriction. What percentage of the total amount of fluid can the client consume during the evening shift? 10% 20% 40% 50%

40% Rationale: When calculating how to distribute fluid to a client under fluid restriction, the nurse usually allows half or 50% of the allotted total oral fluids between 7 a.m. and 3 p.m., the period during which the client is more active, consumes two meals, and takes most of oral medications. Another two fifths(40%)is allotted to the evening shift, and the balance(10%)is allowed during the night.

A nurse is performing an assessment of a client with suspected pheochromocytoma. Which clinical manifestation does the nurse expect to note? Weight gain Flushed face Client complaint of diarrhea A blood pressure higher than the normal range

A blood pressure higher than the normal range Rationale: Pheochromocytoma is a catecholamine-producing tumor arising from cells of the adrenal medulla and sympathetic ganglia. These tumors release excessive amounts of catecholamines, mainly norepinephrine, with associated signs/symptoms that include the "five P's": pressure (paroxysmal increases in blood pressure), palpitations, pallor, perspiration (profuse and generalized), and pain (paroxysmal pulsatile headaches, chest, and abdominal pain). Other clinical signs/symptoms include weight loss, constipation, tremors, hypertensive retinopathy, hyperglycemia, and hypercalcemia.

A nurse assisting with a delivery is monitoring the client for placental separation after the delivery of a viable newborn. Which observation indicates to the nurse that placental separation has occurred? A discoid uterus Sudden sharp vaginal pain Shortening of the umbilical cord A sudden gush of dark blood from the introitus

A sudden gush of dark blood from the introitus Rationale: Placental separation occurs when the placenta separates from the uterus. Signs of placental separation include lengthening of the umbilical cord, a sudden gush of dark blood from the introitus, a firmly contracted uterus, and a change in uterine shape from discoid to globular. The client may experience vaginal fullness but sudden sharp vaginal pain is not usual.

The nurse is the first responder at the scene of a bus crash. After a quick assessment of the victims, which one does the nurse care for first? A victim with a twisted ankle and leg bruises A victim with an open fracture of the arm that is bleeding profusely A victim who is anxiously moving among the victims, searching for her husband A victim who is unresponsive, with severe swelling and bruising around the eyes, and is not breathing

A victim with an open fracture of the arm that is bleeding profusely Rationale: In a disaster situation, the nurse must triage victims on the basis of severity of injury and potential for recovery. Victims with life-threatening injuries that are readily corrected are classified as emergent and are the first priority (in this case, the victim with an open fracture of the arm that is bleeding profusely). Victims with injuries that do not require immediate treatment but that will need to be treated within 1 to 2 hours are classified as urgent and are the second priority (here, the victim with a twisted ankle and leg bruises). Victims with no injuries, those whose condition is noncritical, and victims who are ambulatory are classified as delayed (nonurgent) and are the third priority (in this case, the victim who is anxiously walking about among the victims, searching for her husband). The victim who is unresponsive and not breathing, with severely swollen and bruised eyes, is most likely dead after having sustained a severe head injury and, in this situation, is not the priority. The nurse does not have the resources necessary to save this client, whereas it may be possible for the nurse to save the client who is bleeding profusely by applying pressure to the bleeding site.

A nurse is providing morning care to a client who has undergone surgery to repair a fractured left hip. Which item is most important for the nurse to use in turning the client from side to side to change the bed linens? Trapeze bar Sliding board Adduction device Abduction device

Abduction device Rationale: After surgery to repair a fractured hip, an abduction device is the most important item used to maintain the affected extremity in good alignment and prevent dislocation. A hip abduction pillow helps prevent the hip from turning in or away from the body, and keeps the hip straight while in bed. It is placed between the thighs and attached to the legs with straps wrapped around the thighs and ankles. Some surgeons only allow the client to turn directly onto one side or the other; therefore the nurse checks the postoperative prescription. A trapeze bar may also be used, but it is not the priority item for repositioning the client. A sliding board is used to transfer a client from a bed to a stretcher or vice versa.

NPO status is imposed 8 hours before the procedure on a client scheduled to undergo electroconvulsive therapy (ECT) at 1 p.m. On the morning of the procedure, the nurse checks the client's record and notes that the client routinely takes an oral antihypertensive medication each morning. What action should the nurse take? Administer the antihypertensive with a small sip of water Withhold the antihypertensive and administer it at bedtime Administer the medication by way of the intravenous (IV) route Hold the antihypertensive and resume its administration on the day after the ECT

Administer the antihypertensive with a small sip of water Rationale: The nurse should administer the antihypertensive with a small sip of water. General anesthesia is required for ECT, so NPO status is imposed for 6 to 8 hours before treatment to help prevent aspiration. Exceptions include clients who routinely receive cardiac medications, antihypertensive agents, or histamine (H2) blockers, which should be administered several hours before treatment with a small sip of water. Withholding the antihypertensive and administering it at bedtime and withholding the antihypertensive and resuming administration on the day after the ECT are incorrect actions, because antihypertensives must be administered on time; otherwise, the risk for rebound hypertension exists. The nurse would not administer a medication by way of a route that has not been prescribed.

A nurse is providing dietary instructions to a client who is taking tranylcypromine sulfate. Which foods does the nurse tell the client to avoid while she is taking this medication? Select all that apply. Beer Apples Yogurt Baked haddock Pickled herring Roasted fresh potatoes

Beer Yogurt Pickled herring Rationale: Tranylcypromine sulfate is a monoamine oxidase inhibitor (MAOI) used to treat depression. The client must follow a tyramine-restricted diet while taking the medication to help prevent hypertensive crisis, a life-threatening effect of the medication. Foods to be avoided include meats prepared with tenderizer, smoked or pickled fish, beef or chicken liver, and dry sausages (e.g., salami, pepperoni, bologna). In addition, figs, bananas, aged cheeses, yogurt and sour cream, beer, red wine, alcoholic beverages, soy sauce, yeast extract, chocolate, caffeine, and aged, pickled, fermented, or smoked foods must be avoided. Many over-the-counter medications contain tyramine and must be avoided as well.

A nurse is assessing a client who has been taking amantadine hydrochloride for the treatment of Parkinson's disease. Which finding from the history and physical examination would cause the nurse to determine that the client may be experiencing an adverse effect of the medication? Insomnia Rigidity and akinesia Bilateral lung wheezes Orthostatic hypotension

Bilateral lung wheezes Rationale: Amantadine hydrochloride is an antiparkinson agent that potentiates the action of dopamine in the central nervous system (CNS). The medication is used to treat rigidity and akinesia. Insomnia and orthostatic hypotension are side effects of the medication. Adverse effects include congestive heart failure (evidenced by bilateral lung wheezes), leukopenia, neutropenia, hyperexcitability, convulsions, and ventricular dysrhythmias.

A nurse is monitoring a client with pheochromocytoma who is receiving an intravenous (IV) infusion of phentolamine. Which vital sign does the nurse monitor most closely during the infusion? Apical pulse Respirations Temperature Blood pressure

Blood pressure Rationale: The nurse most closely checks the client's blood pressure every 15 minutes during infusion of the medication. Pheochromocytoma is a catecholamine-producing tumor arising from cells of the adrenal medulla and sympathetic ganglia. These tumors release excessive amounts of catecholamines, mainly norepinephrine. Most affected clients present with hypertensive crises that may be treated with phentolamine. This medication, a short-acting alpha-adrenergic blocker is given by way of IV bolus or infusion for a hypertensive crisis.

A nurse is providing dietary instructions to the mother of a child with celiac disease. What does the nurse tell the mother is acceptable to give the child? Boiled rice Cooked pasta Warm oatmeal Baked macaroni and cheese

Boiled rice Rationale: The nurse tells the mother that corn, rice, and millet may be substituted as grains. Celiac disease is a disorder in which the affected person has an intolerance to wheat, rye, barley, and oats. Dietary management is the mainstay of treatment for the child with celiac disease. Products containing gluten (e.g., wheat, rye, barley, oats, hydrolyzed vegetable protein) should be eliminated from the child's diet. This includes most pasta and baked products, as well as many breakfast cereals. Vitamin supplements, especially fat-soluble vitamins and folate, may be needed in the early period of treatment to correct deficiencies.

The nurse is assessing a client who has a history of Prinzmetal's angina. The nurse knows that what type of medication is given to treat this condition? Inotropes Beta blockers ACE inhibitors Calcium channel blockers

Calcium channel blockers Rationale: Prinzmetal's, or variant, angina is prolonged and severe and occurs at the same time each day, most often at rest. The treatment of choice is usually a calcium channel blocker. Calcium channel blockers relax and dilate the vascular smooth muscle, thus relieving the coronary artery spasm in variant angina. Inotropes, beta blockers, and angiotensin-converting enzyme (ACE) inhibitors are not given to treat this disorder.

A nurse caring for a client with preeclampsia prepares for the administration of an intravenous infusion of magnesium sulfate. Which substance does the nurse ensure is readily available? Vitamin K Protamine sulfate Potassium chloride Calcium gluconate

Calcium gluconate Rationale: Calcium gluconate should be available at the bedside of a client receiving an intravenous infusion of magnesium sulfate to reverse magnesium toxicity and prevent respiratory arrest if the serum magnesium level becomes too high. Magnesium sulfate, which has anticonvulsant properties, is used for a client with preeclampsia to help prevent seizures (eclampsia). It also causes central nervous system depression, however, so toxicity is a concern. Vitamin K is the antidote for warfarin sodium (Coumadin). Protamine sulfate is the antidote for heparin. Potassium chloride is used to treat potassium deficiency.

A client arrives in the emergency department and tells the nurse that he/she is experiencing tingling in both hands and is unable to move his/her fingers. The client states that he/she has been unable to work because of the problem. During the psychosocial assessment, the client reports that 2 days earlier his/her partner said he/she wanted a separation and that he/she would have to support self financially. What problem does the nurse conclude that this client is exhibiting signs/symptoms compatible with? Severe anxiety Conversion disorder Posttraumatic stress disorder (PTSD) Obsessive-compulsive disorder

Conversion disorder Rationale: Conversion disorder is characterized by the presence of one or more signs/symptoms suggesting a neurological problem that cannot be attributed to a medical disorder. Psychological factors such as stress and conflict are associated with the onset or exacerbation of the sign/symptom. A person with severe anxiety may focus on a particular detail or many scattered details. The person may have difficulty noticing what is going on in the environment, even when it is pointed out by another. Learning and problem-solving are not possible at this level of anxiety, and the client may be dazed and confused. PTSD is characterized by repeated re-experiencing of a highly traumatic event that involved actual or threatened death or serious injury to self or others to which the individual responded with intense fear, helplessness, or horror. Obsessions are thoughts, impulses, or images that persist and recur so that they cannot be dismissed from the mind. Compulsions are ritualistic behaviors that an individual feels driven to perform in an attempt to reduce anxiety.

A nurse is providing instructions to a client with glaucoma who will be using acetazolamide daily. Which finding, an adverse effect, does the nurse instruct the client to report to the primary health care provider? Nausea Dark urine Urinary frequency Decreased appetite

Dark urine Rationale: Acetazolamide is a carbonic anhydrase inhibitor. Nephrotoxicity and hepatotoxicity may occur, manifesting as dark urine and stools, lower back pain, jaundice, dysuria, crystalluria, renal colic, and calculi. Bone marrow depression may also occur as an adverse effect. Nausea, urinary frequency, and decreased appetite are side effects of the medication.

Chlorpromazine has been prescribed to a client with Huntington's disease for the relief of choreiform movements (repetitive and rapid, jerky, involuntary movements that appear well-coordinated). Of which common side effect does the nurse warn the client? Headache Drowsiness Photophobia Urinary frequency

Drowsiness Rationale: Chlorpromazine is an antipsychotic, antiemetic, antianxiety, and antineuralgia adjunct. Common side effects of chlorpromazine include drowsiness, blurred vision, hypotension, defective color vision, impaired night vision, dizziness, decreased sweating, constipation, dry mouth, and nasal congestion. Headache, photophobia, and urinary frequency are not specific side effects of this medication.

A nurse checking the fundus of a postpartum woman notes that it is above the expected level, at the umbilicus, and that it has shifted from the midline position to the right. What initial action should the nurse take? Document the findings Encourage the woman to walk Help the woman empty her bladder Massage the fundus gently until it becomes firm

Help the woman empty her bladder Rationale: Ths initial action by the nurse is to help the woman empty her bladder. In the postpartum period, the fundus should be firmly contracted and at or near the level of the umbilicus. If the uterus is found to be higher than the expected level or shifted from the midline position (usually to the right), the bladder may be distended. The location of the fundus should be rechecked after the woman has emptied her bladder. If the fundus is difficult to locate or is boggy (soft), the nurse stimulates the uterine muscle to contract by gently massaging the uterus. Encouraging the woman to walk is inappropriate at this time. The nurse would document fundal position, consistency, and height and any other interventions taken (e.g., uterine massage) after the woman has emptied her bladder.

Captopril is prescribed for a hospitalized client with heart failure. Which action is a priority once the nurse has administered the first dose? Checking the client's apical heart rate Maintaining the client on bed rest for 3 hours Monitoring the client for increased urine output Checking the client's breath sounds for decreased wheezing

Maintaining the client on bed rest for 3 hours Rationale: The client is closely monitored for hypotension at the start of therapy and is maintained on bed rest for 3 hours after the initial dose. Captopril is an angiotensin-converting enzyme (ACE) inhibitor. Excessive hypotension (first-dose syncope) may occur in the client with heart failure or in the client who is severely salt or volume depleted. Checking the apical heart rate will provide information about the client's cardiac status but is not an intervention specifically related to this medication. Increased urine output and decreased wheezing are expected if the client has received a diuretic.

A nurse is caring for a client who has had a cast applied to the left leg and is at risk for acute compartment syndrome. For which early sign of this complication does the nurse monitor the client? Paresthesia Cold, bluish toes Weak pedal pulse Severe pain relieved by medication

Paresthesia Rationale: Sensory deficits such as paresthesia abnormal dermal sensation (e.g., a tingling, pricking, chilling, burning, or numb sensation on the skin) with no apparent physical cause generally appear as an early sign of acute compartment syndrome before changes in vascular or motor signs. Acute compartment syndrome is a serious condition in which increased pressure within one or more compartments causes massive compromise of circulation to the area. Resultant edema causes pressure on the nerve endings and subsequent pain. The client will also complain of severe diffuse pain that is not relieved by analgesics.

A nurse is developing a plan of care for a client admitted to the nursing unit with a diagnosis of paranoid personality disorder. On which characteristic of the disorder does the nurse base the plan of care? Inflexible and rigid Self-sacrificing and submissive Highly critical of self and others Projecting blame, possibly becoming hostile

Projecting blame, possibly becoming hostile Rationale: A client with paranoid personality disorder projects blame, is suspicious of others, and may become hostile or violent. The client also experiences cognitive or perceptual distortions. A client who is inflexible and rigid and is highly critical of self and others is showing signs/symptoms of obsessive-compulsive disorder. Being self-sacrificing and submissive is a characteristic of a client with dependent personality disorder.

The nurse administers intravenous morphine sulfate to a client in pulmonary edema. For which intended effect of the medication does the nurse monitor the client? Relief of pain Relief of anxiety Decreased urine output Increased blood pressure

Relief of anxiety Rationale: Morphine sulfate reduces anxiety in the client in pulmonary edema. It blunts the sympathetic response and increases venous capacitance, thereby decreasing left atrial pressure. It also promotes peripheral vasodilation and causes blood to pool in the periphery. The client receiving morphine sulfate is monitored for signs/symptoms of respiratory depression and extreme decreases in blood pressure, especially when the medication is administered intravenously. Although morphine sulfate is an opioid analgesic and relieves pain, it is not administered to the client with pulmonary edema for its analgesic effect. Furosemide is administered to the client with pulmonary edema to increase urine output.

A nurse reviews arterial blood gas (ABG) values and notes a pH of 7.50 and a Pco2 of 30 mm Hg. What does the nurse interpret these values as indicative of? Metabolic acidosis Metabolic alkalosis Respiratory acidosis Respiratory alkalosis

Respiratory alkalosis Rationale: The nurse interprets these values as indicative of respiratory alkalosis. The normal pH is 7.35 to 7.45. The normal Pco2 is 35 to 45 mm Hg. In respiratory alkalosis, the pH will be higher than normal and the Pco2 will be low.

A nurse is performing an assessment of a client being admitted to the hospital with a diagnosis of multiple sclerosis (MS). The client tells the nurse that she took baclofen for the past 9 months but completely stopped the medication 2 days ago because it was making her feel weak. On the basis of this information, the nurse notes in the plan of care what should be monitored most closely in the client? Spasticity Drowsiness Muscle spasms Seizure activity

Seizure activity Rationale: The nurse should most closely monitor the client for seizure activity. Abrupt withdrawal of baclofen may produce hallucinations or seizures. Baclofen is a skeletal muscle relaxant, acting at the level of the spinal cord to reduce the frequency and amplitude of muscle spasms and spasticity in clients with spinal cord injuries and diseases, as well as multiple sclerosis. Drowsiness may occur as a result of taking the medication but diminishes with continued therapy.

Calcium carbonate is prescribed for a client with mild hypocalcemia. What food does the nurse instruct the client to avoid consuming while taking this medication? Fish Milk Spinach Watermelon

Spinach Rationale: The nurse instructs the client to avoid consuming spinach while taking calcium carbonate. Oral calcium salts are used to treat mild hypocalcemia and to supplement dietary calcium. The client is instructed to take oral calcium with a large glass of water with or after a meal to promote absorption. The client is also instructed to avoid taking calcium with foods that can suppress calcium absorption. Such foods include not only spinach, but Swiss chard, beets, bran, and whole-wheat cereals. The client does not need to avoid fish, milk, or watermelon.

A nurse provides instruction to a pregnant woman about foods containing folic acid. Which of these foods does the nurse tell the client to consume as sources of folic acid? Select all that apply. Bananas Potatoes Spinach Legumes Whole grains Milk products

Spinach Legumes Whole grains Rationale: Some foods high in folic acid are glandular meats, yeast, dark-green leafy vegetables, legumes, and whole grains. Folic acid is needed during pregnancy for healthy cell growth and repair. A pregnant woman should have at least four servings of folic acid-rich foods per day. Bananas provide potassium. Potatoes provide vitamin B6, and milk products are a source of calcium.

A nurse is caring for a client who sustained a missed abortion during the second trimester of pregnancy. For which finding indicating the need for further evaluation does the nurse monitor the client? Spontaneous bruising Decrease in uterine size Urine output of 30 mL/hr Brownish vaginal discharge

Spontaneous bruising Rationale: A major complication of a missed abortion is disseminated intravascular coagulation (DIC). Bleeding at the sites of intravenous needle insertion or laboratory blood draws, nosebleeds, and spontaneous bruising may be early indicators of DIC; they should be reported and require further evaluation. Missed abortion is the term used to describe when a fetus dies during the first half of pregnancy but is retained in the uterus. When the fetus dies, the early signs/symptoms of pregnancy (e.g., nausea, breast tenderness, urinary frequency) disappear. The uterus stops growing and begins to shrink. Red or brownish vaginal bleeding may or may not occur.

A nurse is providing information about the storage of insulin to a client who will be self-administering regular insulin. What does the nurse tell the client? That placing the vial near heat or in sunlight will not affect the insulin To freeze unopened vials and remove a vial from the freezer 24 hours before opening it That insulin stored at room temperature causes more discomfort on injection than does cold insulin That the vial in current use may be kept at room temperature for as long as 1 month without significant loss of activity

That the vial in current use may be kept at room temperature for as long as 1 month without significant loss of activity Rationale: Insulin preparations are stable at room temperature for as long as 1 month without significant loss of chemical activity. Direct sunlight and extreme heat must be avoided. Insulin should not be frozen. If the insulin is frozen, the insulin should be discarded and the nurse should obtain another vial. Insulin stored at room temperature causes less discomfort on injection than does cold insulin.

A client undergoing therapy with carbidopa/levodopa calls the nurse at the clinic and reports that his urine has become darker since he started taking the medication. What should the nurse tell the client? To call his primary health care provider That he needs to drink more fluids That this is an occasional side effect of the medication That this may be a sign/symptom of developing toxicity of the medication

That this is an occasional side effect of the medication Rationale: Carbidopa/levodopa, an antiparkinson agent, may cause darkening of the urine or sweat. The client should be reassured that this is a harmless side effect of the medication and that the medication's use should be continued. Although fluid intake is important, telling the client that he needs to drink more fluid is incorrect and unnecessary. Telling the client that the darkening of his urine may signal developing medication toxicity is incorrect and might alarm the client unnecessarily. There is no need for the client to call the primary health care provider.

A client has a primary health care provider's appointment to get a prescription for sildenafil. The nurse obtains the health history from the client. Which finding indicates that the medication is contraindicated? The client has type 2 diabetes mellitus. The client has a history of renal calculi. The client is taking glargine insulin. The client takes isosorbide dinitrate

The client takes isosorbide dinitrate . Rationale: Sildenafil is used to treat male erectile dysfunction. It is contraindicated in the client who is using isosorbide dinitrate, a form of sodium nitroprusside or organic nitrates in any form, because both sildenafil and nitrates promote hypotension. When these medications are combined, life-threatening hypotension may result. Type 2 diabetes mellitus, a history of renal calculi, and the use of glargine insulin are not contraindications to the use of sildenafil.

Testing of the plasma theophylline level in a client who is receiving a continuous intravenous infusion of theophylline reveals a toxic level of 24 mcg/mL (111 umol/L). What early signs of toxicity does the nurse assess for? Select all that apply. Tremors Lethargy Bradycardia Palpitations Nervousness

Tremors Palpitations Nervousness Rationale: Theophylline toxicity is likely to occur when the serum level is higher than 20 mcg/mL (111 mcmol/L). Early signs of toxicity include restlessness, not lethargy; nervousness; tremors; palpitations; and tachycardia, not bradycardia. The normal therapeutic range for theophylline is 10 to 20 mcg/mL (55.5 to 111 umol/L).

A nurse is caring for a client who has undergone transsphenoidal hypophysectomy to remove a microadenoma of the pituitary gland. Which finding would be of greatest concern to the nurse? Urinary specific gravity is low Blood pressure is 138/80 mm Hg. The client complains of a dry mouth. The client frequently performs deep-breathing exercises.

Urinary specific gravity is low Rationale: After transsphenoidal hypophysectomy, the client is monitored for transient diabetes insipidus. In a client with diabetes insipidus, the greatest concern for the nurse is low urinary specific gravity and excessive urine output. A blood pressure of 138/80 mm Hg is not cause for concern. The client will have nasal packing and a mustache dressing and may complain of dry mouth because of the necessity for mouth breathing. The nurse would provide frequent oral rinses and apply petroleum jelly to dry lips. The client should perform frequent deep-breathing exercises (coughing is contraindicated) to help prevent pulmonary complications.


Ensembles d'études connexes

1.3 Insurer Domicile and Authorization

View Set

A Midsummer Night's Dream - Hermia

View Set

Security Pro Practice Questions Chapter 6

View Set

intermediate financial account chapter 9

View Set

introduction to SPC (statistical Process Control)

View Set